Download as pdf or txt
Download as pdf or txt
You are on page 1of 238

Syllabus

Course Structure Class – X


(Annual Examination)

Unit No. Units Name Marks: 80


I Number Systems 06
II Algebra 20
III Coordinate Geometry 06
IV Geometry 15
V Trigonometry 12
VI Mensuration 10
VII Statistics & Probability 11
Total 80

UNIT I: NUMBER SYSTEMS


1. REAL NUMBERS (15) Periods



Fundamental Theorem of Arithmetic - statements after reviewing work done earlier and after

illustrating and motivating through examples, Proofs of irrationality of 2, 3, 5.

UNIT II: ALGEBRA


1. POLYNOMIALS (8) Periods



Zeros of a polynomial. Relationship between zeros and coefficients of quadratic polynomials.

2. PAIR OF LINEAR EQUATIONS IN TWO VARIABLES (15) Periods



Pair of linear equations in two variables and graphical method of their solution, consistency/
inconsistency.
Algebraic conditions for number of solutions. Solution of a pair of linear equations in two variables
algebraically - by substitution, by elimination. Simple situational problems.

3. QUADRATIC EQUATIONS (15) Periods





2
Standard form of a quadratic equation ax + bx + c = 0, (a ≠ 0). Solutions of quadratic equations
(only real roots) by factorization, and by using quadratic formula. Relationship between discriminant
and nature of roots.
Situational problems based on quadratic equations related to day to day activities to be incorporated.

4. ARITHMETIC PROGRESSIONS (10) Periods



th
Motivation for studying Arithmetic Progression Derivation of the n term and sum of the first n terms

of A.P. and their application in solving daily life problems.
UNIT III: COORDINATE GEOMETRY
Coordinate Geometry (15) Periods


Review: Concepts of coordinate geometry, graphs of linear equations. Distance formula. Section

formula (internal division).

UNIT IV: GEOMETRY


1. TRIANGLES (15) Periods



Definitions, examples, counter examples of similar triangles.
1. (Prove) If a line is drawn parallel to one side of a triangle to intersect the other two sides in
distinct points, the other two sides are divided in the same ratio.
2. (Motivate) If a line divides two sides of a triangle in the same ratio, the line is parallel to the third
side.
3. (Motivate) If in two triangles, the corresponding angles are equal, their corresponding sides are
proportional and the triangles are similar.
4. (Motivate) If the corresponding sides of two triangles are proportional, their corresponding
angles are equal and the two triangles are similar.
5. (Motivate) If one angle of a triangle is equal to one angle of another triangle and the sides
including these angles are proportional, the two triangles are similar.

2. CIRCLES (10) Periods





Tangent to a circle at, point of contact

1. (Prove) The tangent at any point of a circle is perpendicular to the radius through the point of

contact.
2. (Prove) The lengths of tangents drawn from an external point to a circle are equal.

UNIT V: TRIGONOMETRY
1. INTRODUCTION TO TRIGONOMETRY (10) Periods



Trigonometric ratios of an acute angle of a right-angled triangle. Proof of their existence (well

defined); motivate the ratios whichever are defined at 0° and 90°. Values of the trigonometric ratios
of 30°, 45° and 60°. Relationships between the ratios.

2. TRIGONOMETRIC IDENTITIES (15) Periods





2 2
Proof and applications of the identity sin A + cos A = 1. Only simple identities to be given.

3. HEIGHTS AND DISTANCES: Angle of elevation, Angle of Depression. (10) Periods



Simple problems on heights and distances. Problems should not involve more than two right

triangles. Angles of elevation / depression should be only 30°, 45°, and 60°.
UNIT VI: MENSURATION
1. AREAS RELATED TO CIRCLES (12) Periods



Area of sectors and segments of a circle. Problems based on areas and perimeter / circumference
of the above said plane figures. (In calculating area of segment of a circle, problems should be
restricted to central angle of 60°, 90° and 120° only.

2. SURFACE AREAS AND VOLUMES (12) Periods





Surface areas and volumes of combinations of any two of the following: cubes, cuboids, spheres,

hemispheres and right circular cylinders/cones.

UNIT VII: STATISTICS AND PROBABILITY


1. STATISTICS (18) Periods



Mean, median and mode of grouped data (bimodal situation to be avoided).

2. PROBABILITY (10) Periods



Classical definition of probability. Simple problems on finding the probability of an event.

The changes for classes IX-X (2022-23) internal year-end/
Board Examination are as detailed:

(Classes IX-X)

Year End (2021-22) Existing


Examination/ (As per Special Scheme of (2022-23)
Board Assessment for Board Examination – Modified
Examination Circular No. (Annual Scheme)
(Theory) Acad-51/2021 dated 05.07.2021)

u Competency Based Questions would be


Term I – Multiple Choice Question

u
minimum 40%

including case based and assertion
These can be in the form of Multiple Choice
reasoning type MCQs – 100% (30%

Questions, Case based Questions, Source
questions competency based)
Composition based Integrated Questions or any other
u Term II – Case based/ Situation based, types.

Open Ended- short answer/long answer u Objective Type Questions will be 20%
questions (30% questions competency

u Remaining 40% short answer/long answer
based)

questions (as per existing pattern)

Internal Assessment : No change


Internal Assessment : End of year examination = 20:80
Design of Question Paper
MATHEMATICS-Standard
CLASS – X (2022-23)

Max. Marks: 80

%
S. Total
Typology of Questions Weightage
No. Marks
(approx.)
Remembering: Exhibit memory of previously learned
material by recalling facts, terms, basic concepts, and
answers.
1. 43 54
Understanding: Demonstrate understanding of facts
and ideas by organizing, comparing, translating,
interpreting, giving descriptions, and stating main ideas
Applying: Solve problems to new situations by applying
2. acquired knowledge, facts, techniques and rules in a 19 24
different way.
Analysing: Examine and break information into parts by
identifying motives or causes. Make inferences and find
evidence to support generalizations
Evaluating: Present and defend opinions by making
3. judgments about information, validity of ideas, or quality 18 22
of work based on a set of criteria.
Creating: Compile information together in a different
way by combining elements in a new pattern or
proposing alternative solutions
Total 80 100

INTERNAL ASSESSMENT 20 Marks



 Pen Paper Test and Multiple Assessment (5+5) 10 Marks


 Portfolio 05 Marks



 Lab Practical


(Lab activities to be done from the prescribed books) 05 Marks

EXAM BITES

This Pdf Is
Downloaded From
www.exambites.in

Visit www.exambites.in for


More Premium Stuffs,Latest
Books,Test Papers,Lectures etc.
jeeneetadda
jna_official
jeeneetadda

VISIT NOW !!
BASIC CONCEPTS—
A FLOW CHART

Chapter-1: Real Numbers

rn bn

* Now not in syllabus.

Sample Papers 5
Chapter-2: Polynomials

* Now not in syllabus.

6 Mathematics–X
Sample Papers 7
Chapter-3: Pair of Linear Equations in Two Variables

8 Mathematics–X
*

* Now not in syllabus.

Sample Papers 9
Chapter-4: Quadratic Equations

10 Mathematics–X
Chapter-5: Arithmetic Progressions

Sample Papers 11
Chapter-6: Coordinate Geometry

* Now not in syllabus.

12 Mathematics–X
Chapter-7: Triangles

* Now not in syllabus.

Sample Papers 13
Chapter-8: Circles

* Alternate Segment Theorem


An angle between a tangent and a chord E
through the point of contact is equal to the
angle in the alternate segment. D
Hence, BCE = CDE
and, ACD = DEC
are the alternate segment angles. A C B

* Now not in syllabus.

14 Mathematics–X
Chapter-9: Introduction to Trigonometry

* Now not in syllabus.

Sample Papers 15
Chapter-10: Heights and Distances

16 Mathematics–X
Chapter-11: Areas Related to Circles

BQA

OAPB
OBQA

Sample Papers 17
Chapter-12: Surface Areas and Volumes

* Now not in syllabus.

18 Mathematics–X
I

c b

(if hemisphere is solid)

* Now not in syllabus.

Sample Papers 19
Chapter-13: Statistics

* Now not in syllabus.

20 Mathematics–X
*

* *

* Now not in syllabus.

Sample Papers 21
Chapter-14: Probability

zzz

22 Mathematics–X
CBSE SAMPLE QUESTION
PAPER (STANDARD)-2023 [SOLVED]

Time allowed: 3 hours Maximum marks: 80


General Instructions:

(i) This Question Paper has 5 Sections A-E.


(ii) Section A has 20 MCQs carrying 1 mark each.


(iii) Section B has 5 questions carrying 02 marks each.


(iv) Section C has 6 questions carrying 03 marks each.


(v) Section D has 4 questions carrying 05 marks each.


(vi) Section E has 3 case based integrated units of assessment (04 marks each) with sub-parts of


the values of 1, 1 and 2 marks each respectively.


(vii) All Questions are compulsory. However, an internal choice in 2 Questions of 5 marks,


2 Question of 3 marks and 2 Questions of 2 marks has been provided. An internal choice has
been provided in the 2 marks questions of Section E.
(viii) Draw neat figures wherever required. Take π = 22/7 wherever required if not stated.

SECTION-A
Section A consists of 20 questions of 1 mark each.
1. Let a and b be two positive integers such that a = p3q4 and b = p2q3, where p and q are

prime numbers. If HCF (a, b) = pmqn and LCM (a, b) = prqs, then (m + n)(r+s) = 1 

(a) 15 (b) 30 (c) 35 (d) 72

2. Let p be a prime number. The quadratic equation having its roots as factors of p is 1


(a) x2 – px + p=0
(b) x2 – (p+1)x + p=0

2
(c) x + (p + 1) x + p = 0
(d) x2 – px + p + 1=0

3. If α and β are the zeros of a polynomial f(x) = px2 – 2x + 3p and α + β = αβ, then p is

 1
–1
(a) 15 (b) 30 (c) 35 (d)
3

4. If the system of equations 3x + y = 1 and (2k – 1)x + (k – 1) y = 2k + 1 is inconsistent,


then k =  1
(a) –1
(b) 0
(c) 1
(d) 72

Sample Papers 23
5. If the vertices of a parallelogram PQRS taken in order are P(3, 4), Q(–2, 3) and

R(–3, –2), then the coordinates of its fourth vertex S are 1

(a) (–2, –1)


(b) (–2, –3)
(c) (2, –1)
(d) (1, 2)

6. ΔABC ~ ΔPQR. If AM and PN are altitudes of ΔABC and ΔPQR respectively and
AB2 : PQ2 = 4 : 9, then AM: PN = 1
(a) 3:2
(b) 16:81
(c) 4:9
(d) 2:3

7. If x tan 60°cos 60° = sin 60°cot 60°, then x =


1
(a) 15 (b) 30
(c) 35
(d) 72

8. If sinθ + cosθ =

2 , then tanθ + cot θ = 1
(a) 1 (b) 2
(c) 3
(d) 4

9. In the given figure, DE  BC, AE = a units, EC = b units, DE = x units and BC = y


units. Which of the following is true? 1


A

D E

B C
a+b ax ay x a
(a) x = (b) y = (c) x = (d) =
ay a+b a+b y b

10. ABCD is a trapezium with AD  BC and AD = 4 cm. If the diagonals AC and BD intersect

each other at O such that AO/OC = DO/OB = 1/2, then BC = 1

(a) 6 cm
(b) 7 cm
(c) 8 cm
(d) 9 cm

11. If two tangents inclined at an angle of 60° are drawn to a circle of radius 3 cm, then the

length of each tangent is equal to 1


3 3
(a) cm (b) 3 cm (c) 6 cm (d) 3 3 cm
2

12. The area of the circle that can be inscribed in a square of 6 cm is


1
(a) 36 π cm2
(b) 18 π cm2
(c) 12 π cm2
(d) 9 π cm2

13. The sum of the length, breadth and height of a cuboid is 6 3 cm and the length of its

diagonal is 2 3 cm. The total surface area of the cuboid is 1

2 2 2
(a) 48 cm
(b) 72 cm
(c) 96 cm
(d) 108 cm2

14. If the difference of mode and median of a data is 24, then the difference of median and

mean is 1
(a) 8 (b) 12
(c) 24
(d) 36

15. The number of revolutions made by a circular wheel of radius 0.25 m in rolling a

distance of 11 km is 1
(a) 2800
(b) 4000
(c) 5500
(d) 7000

24 Mathematics–X
16. For the following distribution,
1
Class 0–5 5 – 10 10 – 15 15 – 20 20 – 25
Frequency 10 15 12 15 9
the sum of the lower limits of the median and modal class is
(a) 15
(b) 25
(c) 30
(d) 35

17. Two dice are rolled simultaneously. What is the probability that 6 will come up at least

once? 1
1 7 11 13
(a) (b) (c) (d)
6 36 36 36

5 sin b – 2 cos b
18. If 5 tan = 4 then = 1
5 sin b + 2 cos b
q

1 2 3
(a) (b) (c) (d) 6
3 5 5

DIRECTION: In the question numbers 19 and 20, a statement of assertion (A) is followed by a
statement of Reason (R). Choose the correct option.
(a) Both A and R are true and R is the correct explanation for A.

(b) Both A and R are true and R is not the correct explanation for A.

(c) A is true but R is false.


(d) A is false but R is true.


19. Statement (A) : If product of two numbers is 5780 and their HCF is 17, then their LCM

is 340.
Statement (R) : HCF is always a factor of LCM.

20. Statement(A) : If the co-ordinates of the mid-points of the sides AB and AC of ΔABC are

D(3,5) and E(–3, –3) respectively, then BC = 20 units.


Statement(R) : The line joining the mid points of two sides of a triangle is parallel to the

third side and equal to half of it.

SECTION-B
Section B consists of 5 questions of 2 marks each.
21. If 49x + 51y = 499, 51x + 49y = 501, then find the value of x and y.
2
AD AC
22. In the given figure below, = and ∠1 = ∠2. Show that Δ BAE~ ΔCAD. 2
AE BD

1 2
A D
E

23. In the given figure, O is the centre of circle. Find ∠AQB, given that PA and PB are

tangents to the circle and ∠APB = 75°.

Sample Papers 25
A

P O Q

24. The length of the minute hand of a clock is 6 cm. Find the area swept by it when it moves

from 7:05 pm. to 7:40 pm. 2


OR
In the given figure, arcs have been drawn of radius 7cm each with vertices A, B, C and D
of quadrilateral ABCD as centres. Find the area of the shaded region. 2
A

B C

25. If sin(A + B) = 1 and cos(A – B)= √3/2, 0°< A + B ≤ 90° and A> B, then find the

measures of angles A and B. 2


OR
cos i – sin i 1– 3
Find an acute angle θ when = .
cos i + sin i 1 + 3

SECTION-C
Section C consists of 6 questions of 3 marks each.
26. Given that
3 is irrational, prove that 5 + 2 3 is irrational. 3
2
27. If the zeroes of the polynomial x + px + q are double in value to the zeroes of the

polynomial 2 x2 – 5x –3, then find the values of p and q. 3


28. A train covered a certain distance at a uniform speed. If the train would have been

6 km/h faster, it would have taken 4 hours less than the scheduled time. And, if the train
were slower by 6 km/h; it would have taken 6 hours more than the scheduled time. Find
the length of the journey. 3
OR
Anuj had some chocolates, and he divided them into two lots A and B. He sold the first
lot at the rate of `2 for 3 chocolates and the second lot at the rate of `1 per chocolate,
and got a total of `400. If he had sold the first lot at the rate of `1 per chocolate, and the
second lot at the rate of `4 for 5 chocolates, his total collection would have been `460.
Find the total number of chocolates he had.

26 Mathematics–X
29. Prove the following that
3
tan3 i cot2 i
+ = sec i cosec i – 2 sin i cos i
1 + tan2 i 1 + cot2 i

30. Prove that a parallelogram circumscribing a circle is a rhombus.


3
OR
In the figure XY and X'Y' are two parallel tangents to a circle with centre O and another
tangent AB with point of contact C interesting XY at A and X'Y' at B, what is the measure
of ∠AOB.
X P A Y

O C

X′ Q B Y′

31. Two coins are tossed simultaneously. What is the probability of getting
3
(a) at least one head?
(b) at most one tail?

(c) a head and a tail?


SECTION-D
Section D consists of 4 questions of 5 marks each.
32. To fill a swimming pool two pipes are used. If the pipe of larger diameter used for 4

hours and the pipe of smaller diameter for 9 hours, only half of the pool can be filled.
Find, how long it would take for each pipe to fill the pool separately, if the pipe of
smaller diameter takes 10 hours more than the pipe of larger diameter to fill the pool?

 5
OR
In a flight of 600 km, an aircraft was slowed down due to bad weather. Its average speed
for the trip was reduced by 200 km/h from its usual speed and the time of the flight
increased by 30 min. Find the scheduled duration of the flight.
33. Prove that if a line is drawn parallel to one side of a triangle intersecting the other two

sides in distinct points, then the other two sides are divided in the same ratio.
Using the above theorem prove that a line through the point of intersection of the
diagonals and parallel to the base of the trapezium divides the non parallel sides in the
same ratio. 5
34. Due to heavy floods in a state, thousands were rendered homeless. 50 schools collectively

decided to provide place and the canvas for 1500 tents and share the whole expenditure
equally. The lower part of each tent is cylindrical with base radius 2.8 m and height 3.5
m and the upper part is conical with the same base radius, but of height 2.1 m. If the
canvas used to make the tents costs `120 per m2, find the amount shared by each school
to set up the tents. 5

Sample Papers 27
OR
There are two identical solid cubical boxes of side 7 cm. From the top face of the first cube
a hemisphere of diameter equal to the side of the cube is scooped out. This hemisphere
is inverted and placed on the top of the second cube’s surface to form a dome. Find
(i) the ratio of the total surface area of the two new solids formed.

(ii) volume of each new solid formed.


35. The median of the following data is 525. Find the values of x and y, if the total frequency

is 100 5
Class Frequency
0 – 100 2
100 – 200 5
200 – 300 x
300 – 400 12
400 – 500 17
500 – 600 20
600 – 700 y
700 – 800 9
800 – 900 7
900 – 1000 4

SECTION-E
Case study based questions are compulsory.
36. A tiling or tessellation of a flat surface is

the covering of a plane using one or more


geometric shapes, called tiles, with no overlaps
and no gaps. Historically, tessellations were
used in ancient Rome and in Islamic art. You
may find tessellation patterns on floors, walls,
paintings etc. Shown below is a tiled floor in the
archaeological Museum of Seville, made using
squares, triangles and hexagons.
A craftsman thought of making a floor pattern
after being inspired by the above design. To
ensure accuracy in his work, he made the
pattern on the Cartesian plane. He used regular
octagons, squares and triangles for his floor
tessellation pattern

28 Mathematics–X
14
R Q
13
S P
12
11
T 10 O
F E
9

U G 8 N
D
7
6
5
V H C M
4
3
W A B
2 L
1
X 0
K
–1
0 –9 –8 –7 –6 –5 –4 –3 –2 –1 1J 2 3 4 5 6 7 8
I –2
–3
–4

Use the above information to answer the questions that follow:


(i) What is the length of the line segment joining points B and F?
1
(ii) The centre ‘Z’ of the figure will be the point of intersection of the diagonals of

quadrilateral WXOP. Then what are the coordinates of Z? 1

(iii) What are the coordinates of the point on y-axis equidistant from A and G?
2
OR
What is the area of trapezium AFGH?
37. The school auditorium was to be constructed to accommodate at least 1500 people.

The chairs are to be placed in concentric circular arrangement in such a way that each
succeeding circular row has 10 seats more than the previous one.

Use the above information to answer the questions that follow:


(i) If the first circular row has 30 seats, how many seats will be there in the 10th row?1

(ii) For 1500 seats in the auditorium, how many rows need to be there?
 2
OR
If 1500 seats are to be arranged in the auditorium, how many seats are still left to be
put after 10th row?
(iii) If there were 17 rows in the auditorium, how many seats will be there in the middle

row?  1

Sample Papers 29
38. We all have seen the airplanes flying in the sky but might have not thought of how they

actually reach the correct destination. Air Traffic Control (ATC) is a service provided
by ground-based air traffic controllers who direct aircraft on the ground and through
a given section of controlled airspace, and can provide advisory services to aircraft in
non-controlled airspace. Actually, all this air traffic is managed and regulated by using
various concepts based on coordinate geometry and trigonometry.

At a given instance, ATC finds that the angle of elevation of an airplane from a point on
the ground is 60°. After a flight of 30 seconds, it is observed that the angle of elevation
changes to 30°. The height of the plane remains constantly as 3000 3 m.
Use the above information to answer the questions that follow:
(i) Draw a neat labelled figure to show the above situation diagrammatically.
1
(ii) What is the distance travelled by the plane in 30 seconds?
2
OR
Keeping the height constant, during the above flight, it was observed that after
15( 3 –1) seconds, the angle of elevation changed to 45°. How much is the distance
travelled in that duration.
(iii) What is the speed of the plane in km/h?
1

zzz

30 Mathematics–X
SECTION-A
3 4 2 3
1. (c) a = p q , b = p q

So, LCM (a, b) = p3q4 and HCF (a, b) = p2q3


But LCM (a, b) = pmqn and HCF (a, b) = prqs
Now (m + n) +(r + s) = (3 + 4)(2 + 3) = 7 × 5 = 35 1
2. (b)  p is a prime number, so its factors are 1 and p itself.

so, a = 1, and b =p
General form of quadratic equation is
x2 – (sum of zeros) x + product of zeros
⇒ x2 – (1 + p) x + p is the required quadratic equation. 1
2
3. (b) Given polynomial is f(x) = px – 2x + 3p.
–b – ]–2g 2

c 3p
So a+ b= a = = and a.b = a = =3
p p p

2 2
 a + b = ab & p
=3 & p=
3
1

4. (d) Given equations are 3x + y = 1 and (2k –1) x + (k – 1) y = 2k + 1.


a1 b1 c1
 They are inconsistent, so = =
a2 b2 c2

3 1 1
⇒ = =
2k – 1 k –1 2k + 1

Taking first two, we get


3(k – 1) = 1 (2k – 1) ⇒ 3k – 3 = 2k – 1 = k = 2
1
5. (c)  Diagonals of parallelogram bisect each other


S R(–3, –2)
so O is the middle point. Considering PR the
coordinates are
3 + ] –3 g 4 + ] –2 g O
d , n = ^0, 1h
2 2

Let coordinates of S be (x, y).


x + ]–2g

y+3
So =0 and =1 P(3, 4) Q(–2, 3)
2 2

⇒ x = 2 and y = –1 i.e., coordinate is (2, –1). 1


6. (c) 
∆ABC ∼ ∆PQR (Given)
∴ Ratio of squares of sides = ratio of squares of corresponding attitude.
4 AM2 AM 4 2

9
= & PN
= =
9 3
1
PN2

Sample Papers 31
7. (d) Given expression is

x tan 60° cos 60° = sin 60° cot 60°


1 3 1
⇒ x× 3 × = ×
2 2 3

3 1 1
⇒ x×
2
=
2
& DC = = tan 30° 1
3

8. (b) Given sin i + cos i = 2


Squaring both sides, we get


sin2 i + cos2 i + 2 sin i cos i = 2
1
⇒ 1 + 2 sin i cos i = 2 & sin i cos i = 2 ...(i)
sin i cos i sin 2 i + cos 2 i
Now tan i + cot i = + =
cos i sin i sin i cos i

1
= =2 [From (i)] 1
sin i cos i

9. (b) In ∆ ABC,

A
 DE  BC (Given) a

AE DE a x D E

AC
=
BC
& a+ b
= y x b

ay
& x=
a+b

B y C 1
B C
AO DO 1
10. (c)  = = (Given)
OC OB 2

and ∠BOC = ∠AOD (Vertical opp. angles) O


D BDC ∼ D AOD      (By SSS similarity)
A D
AD 1
=
BC 2
& BC = 4 × 2 = 8 cm   [ AD = 4 cm]  1

11.
(c) 
OP bisects ∠P. N

∴ ∠OPN = 30°
In ∆ OPN, O
30°
P
ON
tan 30° =
PN

1 3 M
= & PN = 3 3 cm 1
3 PN


6 cm
12. (d) Length of sides of square = 6 cm

So, radius of circle = 3 cm


Now, area of circle= r2 p


O
3 cm 1
2 2
= p × (3) = 9 cm p

32 Mathematics–X
13.
(c) Let l, b, h be the length, breadth and height of cuboid.

∴ l + b + h = 6 3 and length of diagonal = l2 + b2 + h2 = 2 3


⇒ l2 + b2 + h2 = 12 (squaring both sides)
2 2 2
l + b + h = 2(lb + bd + lh)
= (l + b + h)2 – (l2 + b2 + h2)
= ^6 3 h – 12

= 108 – 12 = 96 cm2 

1
14. (b) mode – median = 24 (given)

 mode = 3 median – 2 mean


⇒ mode = 2 median + median – 2 mean


⇒ mode – median = 2 (median – mean)

24 = 2 (median – mode)
24
⇒ median – mode = = 12 1
2


15. (d) Radius of wheel = 0.25 m


22

Circumference of wheel = 2 × × 0.25


7

distance covered
Now, number of revolution =
circumference of wheel

11×1000
= = 7000 1
22


2× × 0.25
7
16. (b) 
20 is the highest frequency.
∴ Modal class = 15 – 20 i.e., lower limit = 15 ... (i)
66
Again sum of frequency = 10 + 15 + 12 + 20 + 9 = = 33
2

By calculating cumulative frequency the median class


= 10 – 15 i.e., lower limit = 10 ...(ii)
By adding (i) and (ii), we get
= 15 + 10 = 25, which is sum of lower limit of mode and median.  1
17. (c) When two dice are rolled then

total events = 6 × 6 = 36
Favourable events = (1, 6), (2, 6), (3, 6), (4, 6), (5, 6), (6, 6), (6, 1), (6, 2), (6, 3), (6, 4),


(6, 5) = 11

Favourable event 11
So, the probability = = . 1
Total event 36


4
18. (a) tan b = (Given)
5

5 sin b – 2 cos b
Now given expression
5 sin b + 2 cos b

Sample Papers 33
Divide each term by cos , we get
b

5× 4 20 10
5 tan b – 2 –2 –2
5 5 5 10 1
= = = = = 1
5 tan b + 2 5× 4 20 30 30 3


+2 +2
5 5 5
Product of two numbers
19. (b) LCM =
HCF

5780
= = 340 1
17
   

So assertion is correct.
Also HCF is a factor of LCM, so reason is also correct but explanation of assertion
is incorrect.

20. (a) Length of DE = _ x2 – x1 i + _ y2 – y1 i


2 2

  = ]– 3 – 3g2 + ]– 3 – 5g2 D (3, 5) E (3, 3)

  = 36 + 64 = 100 = 10 units  1
As per the reason BC = 2DE
  = 2 × 10 = 20 cm B C

So assertion is correct and reason is also correct with correct explanation.

SECTION-B
21. Given equations are 49x + 51y = 499 and 51x + 49y = 50
 ½
Adding the two equations and dividing by 10, we get : x + y = 10    ...(i) ½
Subtracting the two equations and dividing by –2, we get : x – y = 1    ...(ii)

Adding equations (i) and (ii), we get

11
2x = 11 & x=
2
 ½
Again by subtracting equation (ii) from equation (i), we get

9
2y = 9 & y=
2
 ½
22. In ΔABC,

∠1 = ∠2 (Given)  ½
∴ AB = BD …(i) (Equal angles have equal opposite sides)
Given, C

AD AC B
=
AE BD
 ½
Using equation (i), we get 1 2

A D
E
AD AC
= ...(ii)
AE AB

34 Mathematics–X
In ΔBAE and ΔCAD, by equation (ii),
AC AD
=
AB AE
 ½
∠A = ∠A (common)  ½
∴ ΔBAE ~ ΔCAD [By SAS similarity criterion]
23. ∠PAO = ∠ PBO = 90°
(angle between radius and tangent)  ½
From figure in quadrilateral PAOB  ½
A

P O Q

∠AOB = 105° (By angle sum property of a quadrilateral)  ½


1
∠AQB = × 105° = 52.5° (Angle at the remaining part of the circle is half the
2


angle subtended by the arc at the centre) ½ 

24. We know that, in 60 minutes, the tip of minute hand moves 360°.

360°
In 1 minute, it will move = = 6° ½
60°


∴ From 7 : 05 pm to 7: 40 pm i.e. 35 min, it will move through = 35 × 6° = 210° ½


∴ Area swept by the minute hand in 35 min = Area of sector with sectorial angle θ of


210° and radius of 6 cm


rr 2 i 210
= = × r × 62 ½
360° 360


7 22
= ×
12 7
× 6 × 6 = 66 cm2 

½
OR
Let the measure of ∠A, ∠B, ∠C and ∠D be θ1, θ2, θ3 and θ4 respectively.  ½
Required area = Area of sector with centre A + Area of sector with centre B + Area of


sector with centre C + Area of sector with centre D


i1 i2 i3 i4
= × r × 72 + × r × 72 + × r × 72 + × r × 72 ½
360 360 360 360


_ 1
i + i 2
+ i 3
+ i 4i
= × r × 72 ½
360


]360g 22
= × ×7×7 (By angle sum property of a triangle)
360 7

= 154 cm2 

½
25. The given equations are

sin (A + B) =1 = sin 90° so A + B = 90° ...(i)  ½

Sample Papers 35
3
and cos (A – B)= = cos 30°, so A – B= 30° …(ii) ½
2


From (i) and (ii) ∠A = 60°  1


And ∠B = 30°
OR

cos i – sin i 1– 3
Given identity is = ½
cos i + sin i 1+ 3


Dividing the numerator and denominator of LHS by cos θ, we get


1 – tan i 1 – 3
= = ½
1 + tan i 1 + 3


By comparing, we get tan i = 3  ½


Hence q = 60°.
 ½

SECTION-C
p
26. Let us assume 5 + 2 3 is rational, then it must be in the form of where p and q are
q

co-prime integers and q ≠ 0. 1


p
i.e 5 + 2 3 =
q
 ½
p – 5q
So 3 =
2q
…(i)  ½
Since p, q, 5 and 2 are integers and q ≠ 0, hence RHS of equation (i) is rational. But
LHS of (i) is 3 which is irrational. This is not possible.  ½
This contradiction has arisen due to our wrong assumption that 5 + 2√3 is rational.
So, 5 + 2 3 is irrational.  ½
27. Let α and β be the zeros of the polynomial 2x – 5x – 3.

2
 ½
5
Then α + β = ½
2


–3
And αβ = ½
2


Let 2α and 2β be the zeros x2 + px +q.


Then 2α + 2β = – p ⇒ 2(α + β) = – p  ½
5
⇒ 2× =–p ⇒ p=–5 ½
2

And 2α × 2β = q
3

⇒ 4 αβ = q ⇒ q=4×–
2
=–6  ½
28. Let the actual speed of the train be x km/h and let the actual time taken be y hours.

∴ Distance covered is xy km.  ½


If the speed is increased by 6 km/h, then time of journey is reduced by 4 hours i.e.,
when speed is (x + 6) km/h, time of journey is (y − 4) hours.

36 Mathematics–X
∴ Distance covered =(x + 6)(y − 4)
⇒ xy = (x + 6)(y − 4) ⇒ − 4x + 6y − 24=0  ½
⇒ −2x + 3y − 12 = 0 ...(i) ½
Similarly xy = (x −6)(y + 6)  ½
⇒ 6x − 6y − 36 = 0  ½
⇒ x−y−6=0 ...(ii)
Solving (i) and (ii) we get x = 30 and y = 24
Putting the values of x and y in equation (i), we obtain
Distance = (30 × 24) km = 720 km.
Hence, the length of the journey is 720 km.  ½
OR
Let the number of chocolates in lot A be x and let the number of chocolates in lot B be y.
∴ Total number of chocolates = x + y  ½
2 2
Price of 1 chocolate = ` , so for x chocolates = x.
3 3

and price of y chocolates at the rate of ` 1 per chocolate = y.


2
∴ By the given condition x + y = 400 ½
3


⇒ 2x + 3y = 1200 ...(i)
4
Similarly x + y = 460 ½
5


⇒ 5x + 4y = 2300 ... (ii)


Solving (i) and (ii), we get, x = 300 and y = 200.  ½
∴ x + y = 300 + 200 = 500  ½
So, Anuj had 500 chocolates.  ½
3 3
tan i cot i
29. LHS = + ½
1 + tan2 i 1 + cot2 i


sin3 i cos3 i
cos3 i sin3 i
= + ½
sin2 i cos2 i


1+ 2
1+
cos i sin2 i
sin3 i cos3 i
cos3 i sin3 i
= + ½
^cos i + sin ih
2 2
^sin i + cos2 ih
2


cos2 i sin2 i
sin3 i cos3 i
= + [ sin2 + cos2 = 1]
cos i sin i

q q

sin 4 i + cos 4 i
= ½
cos i sin i


Sample Papers 37
^sin2 i + cos2 ih – 2 sin2 i cos2 i
2
= [ a4 + b4 = (a2 + b2) – 2a2b2]
cos i sin i

1 – 2 sin2 cos2 i 1 2 sin2 i cos2 i


= = – ½
cos i sin i cos i sin i cos i sin i


sec cosec – 2 sin cos = RHS


q q q q  ½
30. Let ABCD be a parallelogram such that its sides touch a circle with centre O.

We know that the tangents to a circle from an exterior point are equal in length.
Therefore, we have  1
AP = AS (Tangents from A) ... (i)
BP = BQ (Tangents from B) ... (ii)
CR = CQ (Tangents from C) ... (iii)
And DR = DS
(Tangents from D) ... (iv)  1
Adding (i), (ii), (iii) and (iv), we have
(AP + BP) + (CR + DR) = (AS + DS) + (BQ + CQ)
⇒ AB + CD = AD + BC

⇒ AB + AB = BC + BC
( ABCD is a parallelogram
\ AB = CD, BC = DA)
⇒ 2AB = 2BC
⇒ AB = BC  ½
Thus, AB = BC = CD = AD  ½
Hence, ABCD is a rhombus.
OR
Join OC.
In APO and ACO, we have
D D

AP = AC
(Tangents drawn from external point A)  1
AO = OA
(Common)
PO = OC
(Radii of the same circle)

\ D APO  ACO
D (By SSS criterion of congruence)


\ ∠PAO = ∠CAO (CPCT)  ½
⇒ ∠PAC = 2∠CAO ... (i)
Similarly, we can prove that OQB  OCB. D D  ½

\ ∠QBO = ∠CBO ⇒ ∠CBQ = 2∠CBO
... (ii)
Now, ∠PAC + ∠CBQ = 180° (Sum of interior angles on the same side of


transversal is 180°)
⇒ 2∠CAO + 2∠CBO = 180° [From equation (i) and (ii)]
⇒ ∠CAO + ∠CBO = 90°  ½
⇒ 180° – ∠AOB = 90° ( ∠CAO + ∠CBO + ∠AOB = 180°)
⇒ 180° – 90° = ∠AOB ⇒ ∠AOB = 90°  ½

38 Mathematics–X
3
31. (i) P(At least one head) = 1
4


3
(ii) P(At most one tail) = 1
4


2 1
(iii) P(A head and a tail) = = 1
4 2


SECTION-D
32. Let the time taken by larger pipe alone to fill the tank= x hours.
½
Therefore, the time taken by the smaller pipe = x + 10 hours.

4
Water filled by larger pipe running for 4 hours = litres.
x

9
Water filled by smaller pipe running for 9 hours = litres. 1
x + 10

According to question,
4 9 1
+ =
x x + 10 2

4 ] x + 10g + 9x 1
⇒ = 1
+
x (x 10) 2


⇒ 2(13x + 40) = x2 + 10x


⇒ 26x + 80 = x2 + 10x ⇒ x2 −16x − 80 = 0
⇒ x2 −20x + 4x −80 = 0 ⇒ x(x – 20) + 4(x – 20)= 0
1
⇒ (x + 4)(x – 20)= 0 ⇒ x = – 4, 20
 ½
x cannot be negative.  ½
Thus, x = 20 and x + 10 = 30 

Hence larger pipe would alone fill the tank in 20 hours and smaller pipe would fill the
tank alone in 30 hours.  ½
OR
Let the usual speed of plane be x km/h  ½
and the reduced speed of the plane be (x – 200) km/h.
Distance = 600 km [Given]
According to the question,
(time taken at reduced speed) – (Schedule time) = 30 minutes = 0.5 hours. 1
600 600 1
– x = 1
x – 200 2


x – (x – 200)
600 e o=
1
⇒ 1
x (x – 200) 2


⇒ 1200 × 200 = x2 – 200x


⇒ x2– 200x − 240000 = 0
⇒ x2 – 600x + 400x − 240000 = 0

Sample Papers 39
⇒ x(x – 600) + 400 (x – 600) = 0  ½
⇒ (x – 600)(x + 400) =0
⇒ x = 600 or x = – 400
But speed cannot be negative.  ½
∴ The usual speed is 600 km/h and
600

the scheduled duration of the flight is


600
= 1 hour.  ½
33. Given: A triangle ABC in which a line parallel to side BC intersects other two sides AB

and AC at D and E respectively.


AD AE
To Prove: =
DB EC

Construction: Join BE and CD and then draw DM ⊥ AC and EN ⊥ AB.  1


1 
Proof: Area of ∆ADE =  base × height 
2 

1
So, ar(∆ADE) = (AD × EN)
2

1
and ar(∆BDE) = (DB × EN)
2

1
Similarly, ar(∆ADE) = (AE × DM) 1
2


1
and ar(∆DEC) = (EC × DM)
2

1
ar ^DADEh 2
AD × EN
AD
Therefore, = = …(i)
ar ^DBDEh 1 DB

DB × EN
2
1
ar ^DADEh 2
AE × DM
AE
and = = …(ii)
ar ^DDECh 1 EC

EC × DM
2
Now, ∆BDE and ∆DEC are on the same base DE and between the same parallel lines BC
and DE.
So, ar(∆BDE) = ar(∆DEC)
...(iii)
AD AE
Therefore, from (i), (ii) and (iii) we have, =  1
DB EC
Second part :
Let ABCD be a trapezium DCAB and EF is a line parallel to AB and hence to DC.
DE CF
To prove : =
EA FB

Construction : Join AC, meeting EF in G.  1


Proof : In ∆ABC, we have GFAB

40 Mathematics–X
CG CF D C
⇒ = [By BPT] ...(i)
GA FB

In ∆ADC, we have EGDC ( EF  AB and AB DC) E F


G

DE CG
⇒ = [By BPT] ...(ii)
EA GA

From (i) and (ii), we get, 

A B 1
DE CF
=
EA FB

34. From the given question,


Radius of the base of cylinder (r) = 2.8 m = Radius of the base of the cone (r)
Height of the cylinder (h) = 3.5 m  1
Height of the cone (H) = 2.1 m.
Slant height of conical part (l) = r2 + H2
= ]2.8g2 + ]2.1g2  1
= 7.84 + 4.41
= 12.25 = 3.5 m  1
Area of canvas used to make tent = CSA of cylinder + CSA of cone
= 2 × π × 2.8 × 3.5 + π × 2.8 × 3.5  1
= 61.6 + 30.8
= 92.4 m2
Cost of 1500 tents at `120 per sq.m
= 1500 × 120 × 92.4
= `16,632,000
16632000
Share of each school to set up the tents = = `332,640 1
50


OR
First Solid Second Solid

(i) SA for first new solid (S1) = TSA of cube + CSA of hemisphere – base area of


hemisphere  1
= 6a + 2 r – r2
2
p
2
p

= 6 × 7 × 7 + 2 π ×3.52 – π × 3.52  1
2
= 294 + 77 – 38.5 = 332.5 cm

Sample Papers 41
Similarly SA for second new solid (S2) is
= 6 × 7 × 7 + 2 π × 3.52 – π × 3.52
= 294 + 77 – 38.5  1
2
= 332.5 cm
Hence, S1: S2 = 1:1
(ii) Volume for first new solid (V1) = Volume of cube – volume of hemisphere

2
=7×7×7 – r × 3.5 3
3

539 1519
= 343 – = cm3 1
6 6


Similarly volume for second new solid


= _V2 i = 7 × 7 × 7 + r × 3.53
2
3

539 2597
= 343 + = cm3 1
6 6


35. Median = 525, so median class = 500 – 600


 ½
Class interval frequency Cumulative frequency
0 – 100 2 2
100 – 200 5 7
200 – 300 x 7+x
300 – 400 12 19 + x
400 – 500 17 36 + x 1½
500 – 600 20 56 + x
600 – 700 y 56 + x + y
700 – 800 9 65 + x + y
800 – 900 7 72 + x + y
900 – 1000 4 76 + x + y

76 + x + y = 100 ⇒ x + y = 24
...(i)

n
– cf
2
Median = l + ×h 1
f


Since, l = 500, h = 100, f = 20, cf = 36 + x and n = 100 ½


Therefore, putting the value in the median formula, we get
50 – ]36 + xg

525 = 500 + ×100 ½


20


⇒ 500 = (14 – x) × 100 ⇒ x=9  ½


⇒ y = 24 – x [From (i)]  ½
⇒ y = 24 – 9 = 15
Therefore, the value of x = 9 and y = 15.

42 Mathematics–X
SECTION-E
36.
(i) B(1, 2), F(– 2,9)

BF2 = (– 2 –1)2 + (9 – 2)²


= (– 3)² + (7)² = 9 + 49 = 58
So, BF = 58 units.  1
(ii) W(–6, 2), X(–4, 0), O(5, 9), P(3, 11)

R 13 Q
S 12 P
11
10
T O
9
F
8 E
U 7 N
G D
6
5
V H 4 C M
3
2 B
W A L
1 K
0
−8 −7 −6 −5 −4 −3 −2 −1 1 2 3 4 5 6 7 8 9
X
I J

Clearly WXOP is a rectangle.


Point of intersection of diagonals of a rectangle is the mid point of the diagonals. So
the required point is mid point of WO or XP  ½
– 6+5 2+9
=d , n ½
2 2


=c m
– 1 11
,
2 2

(iii) A(– 2, 2), G(– 4, 7)


Let the point on y-axis be Z(0, y).  ½


AZ2 = GZ²  ½
⇒ (0 + 2)² + (y – 2)² = (0 + 4)² + (y – 7)²
⇒ (2)² + y² + 4 – 4y = (4)² + y² + 49 – 14y
⇒ 8 – 4y = 65 – 14y
⇒ 10y = 57  ½
So, y = 5.7  ½
i.e., the required point is (0, 5.7).
OR
A(–2, 2), F(–2, 9), G(– 4, 7), H(– 4, 4)  ½
Clearly GH = 7 – 4 = 3 units
AF = 9 – 2 = 7 units
So, height of the trapezium AFGH
= 2 units ½

Sample Papers 43
R 13 Q
S 12 P
11
10
T 9 O
F
8 E
U 7 D N
G
6
5
V H 4 C M
3
A
2 B
W L
X −3 1 K
0
−8 −7 −6 −5 −4 −2 −1 1 2 3 4 5 6 7 8 9
I J

1
So, area of AFGH = (AF + GH) × height
2


1
  = (7 + 3) × 2
2
 ½
  = 10 sq. units  ½
37.
(i) Since each row is increasing by 10 seats, so it is an AP with first term a = 30, and

common difference d=10.  ½


th
So number of seats in 10 row = a10 = a + 9d
⇒ = 30 + 9 × 10 = 120  ½
n
(ii) Sn = (2a + (n –1)d)
2

n
⇒ 1500 =
2
(2 × 30 + (n –1)10)  ½
⇒ 3000 = 50n + 10n2
⇒ n2 + 5n – 300 = 0  ½
2
⇒ n + 20n –15n – 300 = 0
⇒ (n + 20) (n –15) = 0  ½
Rejecting the negative value, n = 15.  ½
OR
No. of seats already put up to the 10 row = S10 th
½
10


S10 =
2
{2 × 30 + (10 –1)10)}  ½
  = 5(60 + 90) = 750  ½
So, the number of seats still required to be put are 1500 – 750 = 750  ½
(iii) If number of rows =17 then the middle row is the 9 row.

th
 ½
   a8 = a + 8d
= 30 + 80  ½
= 110 seats

44 Mathematics–X
38.
(i)

P Q

3600
 1
C X

P and Q are the two positions of the plane flying at a height of 3000 3 m.
A is the point of observation.
PB
(ii) In ∆ PAB, tan 60° = 1
AB


3000 3
Or 3 =
AB
 ½
So AB = 3000 m
QC
tan 30°=
AC

1 3000 3
= ½
3 AC
  

  AC = 9000 m
Distance covered = 9000 – 3000 = 6000 m.
OR
PB P R
In PAB, tan 60° =
AB
D

3000 3
or 3=
AB

⇒ AB = 3000 m 3000

RD
and tan 45° =
AD

3000 3 45°
⇒ 1= D X 1
AD


⇒ AD = 3000 3 m
Distance covered = 3000 3 – 3000 = 3000 ^ 3 – 1 h m  1

distance 6000
(iii) Speed = = = 200 m/s ½
time 30


3600
= 200 × = 720 km/h ½
1000


distance 3000 ^ 3 – 1 h
Alternatively Speed = = = 200 m/s ½
time 15 ^ 3 – 1 h


3600
= 200 × = 720 km/h ½
1000

zzz

Sample Papers 45
Blue-Print
Class-X : Mathematics (Standard)
Based on CBSE Sample Question Paper–2023
Objective Type Questions Case
Form of Questions→ (1 mark)
SAQ–I SAQ–II LAQ based
Units Total
(2 marks) (3 marks) (5 marks) Questions
Chapters ↓ Assertion Reason
MCQ (4 marks)
Questions

I. Number
1. Real Numbers 1(1) 1(1) — 3(1) — — 5(3)
system

2. Polynomials 1(1) — — 3(1) — —

3. Pair of Linear
1(1) — 2(1) 3(1) — —

Equations
II. Algebra 20(8)
4. Quadratic
1(1) — — — 5(1) —

Equations

5. Arithmetic
— — — — — 4(1)

Progressions

III. Coordinate 6. Coordinate


1(1) 1(1) — — — 4(1) 6(3)


Geometry Geometry

7. Triangles 3(3) — 2(1) — 5(1) —


IV. Geometry 16(8)

8. Circles 1(1) — 2(1) 3(1) — —

9. Introduction to
3(3) — 2(1) 3(1) — —

Trigonometry
V. Trigonometry
12(6)
10. Some Applications
— — — — — 4(1)

of Trigonometry

11. Areas Related to


2(2) — 2(1) — — —

Circles
VI. Mensuration 10(5)
12. Surface Areas and
1(1) — — — 5(1) —

Volumes

13. Statistics 2(2) — — — 5(1) —


VII. Statistics

and 11(5)
Probability
14. Probability 1(1) — — 3(1) — —

Total 20(20) 10(5) 18(6) 20(4) 12(3) 80(38)

Note: 1. Number of question(s) is/are given in the brackets.




2. The above Blue Print is only a sample. Suitable internal variations may be made for


generating similar Blue Prints keeping the overall weightage to different form of
questions and typology of questions same.

SAMPLE PAPER
1 [SOLVED]

Time allowed: 3 hours Maximum marks: 80


General Instructions: Same as CBSE Sample Question Paper–2023 (Solved).

SECTION-A
Section A consists of 20 questions of 1 mark each.

1. If HCF (26, 169) = 13 then LCM (26, 169) is


(a) 26
(b) 52
(c) 338
(d) 13

2. The zeroes of the polynomial x2 – 3x – m(m + 3) are


(a) m, m + 3
(b) –m, m + 3
(c) m, – (m + 3)
(d) –m, – (m + 3)

2
3. The value(s) of k for which the quadratic equation 2x + kx + 2 = 0 has equal roots, is

(a) 4
(b) ± 4
(c) – 4
(d) 0

4. The value of c for which the pair of equation cx – y = 2 and 6x – 2y = 3 will have

infinitely many solutions is


(a) 3
(b) –3 (c) –12 (d) no value

5. If A c , 5 m is the mid-point of the line segment joining the points Q(– 6, 7) and
m
3

R(–2, 3), then the value of m is


(a) –12
(b) – 4
(c) 12
(d) – 6

6. QA and PB are perpendicular on AB, if AO = 10 cm, BO = 6 cm



P

and PB = 9 cm, then measure of AQ (see figure) is


(a) 15 cm
(b) 25 cm

A O
B
(c) 10 cm
(d) none of these

4 – sin2 45°
7. The value of is 3.5. What is the value of k? Q
cot k tan 60°

(a) 30°
(b) 45°
(c) 60°
(d) 90°

8. If a cos θ + b sin θ = m and a sin θ – b cos θ = n, then a2 + b2 is equal to


(a) m2 – n2
(b) m2n2
(c) n2 – m2
(d) m2 + n2

Sample Papers 47
AD 3 A
9. In figure, DE  BC. If = and AE = 2.7 cm, then EC is equal to
DB 2

2.7 cm

D E
(a) 2.0 cm
(b) 1.8 cm

(c) 4.0 cm
(d) 2.7 cm

B C
10. Consider the triangles below.

40°

10 cm
A

5c
m
5c

m
B C Q R
10 cm
Which statement is correct?
(a) For triangles to be similar, the measure of ∠A = 40°.

(b) For triangles to be similar, the measure of ∠A = 100°.


(c) Triangles are similar as all isosceles triangles are similar.


(d) Triangles are similar as corresponding sides of the triangles are in the ratio 1 : 2.

11. The value of (1 + cot


q – cosec )(1 + tan
q q + sec ) is q

(a) 1 (b) 2 (c) –1


(d) None of these

12. In the given figure, if PA and PB are tangents to the circle with

centre O such that ∠APB = 50°, then ∠OAB is equal to


(a) 25°
(b) 30°

(c) 40°
(d) 50°

13. The area of the shaded region in the given figure is (Take r = 3.14).

D C
12 cm
(a) 75 cm2 (b) 73 cm2
5 cm

(c) 70 cm2
(d) none of these

A B

14. A wheel has diameter 84 cm. The number of complete revolution it takes to cover

792 m is
(a) 330
(b) 400
(c) 360
(d) 300

15. Consider the following frequency distribution of the height of 60 students of a class:

Height (in cm) 150–155 155–160 160–165 165–170 170–175 175–180

No. of students 15 13 10 8 9 5

The sum of the lower limit of the modal class and upper limit of the median class is
(a) 310
(b) 315
(c) 320
(d) 330

48 Mathematics–X
16. Two identical solid cubes of side k units are joined end to end. What is the volume, in

cubic units, of the resulting cuboid?


(a) 2k3
(b) 3k3
(c) 4k3
(d) 6k3

17. A girl calculates that the probability of her winning the first prize in a lottery is 0.08.

If 6,000 tickets are sold, how many tickets has she bought?
(a) 40
(b) 240
(c) 480
(d) 750

18. The table below shows the time taken by a group of students to complete 100 m race.

Time taken (in sec) 18–20 20–22 22–24 24–26 26–28 28–30
No. of students 3 18 26 19 9 5

Which of these is the mean time taken, in sec, by the group of students to complete the
100 m race when calculated using direct method?
(a) 18.16
(b) 18.96
(c) 23.7
(d) 33.7

DIRECTION: In the question numbers 19 and 20, a statement of assertion (A) is followed by a
statement of Reason (R). Choose the correct option.
(a) Both A and R are true and R is the correct explanation for A.

(b) Both A and R are true and R is not the correct explanation for A.

(c) A is true but R is false.


(d) A is false but R is true.


19. Assertion (A) : For any two positive integers a and b, HCF (a, b) × LCM (a, b) = a × b.

Reason (R) : The HCF of two numbers is 5 and their product is 150. Then their LCM

is 40.
20. Assertion (A) : The value of y is 6, for which the distance between the points P(2, –3) and

Q (10, y) is 10.
Reason (R) : Distance between two given points A (x1, y1) and B (x2, y2) is given by

_ x2 – x1 i + (y2 – y1) 2 .
2
AB =

SECTION-B
Section B consists of 5 questions of 2 marks each.
21. In the given figure, ABCD is a rectangle. Find the values of x and y.

D x+y C

14 cm x–y

A 30 cm B

Sample Papers 49
BE BC
22. In the given figure, DE  AC and DC  AP, Prove that = .
EC CP

B E C P

23. If from an external point P of a circle with centre O, two tangents


PQ and PR are drawn such that ∠QPR = 120°, prove that


2 PQ = PO.
24. A race track is in the form of a ring whose inner circumference is

352 m, and the outer circumference is 396 m. Find the width of the
track.
OR
A piece of wire 22 cm long is bent into the form of an arc of a circle subtending an angle
22
of 60° at its centre. Find the radius of the circle. d Use r = n
7
25. If sin
q = cos , then find the value of 2 tan
q q + cos2 . q

OR
2
If sin A = 2 sin A then find the value of A.

SECTION-C
Section C consists of 6 questions of 3 marks each.

26. Given that


2 is irrational, prove that (5 + 3 2 ) is an irrational number.
27. Quadratic polynomial 2x2 – 3x + 1 has zeros as α and β. Now form a quadratic polynomial

whose zeros are 3α and 3β.


28. A boat goes 30 km upstream and 44 km downstream in 10 hours. In 13 hours, it can go

40 km upstream and 55 km downstream. Determine the speed of the stream and that
of the boat in still water.
OR
The present age of a father is three years more than three times the age of his son.
Three years hence the father’s age will be 10 years more than twice the age of the son.
Determine their present ages.
29. If sin
q + cos q = 3 , then prove that tan q + cot q = 1.
30. A quadrilateral ABCD is drawn to circumscribe a circle (see Fig.).

Prove that AB + CD = AD + BC.

50 Mathematics–X
OR
Prove that the lengths of two tangents drawn from an external point to a circle are
equal.
31. Two different dice are thrown together. Find the probability that the numbers obtained

(i) have a sum less than 7


(ii) have a product less than 16
(iii) is a doublet of odd numbers

SECTION-D
Section D consists of 4 questions of 5 marks each.
1
32. Two taps running together can fill a tank in 3 hours. If one tap takes 3 hours more
13

than the other to fill the tank, then how much time will each tap take to fill the tank?
OR
A motor boat whose speed is 18 km/h in still water takes 1 hour more to go 24 km
upstream than to return downstream to the same spot. Find the speed of the stream.
33. Through the mid-point M of the side CD of a parallelogram ABCD, the line BM is drawn

intersecting AC at L and AD produced to E. Prove that EL = 2BL.


34. A right triangle with sides 3 cm and 4 cm is revolved around its hypotenuse. Find the

volume of double cone thus generated. (Use p = 3.14).


OR
In the given figure a decorative block is shown which is 4.2 cm
made of two solids, a cube and a hemisphere. The base of
the block is a cube with edge 6 cm, and the hemisphere
fixed on the top has a diameter of 4.2 cm. Find.
(i) the total surface area of the block.

22
(ii) the volume of the block formed. cUse r = m 6 cm
7

35. If the median of the following frequency distribution is m


6c

32.5. Find the values of f1 and f2. 6 cm

Class 0–10 10–20 20–30 30–40 40–50 50–60 60–70 Total


Frequency f1 5 9 12 f2 3 2 40

SECTION-E
Case study based questions are compulsory.
36. Your friend Veer wants to participate in a 200 m race. He

can currently run that distance in 51 seconds and with


each day of practice it takes him 2 seconds less. He wants to
complete it in 31 seconds.

Sample Papers 51
Based on the above information answer the following questions.
(i) What is the minimum number of days he needs to practice till his goal is achieved?

(ii) What is the AP formed in this situation?


(iii) If nth term of an AP is given by an = 2n + 3, then write its common difference.


OR
Write the value of x, for which 2x, x + 10, 3x + 2 are three consecutive terms of the AP.
37. Shown below is a town plan on a coordinate grid, where 1 unit = 1 km. Consider the

co-ordinates of each building to be the point of intersection of the respective grid lines.

7
W
S S - School
W - Water Tank
6 P - Pond
P Fire Police
Fire - Fire Station
5 Police - Police Station
7 W
S S - School
6 W - Water Tank
P - Pond
PH1 FirePolice Fire - Fire Station
5 Police - Police Station
H - Hospital
4 H1 - House 1
H2 - House 2
H2H H3 - House 3
3 H4 - House 4
H5 - House 5
H3 H4 H5 H6 H6 - House 6
2 H7 - House 7
H8 - House 8
1
H7 H8
0
1 2 3 4 5 6

H - Hospital
H1 - House 1
4 H2 - House 2
H1 H2 H H3 - House 3
H4 - House 4
3
H5 - House 5
H3 H4 H5 H6 - House 6
H6
2 H7 - House 7
H8 - House 8

H7 H8
0
1 2 3 4 5 6

Based on the above information answer the following questions.


(i) What is the distance between the School and House 1 along the path shown?

(ii) What is the ratio in which House 1 divides the path joining house 3 and the police

station?
(iii) What is the shortest distance between the school and House 3?

OR
Write the shortest distance between House 2 and House 5.
38. A group of students of class X visited India Gate on an education trip. The teacher

and students had interest in history as


well. The teacher narrated that India
Gate, official name Delhi Memorial,
originally called All-India War Memorial,
monumental sandstone arch in New
Delhi, dedicated to the troops of British
India who died in wars fought between
1914 and 1919.The teacher also said
that India Gate, which is located at the

52 Mathematics–X
eastern end of the Rajpath (formerly called the Kingsway), is about 138 feet (42 metres)
in height.
Based on the above information answer the following questions.
(i) What is the angle of elevation if they are standing at a distance of 42 m away from

the monument?
(ii) They want to see the tower at an angle of 60°. So, where should they stand?

(iii) If the altitude of the Sun is at 60° then what is the height of the vertical tower that

will cast a shadow of length 20 m?


OR
If the ratio of the length of a rod and its shadow is 1:1. Then find the angle of
elevation of the Sun.

zzz

SECTION-A
1. (c) HCF (26, 169) = 13

26 × 169
∴ LCM(26, 169) = = 338
13

2
2. (b) Let p(x) = x – 3x – m (m + 3)

⇒ p(x) = x2 – (m + 3) x +mx – m (m + 3)

= x{x – (m + 3)} + m {x – (m + 3)}


For zeros of p(x)
⇒ p(x) = (x + m) {(x – (m + 3)} = 0

⇒ x = – m, m + 3

∴ Its zeros are – m, m + 3.


3. (b) Given quadratic equation, 2x2 + kx + 2 = 0


` Its discriminant, D = (K)2 – 4 × 2 × 2


⇒ D = K2 – 16

For roots to be real and equal


D = 0 ⇒ K2 – 16 = 0 ⇒ K2 = 16

⇒ K =±4

4. (d) Given lines are cx – y = 2 and 6x – 2y = 3.


For infinitely many solutions, we have


a1 b1 c1 c –1 –2 c 1 c 2
= = ⇒ = = ⇒ = and =
a2 b2 c2 6 –2 –3 6 2 6 3

6
⇒ c = and c = 4 ⇒ c = 3 and c = 4
2

Since, c has different values, hence, for no value of c, the pair of equations will have
infinitely many solutions.

Sample Papers 53
5. (a)
We have,
m – 6 + (– 2) –8
= ⇒ m= × 3 ⇒ m = –12
3 2 2

P
6. (a)
In DAOQ and DBOP, we have
+AOQ = +BOP (Vertically opposite angle) 9 cm
10 cm O
+OAQ = +OBP (Both of 90°) A
6 cm B
` DAOQ ~ DBOP (By AA similarity)
AO AQ 10 AQ 10×9
So
BO
=
PB
& 6 = 9 & AQ = 6 = 15 cm Q
2
4 – sin 45°
7. (c) Given = 3.5
cot k tan 60°

2
1
4–e o 4–
1
2 2 = 3.5
& = 3.5 &
cot k 3 3 cot k

7
& = cot k & 7 = cot k
2 3 × 3.5

7 3
1
⇒ = cot k & cot 60° = cot k ⇒ k = 60°

3
8. (d) We have,

m 2 + n 2 = (a cos i + b sin i) 2 + (a sin i – b cos i) 2


= a 2 cos 2 i + b 2 sin 2 i + 2ab cos i sin i + a 2 sin 2 i + b 2 cos 2 i – 2ab sin i cos i
= a 2 (cos 2 i + sin 2 i) + b 2 (sin 2 i + cos 2 i)
⇒ m2 + n2 = a2 × 1 + b2 × 1 = a2 + b2
∴ a2 + b2 = m2 + n2
A
9. (d)
We have,
2.7 cm
In D ABC, DE || BC
D E
AD AE 3 2.7
⇒ = ⇒ =
DB EC 2 EC

B C

2.7 × 2
⇒ EC = = 1.8 cm = 1.8 cm
3

10. (b)
In DPQR we have
PR = QR & +Q = +P = 40°
+R = 180° – (+P + +Q) = 180° – (40° + 40°) = 100°

` +R = 100°
AB 5 1 AC 5 1 AB AC 1
We have, = = , =
PR 10 2 QR 10 2
= & = =
PR QR 2
Condition for both be triangles to be similar is
+A = +R = 100°

54 Mathematics–X
11. (b)
We have,
(1 + cot – cosec ) (1 + tan + sec )
q q q q

cos i 1 sin i 1
= d1 + – n d1 + + n
sin i sin i cos i cos i

sin i + cos i – 1 sin i + cos i + 1


= d nd n
sin i cos i

(sin i + cos i) 2 – (1) 2 sin 2 i + cos 2 i + 2 sin cos i – 1


= e o=
sin i cos i sin i cos i

1 + 2 sin i cos i – 1 2 sin i cos i


= = =2
sin i cos i sin i cos i

12. (a)
∠AOB = 180° – 50° = 130°
Let ∠OAB = ∠OBA = x° (Angles opposite to equal sides)
Then x° + x° + 130° = 180°
⇒ 2x° = 50° ⇒
   x° = 25°
⇒ ∠OAB = 25°

13. (b) Area of shaded region = area of circle – area of rectangle


Diagonal of rectangle = 12 2 + 5 2 = 144 + 25 = 13 cm


13
Radius of circle = cm
2

13 2 3.14 169
Area of shaded region = r × d n – 12 × 5 = × – 60
2 100 4

53060
= – 60 = 132.66 – 60.00 = 72.66 cm 2 = 73 cm2 (approx)
400
    

14. (d) First find the circumference of the wheel and then find number of revolution

distance covered
=
circumference of wheel

Circumference of wheel = 2rr


22
= 2× × 42 = 12 × 22
7
     

79200
Number of revolution = = 300
12 × 22

n 60
15. (b) = = 30, median class = 160 – 165
2 2

Modal class = 150 – 155


Required sum = 150 + 165 = 315
16. (a) Joining two identical cubes of each side k units, we get a cuboid with length (l) = 2k,

breadth (b) = k and height (h) = k.


Volume of resulting cuboid = l × b × h
    = 2k × k × k = 2k3

Sample Papers 55
17. (c) Number of required tickets = 6000 × 0.08 = 480

18. (c) We have


Time taken Mid. No. of students fi . xi


(in sec.) value (fi)
18 – 20 19 3 57
20 – 22 21 18 378
22 – 24 23 26 598
24 – 26 25 19 475
26 – 28 27 9 243
28 – 30 29 5 145
Total 80 1896
Sfi = 80 and Sfixi = 1896
Rfi xi 1896
Mean = = = 23.7
Rfi 80

19. (c) We have,


LCM (a, b) × HCF (a, b) = a × b


  LCM × 5 = 150
150
∴ LCM = = 30
5
   

⇒ LCM = 30, i.e., reason is incorrect and assertion is correct.


20. (d)
PQ = 10 ⇒ PQ2 = 100 ⇒ (10 – 2)2 + (y + 3)2 = 100
⇒ (y + 3)2
⇒ 100 – 64 = 36 ⇒ y+3=6
⇒ y=–36
⇒ y = 3, – 9
So, A is incorrect but R is correct.

Section–B
21. Given, rectangle ABCD.

⇒ Opposite sides are equal.


Hence, x + y = 30
...(i)
x – y = 14 ...(ii)
(i) + (ii) 2x = 44 ⇒ x = 22
Substituting in (i), 22 + y = 30 ⇒ y=8
⇒ x = 22, y = 8
22. We have,

In DABC, DE AC
BD BE
∴ = ...(i)
DA EC

Also, in ABP, DC AP

BD BC
∴ = ...(ii)
DA CP

56 Mathematics–X
From (i) and (ii), we have
BE BC
=
EC CP

23. Given, ∠QPR = 120°


 Radius is perpendicular to the tangent at the point of contact.


∴ ∠OQP = 90° and
∠QPO = 60° (Tangents drawn to a circle from an external point are equally inclined to the
segment, joining the centre to that point.)
PQ 1 PQ
In QPO, cos 60° = ⇒ = ⇒ 2PQ = PO
PO 2 PO
∆    

24. Let the outer and inner radii of the ring be R m and r m respectively. Then,

2pR = 396 and 2pr = 352


22 22
⇒ 2# # R = 396 and 2# # r = 352
7 7

7 1 7 1
⇒ R = 396 × # and r = 352 × #
22 2 22 2

⇒ R = 63 m and r = 56 m
Hence, width of the track = (R – r) = (63 – 56) = 7 m
OR
Let r be the radius and q be the angle subtended by the arc at the centre of the circle.
∴ Length of arc = length of piece of wire
i 60°
× 2rr = 22
360

60 rr

360
× 2rr = 22 & 3
= 22
3 × 22 3 × 22
⇒ r= = = 21 cm
r 22
    

7
∴ Radius = 21 cm
25. Sin
q = cos q (Given)
It means value of q = 45°.
2
Now, 2 tan q + cos q = 2 tan 45° + cos2 45°
2
1 5 a tan 45° = 1, cos 45° = 1
= 2 ×1 + e o = 2+ = e o
1 4+1
=
2 2 2 2 2

OR
sin2 A = 2 sin A
⇒ sin2 A – 2 sin A = 0 ⇒  sin A (sin A – 2) = 0
⇒ either sin A = 0 or sin A – 2 = 0.
⇒ A = 0° (sin A = 2, which is not possible)

\ Value of ∠A = 0°

Sample Papers 57
SECTION-C
26. Given
2 is irrational.
To prove: 5 + 3 2 is irrational.
a

Proof: Let us assume 5 + 3 2 is rational. So it can be written in the form where a, b


b

∈ Z, b ≠ 0, HCF (a, b)=1.


a a
5+3 2 = ⇒ 3 2= –5
b b
     

a – 5b a – 5b
3 2= ⇒ 2=
b 3b
     

This shows that 2 is rational (a – 5b and 3b are integers). But we know that 2 is
irrational. This contradicts our assumption that 5 + 3 2 is rational.
⇒ 5 + 3 2 is irrational, hence proved.
27. It is given that α and β are zeros of the polynomial 2x2 – 3x + 1.

– ]–3g 3 1
∴ a+b= = and ab =
2 2 2

Now, new quadratic polynomial whose zeros are 3α and 3β is given by


x2 – (sum of zeros)x + product of zeros
= x2 – (3α + 3β)x + 3α × 3β = x2 – 3(α + β)x + 9αβ
3 1
= x2 – 3 × x + 9 ×
2 2

9 9 1
= x 2 – x + = ^2x 2 – 9x + 9h
2 2 2

28. Let the speed of the boat be x km/h and speed of the stream be y km/h.


\ Speed of boat in upstream = (x – y) km/h
and speed of boat in downstream = (x + y) km/h

\ According to question,
30 44
+ = 10 ...(i)
x– y x+y

40 55
and + = 13 ...(ii)
x– y x+y

(i) × 4 – (ii) × 3 gives


1
(176 – 165) = 1
x+y

⇒ x + y = 11 ...(iii)
Putting the value of x + y = 11 in equation (i), we have
30 44
+ = 10
x – y 11

30 30
⇒ + 4 = 10 ⇒ = 10 – 4 = 6
x–y x–y

58 Mathematics–X
⇒ x–y=5 ...(iv)
By adding equations (iii) and (iv), we have
x + y = 11
x–y=5
2x = 16
⇒ x=8
Putting x = 8 in equation (iii), we have
8 + y = 11

y = 11 – 8 = 3 ⇒ y=3


\ Speed of boat is 8 km/h and speed of stream is 3 km/h.
OR
Let father’s present age be x years and his son’s present age be y years.
According to question,
x = 3y + 3 ⇒ x – 3y = 3 ...(i)

After three years,


Father’s age will be (x + 3) years and his son’s age will be (y + 3) years.
Again, according to question
⇒ x + 3 = 2(y + 3) + 10
⇒ x + 3 = 2y + 6 + 10 ⇒ x – 2y = 13 ...(ii)

Subtracting (ii) from (i), we get


x – 3y = 3

– x  2y = –13
– y = –10
  ⇒ y = 10
Putting the value of y = 10 in equation (i), we get
x – 3 × 10 = 3 ⇒ x = 33

∴ Present age of father is 33 years and his son’s present age is 10 years.
29. sin
q + cos = 3 q

⇒ (sin + cos )2 = 3
q q

⇒ sin2 + cos2 + 2 sin


q q q cos q =3
⇒ 2 sin cos = 2
q q ( sin2 q + cos2 q = 1)
⇒ sin . cos = 1 = sin2
q q q + cos2 q

sin 2 θ + cos2 θ sin2 i cos2 i


⇒ 1= ⇒ 1= +
sin θ cos θ sin i cos i sin i cos i

⇒ 1 = tan + cot q q

Therefore tan + cot = 1. q q

30. Given: A quadrilateral circumscribing a circle, with centre O, such that it touches side


AB, BC, CD, AD at P, Q, R and S. D R C


To prove: AB + CD = BC + DA
S

Q
Proof:  Length of tangent drawn from external point are equal.
AP = AS — (at A) ...(i)
A P B

Sample Papers 59
BP = BQ — (at B) ...(ii)
DR = DS — (at C) ...(iii)
CR = CQ — (at D) ...(iv)
Adding equation (i), (ii), (iii), (iv)
AP + BP + DR + CR = AS + DS + BQ + CQ
& AB + CD = AD + BC
Hence Proved.
OR
Given: Circle (O, r) AP and PB are tangents drawn to the circle.
To prove : PA = PB. A
Construction: Join OA, OB and OP.
Proof : OA = OB [Radii].
+OAP = +OBP = 90° (Right angle)
[ a Radius is perpendicular


B
to tangent at point of
contact]
OP = OP (Hypotenuse)
So in DOAP and DOBP
& DOAP , DOBP [By RHS congruency]
& AP = BP [By CPCT]
Hence Proved.
31. Total number of outcomes = 36

(i) Favourable outcomes are (1, 1,) (1, 2) (1, 3) (1, 4) (1, 5) (2, 1) (2, 2) (2, 3) (2, 4) (3, 1)

(3, 2) (3, 3) (4, 1) (4, 2) (5, 1) i.e., 15.


15 5
∴ P (sum less than 7) = or
36 12

(ii) Favourable outcomes are (1, 1) (1, 2) (1, 3) (1, 4) (1, 5) (1, 6) (2, 1) (2, 2) (2, 3) (2, 4)

(2, 5) (2, 6) (3, 1) (3, 2) (3, 3) (3, 4) (3, 5) (4, 1) (4, 2) (4, 3) (5, 1) (5, 2) (5, 3) (6, 1) (6,
2) i.e., 25.
25
∴ P (product less than 16) =
36

(iii) Favourable outcomes are (1, 1) (3, 3) (5, 5) i.e., 3.


3 1
∴ P (doublet of odd number) = or
36 12

SECTION-D
32. Let, time taken by faster tap to fill the tank be x hours.

Therefore, time taken by slower tap to fill the tank = (x + 3) hours


Since the faster tap takes x hours to fill the tank.
1

Portion of the tank filled by the faster tap in one hour =


x

\

1
Portion of the tank filled by the slower tap in one hour =
x+3

60 Mathematics–X
1 13
Portion of the tank filled by the two tap together in one hour = =
40 40

According to question, 13
1 1 13 x+3+x 13
⇒ x + x + 3 = 40 ⇒

x ( x + 3)
=
40
⇒ 40 (2x + 3) = 13x (x + 3) ⇒
80x + 120 = 13x2 + 39x
2
⇒ 13x – 41x – 120 = 0 ⇒
13x2 – 65x + 24x – 120 = 0
⇒ 13x (x – 5) + 24 (x – 5) = 0 ⇒
(x – 5) (13x + 24) = 0
Either x – 5 = 0
or 13x + 24 = 0
– 24
⇒ x=5 or x=
13

⇒ x=5 [ x cannot be negative]


Hence, time taken by faster tap to fill the tank = x = 5 hours and time taken by slower
tap = x + 3 = 5 + 3 = 8 hours.
OR
Let the speed of stream be x km/h.
∴ The speed of the boat upstream = (18 – x) km/h 
and Speed of the boat downstream = (18 + x) km/h 
According to question, 24 − 24 = 1
18 − x 18 + x
⇒ 24 ( 18 + x) – 24 (18– x) = 324 – x2

⇒ 48x = 324 – x2 ⇒ x2 + 48x – 324 = 0


⇒ x2 + 54x – 6x – 324 = 0 ⇒ x (x + 54) – 6 (x + 54)
⇒ (x + 54) (x – 6) = 0
But x ≠ – 54, ∴ x = 6 [Speed can’t be negative]
∴ Speed of the stream = 6 km/h.
33. In BMC and EMD, we have
∆ ∆

MC = MD ( M is the mid-point of CD)


∠CMB = ∠DME (Vertically opposite angles)
and ∠MBC = ∠MED (Alternate angles)
BMC  EMD
∆ ∆ (By AAS criterion of congruence)


⇒ BC = DE (CPCT)
Also, BC = AD ( ABCD is a parallelogram)
Now, in AEL and CBL, we have
∆ ∆

∠ALE = ∠CLB (Vertically opposite angles)


∠EAL = ∠BCL (Alternate angles)
∴ ∆ AEL ~ CBL∆ (By AA similarity)
EL AE EL 2BC
⇒ = ⇒
= ( AE = AD + DE = BC + BC = 2BC)

BL CB BL BC
EL

=2 ⇒ EL = 2BL
BL

Sample Papers 61
34. In the given Fig., DPQR is a right triangle, where PQ = 3 cm, PR = 4 cm and

QR = 5 cm.
Let OQ = x ⇒ OR = 5 – x and OP = y
Now in right angled-triangle POQ, we have
PQ2 = OQ2 + OP2 (By Pythagoras Theorem)
2 2 2
⇒ (3) = x + y ⇒ y2 = 9 – x2 …(i)
Also from right angled triangle POR, we have

2 2 2
OP + OR = PR
⇒ y2 + (5 – x)2 = (4)2

⇒ y2 = 16 – (5 – x)2

…(ii)
From (i) and (ii), we get
9 – x2 = 16 – (5 – x)2
⇒ 9 – x2 = 16 – (25 + x2 – 10x)
9
or 9 – x2 = – 9 – x2 + 10x ⇒ 10x = 18 ⇒ x=
5

9 16
OR = 5 – x = 5 – =
5 5

\

9
Now putting x = in (i), we get
5

9 2 81 12
y2 = 9 – c m = 9 –
144
= ⇒ y=
5 25 25 5

12
OP = y =
5

\

12 9
Now for the cone PQM, radius OP = cm, height OQ = cm
5 5

1 12 2 9 432r
Volume = rr2 h = r c m # =
1
cm3
3 3 5 5 125

\

12 16
Also for the cone PRM, radius OP = cm, height OR = cm
5 5

2
12
Volume = r c m #
1 16 768r
= cm3
3 5 5 125
\  

Hence total volume, i.e., volume of the double cone


432r 768r 1200r
= c + m= = 9.6 × 3.14 cm3 = 30.144 cm3
125 125 125

OR
4.2 cm
d 4.2
Radius of the hemiphere = = cm = 2.1 cm
2 2

Side of cube = 6 cm
(i) Total surface area of block = Total surface area of

6 cm
cube + Cirved surface area of hemisphere – area
enclosed by base of hamisphere
m
6 cm 6c

62 Mathematics–X
4rr 2 2 2
= 6a 2 + – rr 2 = 6a + rr
2

22 22
= 6 × 62 + × (2.1) 2 cm 2 = 216 + × 2.1 × 2.1 cm 2
7 7

= (216 + 13.86) cm2 = 229.86 cm2


(ii) Volume of block formed = Volume of cube + Volume of hemisphere

2 3
= a3 + rr
3

2 22 2 22 21×21×21
= 6 3 + × × 2.1 3 = 216 + × ×
3 7 3 7 1000

= 216 + 19.404 = 235.404 cm3


35.

Class Frequency Cumulative Frequency


0–10 f1 f1
10–20 5 5 + f1
20–30 9 14 + f1
30–40 12 26 + f1
40–50 f2 26 + f1 + f2
50–60 3 29 + f1 + f2
60–70 2 31 + f1 + f2
Sf = 40
Median = 32.5 ⇒ Median class is 30–40.
10
Now 32.5 = 30 + (20 – 14 – f1) ⇒ f1 = 3
12

Also 31 + f1 + f2 = 40 ⇒
f2 = 6

SECTION-E
36. (i) Let n be the minimum number of days for AP. 51, 49, 47 .....

∴ an = 31

⇒ a +(n – 1)d = 31 ⇒ 51 +(n – 1) × (–2) = 31


⇒ (n – 1) × (–2) = –20 ⇒ n – 1 = 10

⇒ n = 11
(ii) Since Veer currently runs the distance in 51 seconds and with each day of practice

it takes him 2 seconds less.


Thus, AP formed is 51, 49, 47, ....
(iii) Given nth term of an AP is

an = 2n + 3
∴ d = an – an – 1 = (2n + 3) – (2(n – 1) + 3)
⇒ d=2
OR
Since, 2x, x + 10, 3x + 2 are three consecutive terms are in AP.
∴ (x + 10) – 2x = (3x + 2) – (x + 10)
⇒ 10 – x = 2x – 8
⇒ 18 = 3x ⇒ x = 6

Sample Papers 63
37. (i) Coordinates of school are (1, 6).

Coordinates of House 1 are (3, 3).


` Distance between school and House 1, is
(3 –1) 2 + (3 – 6) 2 = 4 + 9 = 13 km
(ii) Coordinates of House 1 (H1) are (3, 3).

k 1
Police station
House 3 (H3) coordinates are (2, 2). H3 H1 (5, 5)
Coordinates of police station are (5, 5). (2, 2) (3, 3)

Let H1 divides path joining H3 and police station in the ratio k : 1.


By section formula we have
k ( 5) + 1 ( 2)
3=
k+1

& 3k + 3 = 5k + 2
& 3 – 2 = 5k – 3k
1
& 2k = 1 &    k=
2
1
` k : 1 = :1 = 1 : 2
2

(iii) Shortest distance


= (2 – 1) 2 + (2 – 6) 2 = 1 + 16 = 17 km
OR
Shortest distance = (4 – 4) 2 + (3 – 2) 2 = 0 + 1 = 1 km
38. (i) Let θ be the angle of elevation.

India Gate
42 m
42
∴ tan i = =1
42

⇒ tan θ = 1 = tan 45° ⇒ θ = 45° 42 m

∴ Angle of elevation is 45°.


(ii) Let x m be the required distance.

India Gate 42 m

42
∴ tan 60° = x
42 42
⇒ 3= x ⇒ x=
3 60°
xm
42 3
42 3
⇒ x= × = = 14 3 = 14 × 1.732 = 24.248 m
3

3 3
(iii) Let h m be the height of the vertical tower.

We have,
h
∴ tan 60° =
hm

20

h
⇒ 3= ⇒ h = 20 3 m
20

60°
OR 20 m
Let θ be the angle of elevation.
1
∴ tan i = =1 ⇒ tan θ = 1 = tan 45° ⇒ θ = 45°
1

zzz

64 Mathematics–X
SAMPLE PAPER
2 [SOLVED]

Time allowed: 3 hours Maximum marks: 80


General Instructions: Same as CBSE Sample Question Paper–2023 (Solved).

SECTION-A
Section A consists of 20 questions of 1 mark each.

1. HCF × LCM for the numbers 150 and 10 is


(a) 1500
(b) 150
(c) 10
(d) None of these

2. The roots of the quadratic equation x2 – 0.04 = 0 are


(a) ± 0.2
(b) ± 0.02
(c) 0.4
(d) 2

3. Given that m + 2, where m is a positive integer, is a zero of the polynomial


q(x) = x2 – mx – 6 Which of these is the value of m?


(a) 4
(b) 3
(c) 2
(d) 1

4. If 2x – 3y = 7 and (a + b)x – (a + b – 3)y = 4a + b represent coincident lines, then a


and b satisfy the equation


(a) a + 5b = 0
(b) 5a + b = 0
(c) a – 5b = 0
(d) 5a – b = 0

5. Which of the following are the coordinates of the intersection points of the diagonals

of the rectangle ABCD with vertices A(0, 3), B(3, 0), C(1, –2) and D(–2, 1)?

(a) d , n (b) d – , – n
1 1 1 1
(c) (1.5, 1.5) (d) (2, –1)
2 2 2 2

6. In the given figure, PQ || CB.


C
To the nearest tenth, what is the length of QB?
cm
(a) 1.4 cm

3.3
P
(b) 1.7 cm

cm
(c) 1.8 cm

2.4

(d) 2.2 cm

A
1.3 cm Q
B

7. If a + b = 90° and a = 2b, then cos2a + sin2b is equal to


1
(a) 1 (b) (c) 0 (d) 2
2

Sample Papers 65
8. If sin q + cos q =
2 cos q, (q ≠ 90°) then the value of tan q is
(a) 2 –1 (b) 2 +1 (c) 2 (d) – 2

9. D and E are respectively the points on the sides AB and AC of a triangle ABC such that

AD = 3 cm, BD = 5 cm, BC = 12.8 cm and DE || BC. Then length of DE (in cm) is


(a) 4.8 cm
(b) 7.6 cm
(c) 19.2 cm
(d) 2.5 cm

A D
6 cm
m 30°
10. In the given figure, two line segments AC and BD intersect
5c
P
each other at the point P such that PA = 6 cm, PB = 3 cm, 50°
2.5 c
PC = 2.5 cm, PD = 5 cm, ∠APB = 50° and ∠CDP = 30°. 3c
m m
C
B
Then, ∠PBA is equal to
(a) 50°
(b) 30°
(c) 60°

(d) 100°

11. In the given figure, the pair of tangents AP and AQ drawn from

an external point A to a circle with centre O are perpendicular to


each other and length of each tangent is 5 cm. Then radius of the
circle is
(a) 10 cm (b) 7.5 cm

(c) 5 cm
(d) 2.5 cm

12. A circular garden, of circumference 88 m is surrounded by a pathway of width 3.5 m.


Ajay wants to put fence around the pathway. What is the cost of fencing the pathway at
22
the rate of ` 70 per metre? d Use r= n
7
(a) ` 3,080
(b) ` 3,850
(c) ` 6,160
(d) ` 7,700

13. The volume of the largest right circular cone that can be cut out from a cube of edge

4.2 cm is
(a) 9.7 cm3
(b) 77.6 cm3
(c) 58.2 cm3
(d) 19.4 cm3

14. When calculated using direct method, the mean of the data set shown in the table below

is 31.
Class Interval Frequency
0–10 22
10–20 24
20–30 35
30–40 30
40–50 27
50–60 m
60–70 6
70–80 4

What is the frequency for the class interval 50–60?


(a) 8 (b) 10
(c) 12 (d) 20

66 Mathematics–X
15. An arc of a circle of radius 14 cm, subtends an angle of 45° at the centre as shown:

Which of these options is correct?


(a) The arc shown is a minor arc and its length is 5.5 cm.

(b) The arc shown is a major arc and its length is 77 cm.

O
(c) The arc shown is a major arc and its length is 38.5 cm.

14
cm
45°
(d) The arc shown is a minor arc and its length is 11 cm.

16. For the following distribution :


Marks Number of students


Below 10 3
Below 20 12
Below 30 27
Below 40 57
Below 50 75
Below 60 80

The modal class is


(a) 10–20
(b) 20–30
(c) 30–40
(d) 50–60

17. In a lottery, there are 8 prizes and 16 blanks. The probability of getting a prize is

2 1 1 1
(a) (b) (c) (d)
3 3 2 4

cos x + sin x
18. If 4 tan x = 3, than is equal to
cos x – sin x

1 1
(a) 7 (b) (c) – 7 (d) –
7 7

DIRECTION: In the question numbers 19 and 20, a statement of assertion (A) is followed by a
statement of Reason (R). Choose the correct option.
(a) Both A and R are true and R is the correct explanation for A.

(b) Both A and R are true and R is not the correct explanation for A.

(c) A is true but R is false.


(d) A is false but R is true.


19. Assertion (A) : 6n ends with the digit zero, where n is natural number.

Reason (R) : Any number ends with digit zero, if its prime factor is of the form

2m × 5n, where m, n are natural numbers.


20. Assertion (A) : The point (– 1, 6) divides the line segment joining the points (– 3, 10) and

(6, – 8) in the ratio 2 : 7 internally.


Reason (R) : Mid point of line segment PQ whose co-ordinate are P(x1, y1) and

x1 + x2 y1 + y2
Q(x2, y2) is given by R f , p.
2 2

Sample Papers 67
SECTION-B
Section B consists of 5 questions of 2 marks each.
21. Find the value of c for which the pair of equations cx – y = 2 and 6x – 2y = 3 will have

infinitely many solutions.


AM AN
22. In given figure, if LM || CB and LN || CD, prove that = .
AB AD

23. Prove that the perpendicular at the point of contact to the tangent to a circle passes

through the centre.


24. What is the ratio of areas of two circles whose circumferences are in the ratio 3 : 4?

OR
If a circle is inscribed in a square, what is the ratio of the area of the circle and the
square?
25. In given figure, find tan P – cot R.

OR
1
If tan (A +B) = 3 and tan (A – B) = ; 0° < A + B ≤ 90°; A > B, find A and B.
3

SECTION-C
Section C consists of 6 questions of 3 marks each.

26. Prove that 2 + 5 3 is an irrational number, given that 3 is an irrational number.


27. If one zero of the polynomial 3x2 – 8x + 2k + 1 is seven times the other, find the value

of k.
28. Taxi charges in a city consist of fixed charges and the remainings charges depend upon

the distance travelled. For a journey of 10 km, the charge paid is `75 and for a journey
of 15 km, the charge paid is `110. Find the fixed charge and charges per km. Hence,
find the charge of covering a distance of 35 km.

68 Mathematics–X
OR
Students of a class are made to stand in rows. If one student is extra in each row, there
would be 2 rows less. If one student is less in each row, there would be 3 rows more. Find
the number of students in the class.
sin A – 2 sin 3 A
29. Prove that : = tan A (q is replaced by A)
2 cos 3 A – cos A

30. If a circle touches the side BC of a triangle ABC at P and extended sides AB and AC at Q

1
and R, respectively, prove that AQ = (BC + CA + AB).
2
OR
Prove that the tangents drawn at the end points of a chord of a circle make equal angles
with the chord.

31. From a pack of 52 playing cards, Jacks, Queens and Kings of red colour are removed.

From the remaining, a card is drawn at random. Find the probability that drawn card is:
(i) a black King (ii) a card of red colour (iii) a card of black colour

SECTION-D
Section D consists of 4 questions of 5 marks each.
32. ` 9,000 were divided equally among a certain number of persons. Had there been 20

more persons, each would have got ` 160 less. Find the original number of persons.
OR
A two digit number is such that the product of its digits is 18. When 63 is subtracted
from the number, the digits interchange their places. Find the number.
33. In Fig., P is the mid-point of BC and Q is the mid-point of AP. If BQ

1
when produced meets AC at R, prove that RA = CA.
3
34. Water is flowing through a cylindrical pipe of internal diameter

2 cm, into a cylindrical tank of base radius 40 cm at the rate of


0.7 m/sec. By how much will the water rise in the tank in half an
hour?
OR
Rasheed got a playing top (lattu) as his birthday present, which
surprisingly had no colour on it. He wanted to colour it with his
crayons. The top is shaped like a cone surmounted by a hemisphere
(see figure). The entire top is 5 cm in height and the diameter of the top is
22
3.5 cm. Find the area he has to colour. c Take r = m
7
35. The distribution given below shows the number of wickets taken by

bowlers in one-day cricket matches. Find the mean and the median of the number of
wickets taken.

Sample Papers 69
Number of wickets 20 – 60 60 – 100 100 – 140 140 – 180 180 – 220 220 – 260

Number of bowlers 7 5 16 12 2 3

SECTION-E
Case study based questions are compulsory.

36. India is competitive manufacturing location due to the low cost of manpower and strong

technical and engineering capabilities contributing to higher quality production runs.


The production of TV sets in a factory increases uniformly by a fixed number every year.
It produced 16000 sets in 6th year and 22600 in 9th year.

On the basis of above information, answer the following questions.


(i) What is the production during 8th year?

(ii) Write the total production in (during) first 3 years.


(iii) In which year, the production is `29,200?


OR
Write the difference of the production during 7th year and 4th year.
D C
37. In order to conduct Sports Day activities in your School, lines

have been drawn with chalk powder at a distance of 1 m each,


in a rectangular shaped ground ABCD, 100 flowerpots have G

been placed at a distance of 1 m from each other along AD, as R

shown in given figure below. Niharika runs 1/4 th the distance


AD on the 2nd line and posts a green flag. Preet runs 1/5th
distance AD on the eighth line and posts a red flag.
Based on the above information, answer the following 2
1
questions. A B
1 2 3 4 5 6 7 8 9 10
(i) Write the position of red flag.

(ii) What is the distance between both the flags?


70 Mathematics–X
(iii) If Rashmi has to post a blue flag exactly halfway between the line segment joining

the two flags, where should she post her flag?


OR
If Joy has to post a flag at one-fourth distance from green flag in the line segment
joining the green and red flags, then where should he post his flag?
38. A guard, stationed at the top of a 300 m tower,

observed an unidentified boat coming towards


it. A clinometer or inclinometer is an instrument
used for measuring angles or slopes(tilt). The
guard used the clinometer to measure the angle
of depression of the boat coming towards the
lighthouse and found it to be 30°.
Based on the above information, answer the
following questions.
(i) Make a labelled figure on the basis of the

given information and calculate the distance of the boat from the foot of the
observation tower.
(ii) After 10 minutes, the guard observed that the boat was approaching the tower

and its distance from tower is reduced by 300 ( 3 – 1) m. He immediately raised


the alarm. What was the new angle of depression of the boat from the top of the
observation tower?
(iii) When angle of depression of boat coming towards the light house is found to be

60°, then find the distance of boat from the tower.


OR
Write the difference of the distance when angle of depression changes from 45° to
60°. (use 3 = 1.73 )

zzz

Sample Papers 71

SECTION-A
1. (a) LCM × HCF = Product of numbers

∴ For 150 and 10


LCM × HCF = 150 × 10 = 1500
2. (a)
We have,
x2 – 0.04 = 0
⇒ (x2) – (0.2)2 = 0 ⇒ (x – 0.2) (x + 0.2) = 0
⇒ x – 0.2 = 0 and x + 0.2 = 0
⇒ x = 0.2 and x = – 0.2
⇒ x = 0.2, – 0.2
⇒ Roots are ± 0.2.
3. (d) We have polynomial, q(x) = x2 – mx – 6.

Since (m + 2) is a zero of q(x).


\ (m + 2)2 – m (m + 2) – 6 = 0

2 2
⇒m + 4m + 4 – m – 2m – 6 = 0

⇒ 2m – 2 = 0

⇒ 2m = 2
⇒ m=1
4. (c) Given lines,

2x – 3y – 7 = 0 …(i)
and, (a + b)x – (a + b – 3)y – (4a + b) = 0 …(ii)
For both lines to be coincident
2 –3 –7
=– =
( a + b) (a + b–3) – (4a + b)

2 3
⇒ =
a+b a+ b – 3

⇒ 2a + 2b – 6 = 3a + 3b
⇒ 3a + 3b – 2a – 2b + 6 = 0
⇒ a + b + 6 = 0 ⇒ a +b = – 6

2 7
Again, =
a+ b 4a + b

⇒ 8a + 2b = 7a + 7b
⇒ 8a – 7a = 7b – 2b ⇒ a = 5b
⇒ a – 5b = 0

72 Mathematics–X
5. (a)
Since the diagonals of a rectangle bisect each other. D (–2, 1) C (1, –2)

Let intersection point be O.


` Co-ordinates of point O be mid point of AC and BD.
O
0 + 1 –2 + 3
d n=d , n
1 1
,
2 2 2 2

A (0, 3) B (3, 0)
6. (c) We have

In DABC PQ ; ; BC
AQ AP 1.3 2.4 24
`
QB
=
PC
&

QB
=
3.3
=
33
1.3 8 1.3 ×11
& QB =
11
& QB = 8
= 1.78 = 1.8 cm

7. (b)
Given a + b = 90° and a = 2b
& 2b + b = 90° & b = 30°
So, a = 2b = 2 × 30° = 60°
Now value of cos2 a + sin2 b
Put the values of a and b, we get
& cos2 (60°) + sin2 30°
2
1 1 2 1 1 2 1
&d2n +d n = + = =
2 4 4 4 2
8. (a)
Given, sin i + cos i = 2 cos i
Dividing both sides by cos , we get θ

sin i
+ 1 = 2 & tan i = 2 – 1
cos i

9. (a)
DE = ?
From figure, A
 DADE ∼ DABC (By AA similarity)
m
3c

? E
AD DE D
∴ =
AB BC

m
5c

3 DE 12.8 × 3 B C
⇒ = ⇒ DE = = 4.8 cm 12.5 cm
8 12.8 8
    

10. (d)
We have,
In DPAB and DPDC
BP AP
=
PC PD

∠APB = ∠DPC = 50° (Vertically opposite angles)


∴ DPAB ~ DPDC (By SAS similarity criteria)
∴ ∠A = ∠D = 30°
In DABP, we have
∠A + ∠B + ∠P = 180° ⇒ 30° + ∠B + 50° = 180°
⇒ ∠B = 100°
⇒ ∠PBA = 100°

Sample Papers 73
11. (c) Since OP and OQ are radii and AP and AQ are respective tangents

∴ ∠APO = ∠AOO = 90°


∠O = 360° – (90° + 90° + 90°) = 90°
Also, AP = AQ
⇒ AQOP is a square.

⇒ Radius of the circle, OP = 5 cm


12. (d)
Let r be the radius of circular garden
22
∴ 2πr = 88 m ⇒ 2 × × r = 88 ⇒ r = 14 m
7

Now, Radius of fence(circular) = 14 + 3.5 = 17.5 m


22
∴ Circumference of circular fence = 2πr = 2 × ×17.5 = 2 × 22 × 2.5 = 110 m
7

∴ Cost of fencing = `110 × 70 = `7700


4.2 2
×d n × 4. 2
1 2 1 22
13. (d) Volume of largest cone = r h= ×
3 3 7 2
p

1
= × 22 × 0.6 × 2.1 × 2.1 = 19.404 cm3
3

  = 19.4 cm3
14. (c) We have

Class interval Mid value (xi) Frequency (fi) fi . xi


0 – 10 5 22 110
10 – 20 15 24 360
20 – 30 25 35 875
30 – 40 35 30 1050
40 – 50 45 27 1215
50 – 60 55 m 55 m
60 – 70 65 6 390
70 –80 75 4 300
Total m + 148 55 m + 4300

Rfi xi 55 m + 4300
Thus, mean =
Rfi
& 31 =
m + 148

⇒ = 31 m + 4588 = 55 m + 4300
⇒ = 55 m – 31 m = 4588 – 4300 = 288
288
⇒ = 24 m = 288 m = = 12 ⇒ m = 12
24

i
15. (d) Length of the minor arc = × 2 rr
360°

45 22 1
= ×2× ×14 = × 88 = 11 cm
360° 7 8

74 Mathematics–X
16. (c)

Class Interval 0 –10 10 – 20 20 – 30 30 – 40 40 – 50 50 – 60


Frequency 3 9 15 30 18 5
Since maximum is 30 and its corresponding class interval is 30 – 40.
Modal class is fourth class i.e., 30 – 40.
8 8 1
17. (b) The probability of getting a prize = = =
8 + 16 24 3

18. (a)
We have,
3
4 tan x = 3 ⇒ tan x =
4

cos x + sin x 1 + tan x


∴ = (Divide Nr and Dr by cos x)
cos x – sin x 1 – tan x

3
1+
4
= =7
3

1–
4
19. (d)
6n = (2 × 3)n = 2n × 3n, Its prime factors do not contain 5n i.e., of the form 2m × 5n,
where m, n are natural numbers. Here assertion is incorrect but reason is correct.
20. (b) Using section formula, we have
k× 6 + 1 × ] – 3 g

– 1= ⇒ – k – 1 = 6k – 3
k+1

2
⇒ 7k = 2 ⇒ k =
7

⇒ Ratio be 2 : 7 internally.

So, both A and R are correct but R does not explain A.

SECTION-B
21. If the system of linear equations has infinitely many solutions then

a1 b1 c1
a2 = = c2 .
b

c –1 –2
⇒ = =
6 –2 –3

c 1 2 1 2
⇒ = ! Here !
6 2 3 2 3

b1 c1
i.e; ! c
b2

∴ There is no value of c for which lines have infinitely many solutions.


22. Firstly, in ABC, we have

LM || CB (Given)
Therefore, by Basic Proportionality Theorem, we have
AM AL
= ...(i)
AB AC

Sample Papers 75
Again, in ACD, we have

LN || CD (Given)

\ By Basic Proportionality Theorem, we have
AN AL
= ...(ii)
AD AC

AM AN
Now, from (i) and (ii), we have = .
AB AD

23. Let the tangents to a circle with centre O be ABC and XYZ.

Construction : Join OB and OY.


Draw OP||AC
 AB||PO

∴ ∠ABO + ∠POB = 180° (Adjacent interior angles)


∠ABO = 90° (A tangent to a circle is perpendicular to the radius through the point of


contact)
90° + ∠POB = 180° ⇒ ∠POB = 90°
Similarly, ∠POY = 90°

∠POB + ∠POY= 90° + 90° = 180°


Hence, BOY is a straight line passing through the centre of the circle.
24. Let r1 and r2 be the radius of two circles, respectively.

2rr1
∴ Ratio of the circumference of circles =
2rr2

3 r1 r1 3

4
= r & r2 = 4 ...(i)

2
Now, ratio of the area of two circles be
rr12 r12 r1
2
3 2
=f p =c m =
9
= from equation (i)

rr22 r22 r2 4 16

∴ Required ratio be 9 : 16.


OR
Let ABCD be a square of side x unit and a circle with centre O is inscribed in it.

Therefore, diameter of the circle = x D C


x
⇒ Radius of circle = unit
2

O
x 2
rc m
x2
2 r×
Area of circle 4 r
∴ = = =
Area of square (x) 2 2 4 A B

Thus, required ratio be p : 4.

76 Mathematics–X
25. Using Pythagoras Theorem, we have

PR2 = PQ2 + QR2


⇒ (13)2 = (12)2 + QR2 ⇒ 169 = 144 + QR2
⇒ QR2 = 169 – 144 = 25 ⇒ QR = 5 cm
QR 5 QR 5
Now, tan P = = and cot R = =
PQ 12 PQ 12

5 – 5
tan P – cot R = =0
12 12

\

OR
We have, tan (A + B) = 3
⇒ tan (A + B) = tan 60°

\ A + B = 60° …(i)
1
Again, tan (A – B) =
⇒ tan (A – B) = tan 30°
3

\ A – B = 30° …(ii)
Adding (i) and (ii), we have
2A = 90° ⇒ A = 45°

Putting the value of A in (i), we have


45° + B = 60°

\ B = 60° – 45° = 15°
Hence, A = 45° and B = 15°.

SECTION-C
26. On the contrary, let us assume that 2 + 5 3 is a rational number. So it is in the form of
a

, where a, b ∈ z, b ≠ 0 and HCF (a, b) = 1.


b a a
2+5 3 = ⇒5 3 =
b b

a – 2b a – 2b
5 3 = ⇒ 3 =
b 5b

a – 2b
Since it is given that 3 is irrational, but is rational.
5b

This contradicts our assumption that 2 + 5 3 is rational.


Hence, 2 + 5 3 is irrational.
27. Let a and b be the zeros of the polynomial. Then as per question b = 7a.

Now sum of zeros = a + b = a + 7a = – c m


–8
3

8 1 2k + 1
⇒ 8 a= or a= and a # b = a # 7a =
3 3 3
        

2k + 1 1 2 2k + 1
⇒ 7a2 = ⇒ 7c m =
3 3 3
    

Sample Papers 77
7 2k + 1 7
⇒ = ⇒ = 2k + 1
9 3 3

7 2
⇒ – 1 = 2k ⇒ k=
3 3

28. Let ` x be the fixed charge and ` y be the charge per km.

Therefore, charge paid for 10 km = x + 10y


⇒ 75 = x + 10y
⇒ x + 10y = 75 ...(i)
Also, charge paid for 15 km = x + 15y
⇒ x +15y = 110 ...(ii)
Subtracting equation (ii) from (i), we have
x + 10y = 75
x + 15y = 110

– – –
– 5y = – 35 &y=7
Putting the value of y = 7 in equation (i), we get
x + 10 × 7 = 75 ⇒ x = 75 – 70 = 5

x=5
∴ Fixed charge = ` 5 and charge per km is ` 7
Now, charge for covering a distance of 35 km
= x + 35 y
= 5 + 35 × 7 = 5 + 245 = ` 250
OR
Let total number of rows be y and total number of students in each row be x.

\ Total number of students = xy
Case I: If one student is extra in each row, there would be two rows less.

Now, number of rows = (y – 2)


Number of students in each row = (x + 1)
Total number of students = number of rows × number of students in each row
xy = (y – 2)(x + 1)

⇒ xy = xy + y – 2x – 2

⇒ xy – xy – y + 2x = –2

⇒ 2x – y = –2
…(i)
Case II: If one student is less in each row, there would be 3 rows more.
Now, number of rows = (y + 3) and number of students in each row = (x – 1)
Total number of students = number of rows × number of students in each row

\ xy = (y + 3)(x – 1)

⇒ xy = xy – y + 3x – 3

xy – xy + y – 3x = –3

⇒ – 3x + y = – 3
…(ii)

78 Mathematics–X
On adding equations (i) and (ii), we have
2x – y = – 2

– 3x + y = – 3
– x=– 5
or x =5

Putting the value of x in equation (i), we get


2 × 5 – y = –2

⇒ 10 – y = –2

– y = –2 – 10

⇒ – y = – 12 or y = 12

\ Total number of students in the class = 5 × 12 = 60.
sin A – 2 sin 3 A
29. To Prove : = tan A
2 cos 3 A – cos A

sin A (1 – 2 sin 2 A)
Simplifying LHS, =
cos A (2 cos 2 A –1)

sin A – 2 sin 3 A
2 cos 3 A – cos A

sin A 1 – 2 sin 2 A
= > H
cos A 2 cos 2 A –1

sin A sin 2 A + cos 2 A – 2 sin 2 A


= > H [a sin 2 A + cos 2 A = 1]
cos A 2 cos 2 A – (sin 2 A + cos 2 A)

sin A cos 2 A – sin 2 A sin A


= > H = ×1
cos A cos 2 A – sin 2 A cos A

sin A
= tan A <a = tan AF
cos A

LHS = RHS Hence Proved.


30. Given: A triangle ABC and a circle touching the side BC at P and AB, AC produce to at

Q and R respectively.
To prove: AQ = ] BC + CA + ABg
1
2

Proof: We know that lengths of tangents drawn from an


external point to a circle are equal.


⇒ AQ = AR, BQ = BP, CP = CR
Now, perimeter of ∆ABC = AB + BC + CA
= (AQ – BQ) + (BP + PC) + (AR – CR)
= AQ – BQ + BQ + CR + AQ – CR
 ( a AQ = AR, BP = BQ, PC = CR)
= 2 AQ
∴ AB + BC + CA= 2 AQ

Sample Papers 79
1
∴ AQ = (AB + BC + CA)
2

AQ = ] BC + CA + ABg
1
⇒ Proved
2

OR

Given: Chord AB.


Tangents AP and BP at A and B
To prove: +PAM = +PBM
Construction: Join centre O to P O
M
Let OP meet AB at M.
Proof: In DAMP and DBMP

AP = BP (Tangents from some point to a circle are equal)


MP = MP (Common side)
+APM = +BPM  (Tangents are equally dived to line joining the point of
emergence to circle's centre.)
by SAS criterion, DAMP , DBMP
by CPCT, +PAM = +PBM
Hence, tangents at end points of a chord make equal angle with it.
31. Total cards = 52

Cards removed = 6
Card lift = 52 – 6 = 46
Total black king = 2
2 1

Probability of drawing black king = =


46 23

Total red card = 26 – 6 = 20


20 10
Probability of drawing red colour card =
46 23

Total card of black colour = 26


26 13

Probability of drawing black colour card = =


46 23

SECTION-D
32. Let the original number of persons be x.

9000
∴ Each person will get amount = ` x
When 20 persons will increase
9000
∴ Each person will get = `
x + 20

ATQ,
9000 9000

x – x + 20 = 160
x + 20 – x
⇒ 9000 c x – m = 160 9000 e o = 160
1 1

x + 20 x ^ x + 20h

80 Mathematics–X
⇒ 180000 = 160 (x2 + 20x)
⇒ x2 + 20x = 1125 ⇒ x2 + 20x – 1125 = 0
⇒ x2 + 45x – 25x – 1125 = 0 ⇒ x(x + 45) – 25(x + 45) = 0
⇒ (x – 25) (x + 45) = 0
⇒ x – 25 = 0 (Since x + 45 ≠ 0 ⇒ x ≠ – 45)
∴ x = 25
∴ Original number of persons is 25.
OR
Let the digit at tens place be x.
18
Then, digit at unit place = x
18

\ Number = 10x + x
18
and number obtained by interchanging the digits = 10 × x + x
According to question,
a10x + x k – 63 = 10 # x + x a10x + x k – a10 # x + x k = 63
18 18 18 18

18 180 162
⇒ 10x + x – x – x = 63 ⇒ 9x – x – 63 = 0

⇒ 9x2 – 63x – 162 = 0 ⇒ x2 – 7x – 18 = 0


2
⇒ x – 9x + 2x – 18 = 0 ⇒ x(x – 9) + 2(x – 9) = 0
⇒ (x – 9) (x + 2) = 0 ⇒ x = 9 or x=–2
⇒ x=9 [ a digit can never be negative]
18
Hence, the required number = 10 × 9 + = 92.
9

33. Given: In ∆ABC, P is the mid-point of BC, Q is the mid-point of AP such that BQ

produced meets AC at R.
1
To prove: RA = CA
3

Construction: Draw PS ||BR, meeting AC at S.


Proof: In ∆BCR, P is the mid-point of BC and PS ||BR

\ S is the mid-point of CR.


⇒ CS = SR …(i)
In ∆APS, Q is the mid-point of AP and QR ||PS.

\ R is the mid-point of AS.
⇒ AR = RS …(ii)
From (i) and (ii), we get
AR = RS = SC
⇒ AC = AR + RS + SC = 3 AR
1 1
⇒ AR =AC = CA
3 3

Hence proved.

Sample Papers 81
34. For pipe, r = 1 cm

Length of water flowing in 1 sec, h = 0.7 m = 7 cm


Cylindrical Tank, R = 40 cm, rise in water level = H
Volume of water flowing in 1 sec = rr2 h = r ×1×1×70 = 70 p

Volume of water flowing in 60 sec = 70 × 60 p

Volume of water flowing in 30 minutes = 70 × 60 × 30 p

2
Volume of water in Tank = r H = p p × 40 × 40 × H
Volume of water in Tank = Volume of water flowing in 30 minutes
⇒ p × 40 × 40 × H = 70 × 60 × 30
p

⇒ H = 78.75 cm

OR
Total surface area of the top
= Curved surface area of hemisphere + Curved surface area of cone.
22 3.5 3.5
Now, the curved surface area of hemisphere = 2 r2 = c 2 # # # m cm2
7 2 2
p

Also, the height of the cone = Height of the top – Height (radius) of the hemispherical

part
3 .5
= c5 – m cm = 3.25 cm
2

3.5 2
So, the slant height of the cone (l) = r2 + h2 = c m + (3.25) 2 cm = 3.7 cm (approx)
2

22 3.5
Therefore, curved surface area of cone = rl = c # # 3.7 m cm2
7 2
p

22 3.5 3.5 22 3.5


Thus, the surface area of the top = c 2 # # # m cm2 + c # # 3.7 m cm2
7 2 2 7 2

22 3.5 22 3.5
= # (3.5 + 3.7) cm2 = # × 7.2 cm2
7 2 7 2

= 39.6 cm2 (approx).


35. We have,

For Mean
(fi) ( x i) xi – A
No. of wickets No. of bowlers Mid values ui = fi . ui
h
20 – 60 7 40 –3 –21
60 –100 5 80 –2 –10
100 – 140 16 120 –1 –16
140 – 180 12 160 = A 0 0
180 – 220 2 200 1 2
220 – 260 3 240 2 6
Total 45 –39

82 Mathematics–X
We have,
A = 160, h = 40, ∑fi = 45, ∑fiui = –39
/ fi .ui
Mean = A + #h
/ fi

–39 # 40
= 160 + = 160 – 34.67 = 125.33
45

For Median
Number of wickets Number of bowlers Cumulative Frequency
20 – 60 7 7
60 –100 5 12
100 – 140 16 28
140 – 180 12 40
180 – 220 2 42
220 – 260 3 45
Total 45

We have,
n
n = 45 ⇒ = 22.5
2
   

∴ 100 – 140 is the median class.


∴ l = 100, cf = 12, f = 16, h = 40
n
– cf
2
∴ Median = l + ×h
f

22.5 – 12 10.5 # 40
= 100 + # 40 = 100 +
16 16

= 100 + 26.25 = 126.25


∴ Median = 126.25

SECTION-E
36. (i) Production during 8th year, a8 = a + (8 – 1)d = a + 7d

= 5000 + 7 × 2200 = 5000 + 15400 = 20400


(ii) Total production in (during) first 3 years

= ^2a + ]n – 1g dh = ^2 × 5000 + 2 × 2200h


n 3
2 2

= 3(5000 + 2200) = 3 × 7200 = 21600


(iii) Let in nth year, the production is 29200.

⇒ an = 29200 ⇒ a + (n – 1)d = 29200


  ⇒ 5000 + (n – 1) × 2200 = 29200

24200
⇒ (n – 1) × 2200 = 24200 ⇒ (n – 1) = = 11
2200

⇒ n = 12
∴ In 12th year the production is 29200.

Sample Papers 83
OR
Difference of the production during 7th year and 4th year
= (a + 6d) – (a + 3d) = 3d
= 3 × 2200 = 6600
1
37. (i) Preet runs th distance AD on the eighth line and posts a red flag.
5

∴ Position of the red flag is c8, ×100 m , i.e., (8, 20).


1
5

Its position is (8, 20).


(ii) Distance between green flag and red flag is

= (20 – 25) 2 + (8 – 2) 2 = 25 + 36 = 61 units


2 + 8 25 + 20
(iii) Position of the blue flag = d , n = (5, 22.5)
2 2

OR
Let P (x, y) be the position of a flag posted by Joy. 1 4
Using section formula, we have G P R
1× 8 + 4 × 2 16 (2, 25) (x, y) (8, 20)
x= = = 3 .2
1+4 5

1× 20 + 4 × 25 120
y= = = 24
1+4 5

∴ Position of the flag posted by Joy is (3.2, 24).


A
300
38. (i) In DABC, tan 30° = x & x = 300 3 m

30°

(ii) Distance of boat from tower


= 300 3 – 300 ^ 3 – 1h = 300 m


300 m

Let the angle of depression = i .


30°
300
tan i = = 1 & i = 45° C x B
300

(iii) In DABC, we have


A
300 300
tan 60° =
x
& 3= x 60°

300 3 300 3
& x= × =
3
= 100 3 m 300 m
3 3

OR 60°
Difference of the distance = 300 – 100 3 C x B

= 300 – 100 × 1.73


= 300 – 173
= 127 m

zzz

84 Mathematics–X
SAMPLE PAPER
3 [SOLVED]

Time allowed: 3 hours Maximum marks: 80


General Instructions: Same as CBSE Sample Question Paper–2023 (Solved).

SECTION-A
Section A consists of 20 questions of 1 mark each.
1. The largest number which divides 70 and 125 leaving remainders 5 and 8 respectively

is
(a) 13
(b) 65 (c) 875
(d) 1750

2. The polynomial (x − a), where a > 0, is a factor of the polynomial q (x) = 4 2 x 2 – 2 .


1
Which of these is a polynomial whose factor is d x – n ?
a
(a) x2 + x + 6
(b) x2 + x – 6
(c) x2 – 5x + 4
(d) x2 + 4x – 3

2 2
3. Consider the equation kx + 2x = c(2x + b). For the equation to be quadratic, which of

these cannot be the value of k?


(a) 2c
(b) 3c (c) 4c
(d) 2c + 2b

4. For what value of k, do the equations 3x – y + 8 = 0 and 6x – ky = – 16 represent


coincident lines?
1 1
(a) (b) – (c) 2 (d) –2
2 2

5. The co-ordinates of the point which is reflection of point (–3, 5) in x-axis are

(a) (3, 5)
(b) (3, –5)
(c) (–3, –5)
(d) (–3, 5)

6. Two similar figures are shown.


10 m 76°

C
4m
5m 76° S x
2m
D 58° 2m y 98°
130° 98°
1m
B P Q
A 3m 6m

What are the values of x and y?


(a) x = 58°, y = 130°
(b) x = 98°, y = 76°
(c) x = 82°, y = 84°
(d) x = 130°, y = 84°

Sample Papers 85
7. In the given figure, from an external point P, two tangents PQ and
Q

PR are drawn to a circle of radius 4 cm with centre O. If ∠QPR =


P

4 cm
90°, then length of PQ is
(a) 3 cm
(b) 4 cm

O

(c) 2 cm
(d) 2 2 cm

R

AB AC
8. If in triangles ABC and DEF, = , then they will be similar
EF DE

when

(a) ∠A = ∠D
(b) ∠A = ∠E
(c) ∠B = ∠E
(d) ∠C = ∠F

9. The diameter of a wheel is 1 m. The number of revolutions it will make to travel a


distance of 22 km will be
(a) 2,800
(b) 4,000 (c) 5,500
(d) 7,000

10. A solid spherical ball fits exactly inside the cubical box of side 2a. The volume of the

ball is
16 3 1 3 32 3 4 3
(a) ra (b) ra (c) ra (d) ra
3 6 3 3

11. In the given figure, QR || AB, RP || BD, CQ = x + 2, QA = x,



A
x
CP = 5x + 4, PD = 3x. Q
x+
The value of x is 2

(a) 1 (b) 6

B
R
C

(c) 3
(d) 9

+ 4
5x

3 – sin2 60°
P
3x
12. What is the value of ? D
tan 30° tan 60°

1 1 3 3
(a) 2 (b) 3 (c) 2 (d) 3
4 4 4 4

D C
13. A paper is in the form of a rectangle ABCD in which

AB = 18 cm and BC = 14 cm. A semicircular portion with


BC as diameter is cut off. The area of the remaining paper is 14 cm
2 2
(a) 175 cm
(b) 165 cm

2
(c) 145 cm

(d) none of them



A 18 cm B

14. If sin 77° = x, then the value of tan 77° is


1 x x
(a) 2
(b) (c) (d) None of these
+ 1 + x2 1 – x2

1 x
15. Consider the following frequency distribution:

Class 0–5 6–11 12–17 18–23 24–29


Frequency 13 10 15 8 11
The upper limit of the median class is
(a) 17
(b) 17.5
(c) 18 (d) 18.5

86 Mathematics–X
16. A bag contains 3 red, 5 black and 7 white balls. A ball is drawn from the bag at random.

The probability that the ball drawn is not black, is


1 9 1 2
(a) (b) (c) (d)
3 15 2 3

17. The two legs AB and BC of a right triangle ABC are in a ratio 1 : 3. What will be the

value of sin C ?
1 3 1
(a) 10 (b) (c) (d)
10 10 2

18. For the following distribution


Class 0–5 5–10 10–15 15–20 20–25


Frequency 10 15 12 20 9

The sum of lower limits of the median class and modal class is
(a) 15
(b) 25
(c) 30
(d) 35

DIRECTION: In the question numbers 19 and 20, a statement of assertion (A) is followed by a
statement of Reason (R). Choose the correct option.
(a) Both A and R are true and R is the correct explanation for A.
(b) Both A and R are true and R is not the correct explanation for A.
(c) A is true but R is false.
(d) A is false but R is true.
19. Assertion (A) : 2 +
3 is an irrational number.
Reason (R) : Sum of rational and irrational number is always irrational.

20. Assertion (A) : The point (0, 4) lies on y-axis.


Reason (R) : The x co-ordinate of the point on y-axis is zero.


SECTION-B
Section B consists of 5 questions of 2 marks each.
21. Solve the following pair of linear equations:

3x – 5y = 4
2y + 7 = 9x
22. In Fig., DE  BC. If AD = x, DB = x – 2, AE = x + 2 and EC = x – 1, find the value of x.

Sample Papers 87
23. In Fig., AB is the diameter of a circle with centre O and AT is a tangent if

∠AOQ = 58°, find ∠ATQ.


B

O
58° Q

A T

cot2 a
24. Prove that 1 + = cosec a.
1 + cosec a

OR
3
If sin A = , calculate cos A and tan A.
4
25. In figure, sectors of two concentric circles of radii 7 cm and 3.5 cm are shown. Find the

area of the shaded region.

OR
The circumference of a circle exceeds the diameter by 16.8 cm. Find the radius of the
circle.

Section-C
Section C consists of 6 questions of 3 marks each.

26. Find HCF and LCM of 404 and 96 and verify that HCF × LCM = Product of the two

given numbers.
11 2
27. Find the zeros of the quadratic polynomial 7y2 – y– and verify the relationship
3 3

between zeros and the coefficients.


28. The sum of the digits of a two-digit number is 9. Also, nine times this number is twice

the number obtained by reversing the order of the digits. Find the number.
OR
The area of a rectangle gets reduced by 9 square units, if its length is reduced by 5 units
and breadth is increased by 3 units. If we increase the length by 3 units and the breadth
by 2 units, the area increases by 67 square units. Find the dimensions of the rectangle.

88 Mathematics–X
29. In Fig., two equal circles, with centres O and O', touch each other at X. OO' produced

meets the circle with centre O' at A. AC is tangent to the circle with centre O, at the point
DO'
C. O'D is perpendicular to AC. Find the value of .
CO

OR
If a, b, c are the sides of a right triangle where c is the hypotenuse, prove that the radius
a+b– c
r of the circle which touches the sides of the triangle is given by r = .
2
30. Prove that: (sin θ + 1 + cos θ)(sin θ – 1 + cos θ). sec θ cosec θ = 2

31. A game of chance consists of spinning an arrow which comes to rest pointing at one of the

numbers 1, 2, 3, 4, 5, 6, 7, 8 (see Fig.), and these are equally likely outcomes. What is the
probability that it will point at:

(i) 8 ?

(ii) an odd number?


(iii) a number greater than 2?


(iv) a number less than 9?


SECTION-D
Section D consists of 4 questions of 5 marks each.
32. In a class test, the sum of Shefali’s marks in Mathematics and English is 30. Had she got

2 marks more in Mathematics and 3 marks less in English, the product of her marks
would have been 210. Find her marks in the two subjects.
OR
The diagonal of a rectangular field is 60 metres more than the shorter side. If the longer
side is 30 metres more than the shorter side, find the sides of the field.
33. CD and GH are respectively the bisectors of ∠ACB and ∠EGF such that D and H lie on

sides AB and FE of ∆ABC and ∆EFG. If ∆ABC ~ ∆FEG, show that


CD AC
(i) = (ii) ∆DCB ~ ∆HGE (iii) ∆DCA ~ ∆HGF
GH FG

Sample Papers 89
34. A well of diameter 3 m is dug 14 m deep. The earth taken out of it has been spread

evenly all around it in the shape of a circular ring of width 4 m to form an embankment.
Find the height of the embankment.
OR
A sphere of diameter 12 cm, is dropped in a right circular cylindrical vessel, partly,
filled with water. If the sphere is completely submerged in water, the water level in the
5
cylindrical vessel rises by 3 cm. Find the diameter of the cylindrical vessel.
9
35. The mean of the following frequency table is 50. But the frequencies f1 and f2 in class

20–40 and 60–80 respectively are missing. Find the missing frequencies.
Classes 0–20 20–40 40–60 60–80 80–100 Total
Frequency 17 f1 32 f2 19 120

SECTION-E
Case study based questions are compulsory.
36. Stadium seating surrounds the centre pitch. Each row in the seating is positioned at

a slightly higher level than the one in front of it. A safe seating - standing section of a
stadium is shown in the figure.

There are 22 rows in the section. Each row in the section is 700 mm in height, excluding
the seat and has one more seat than the previous row, starting from the second row. The
first row has 5 seats.
On the basis of above information answer the following questions.
(i) Aarav is seating in the centre of Row 15 in the section then how many seats are on

his right?
(ii) Write the total seating capacity of the section.

(iii) Ramesh is standing in row 20 and Rohit is standing in Row 2, then how much

higher is Ramesh‘s row than Rohit‘s?


OR
The height of the section is measured from the foot of the first row to the row. What
is the height of the section?

90 Mathematics–X
37. Shaurya made a map of his locality on a coordinate plane.


Y
Kartik’s Temple
House

Post Office
School Park
O
X
Shaurya’s
Market House Railway
Station
Scale : 1 Square = 1 unit

On the basis of above information answer the following questions.


(i) If he considered his house as the origin, then write the coordinates of market.

(ii) Write the distance of his friend Kartik’s house from his house.

(iii) There is a fort at a distance of 10 units from his house. If its ordinate is 6, then find

its abscissa.
OR
Find the co-ordinates of the point which divides the line segment joining the points
denoted school and park internally in the ratio 3 : 2.
38. At the toll plaza, a traffic monitoring camera is installed at a height of 5.7 m. it takes

pictures of moving vehicles at regular intervals.


The diagram below shows the position of the camera and a car moving away from it after
paying the toll in four instances.
C C C C

A B A B A B A B

On the basis of above information answer the following questions.


(i) When the car makes on elevation of 30° with the camera, then how far is the car

away from BC ? (Use 3 = 1.73)


(ii) How for is the car away from BC when angle of depression of the car from camera

is 60°?
(iii) When angle of depression of the car increases from 30° to 45° then write the distance

travelled during this.


OR
Find the distance travelled by the car when angle of elevation of the car changes
from 30° to 60°.

zzz

Sample Papers 91
SECTION-A
1. (a) Required number = HCF of (70 – 5, 125 – 8)

= HCF (65, 117)


Now, prime factors of 65 = 5 × 13
117 = 9 × 13
Hence, HCF (65, 117) = 13
2. (b)
We have q(x) = 4 2 x2 – 2

Since, (x – a) is a factor of q(x).


\ q(a) = 0


2
4 2 (a) – 2 = 0 ⇒
4a2 – 1 = 0

1 1
⇒ a2 = ⇒ =4
4 a2

1

a = ±2

1
\ x – a = (x – 2) or (x + 2)

Now, x2 + x – 6 = x2 + 3x – 2x – 6 = x(x + 3) – 2(x + 3) = (x – 2) (x + 3)


⇒ (x – 2) is a factor of x2 + x – 6

\ c x – a m is a factor of x2 + x – 6.
1

3. (a) Given equation,


kx2 + 2x = c(2x2 + b)
⇒ kx2 + 2x = 2cx2 + bc
⇒ (k – 2c)x2 + 2x – bc = 0

For this equation to be quadratic,


k – 2c ≠ 0
⇒ k ≠ 2c
4. (c) Given pair of linear equations

3x – y + 8 = 0 and 6x – ky = – 16
6x – ky + 16 = 0
For both lines to be coincident, we have
a1 b1 c1

a2 = b = c2
2
3 –1 8 1 1 1

6
=
–k
=
16
& 2
= =
k 2
   ⇒ k = 2

5. (c) We have,

Centre of the circle is the mid-point of the end points of its diameter.

n i.e c0, m .
– 6+6 3+ 4
Co-ordinates of the centre are d
7
,
2 2 2

92 Mathematics–X
6. (a)
We have,
AB = 3 m, BC = 2 m, CD = 5 m, DA = 1 m
and PQ = 6 m, QR = 4 m, RS = 10 m, SP = 2 m
AB BC CD DA 1
` = = = =
PQ QR RS SP 2

` Quadrilateral ABCD ~ 4PQRS


` ∠A = ∠P and ∠D = ∠S
(Corresponding angles)
& 130° = y and 58° = x
` x = 58° and y = 130°

7. (b) Join OR it is given that ∠QPR = 90°


Now, ∠OQP= ∠ORP = 90° (Since PQ and PR an tangent to the cirlce at Q and R


respectively, and OQ and OR are radii)


∴ ∠QOR = 90°
Q
In POQ and POR, we have
D D

4 cm
PQ = PR (Tangent drawn for an extarnal


point) O

OQ = OR (Radii) R

OP = OP (Common)
∴D POQ ≅ POR D (By SAS congruency)
∴ ∠QPO = ∠RPO = 45° and ∠QOP = ∠ROP = 45°
∴ ∠QOP = ∠QPO = 45°
⇒ OQ = PQ = 4 cm
Hence, PQ = 4 cm
AB AC A E
8. (b) Given, =
EF DE

For two triangles (ABC and EFD) to be similar when


∠A = ∠E (Corresponding angles)
C F D

22 1 22
9. (d) Circumference of the wheel = 2 × × m= m
7 2 7

22
Revolutions made in m=1
7

7
Revolutions made in 22000 m = × 22000 = 7000
22

10. (d) Since the solid spherical ball exactly fits inside the cubical box of side 2a.

Therefore, diameter of spherical ball = 2a


⇒ 2r = 2a ⇒ r=a
Radius of sphrical ball = a
4 3 4 3
Volume of the spherical ball = pr = pa
3 3

Sample Papers 93
11. (a) In ABC, QR  AB
D

AQ BR
= ...(i)
QC RC

Also, in DBC , RP  BD
D

DP BR
= ...(ii)
PC RC

From (i) and (ii), we have


AQ DP x 3x 1 3

QC
=
PC

x+2
=
5x + 4
& x+2
=
5x + 4
⇒ 5x + 4 = 3x + 6
⇒   2x = 2    ⇒    x=1
12. (a) We have,

2
3
3 – sin2 60° 3–e o
3 1
2
= =3– =2
tan 30° . tan 60° 1 4 4

× 3
3
13. (a) Area of remaining paper = area of rectangle – area of semicircle

r× ( 7 ) 2
= 18 ×14 –
2

22 7 × 7
= 252 – ×
7 2

=252 – 77 = 175 cm2 A

14. (c) We have,


sin 77° = x
cos 77° = 1 – x2 [a cos2 i = 1 – sin2 i]
x

x
tan 77° =
1 – x2

77°
15. (b)

B C

Class Interval 0 – 5.5 5.5 – 11.5 11.5 – 17.5 17.5 – 23.5 23.5 – 29.5
Frequency 13 10 15 8 11
Cumulative 13 23 38 46 57
frequency

ca = 28.5 m
n 57
Median class = 11.5 – 17.5 =
2 2

Hence the upper limit of the median class is 17.5.


16. (d) Total number of balls = (3 red + 5 black + 7 white) balls

= 15 balls
∴ Total number of possible outcomes = 15
and number of favourable outcomes i.e., not black = 3 + 7 = 10
10 2
Required probability = =
15 3

94 Mathematics–X
A
17. (b)
Let AB = x, BC = 3x
` AC = AB2 + AC2 = x2 + 9x2 = 10x2 = 10 x 10x
x
AB x 1
` sin C = = =
AC 10 x 10

18. (b)

B 3x C

Class Interval 0–5 5–10 10–15 15–20 20–25


Frequency 10 15 12 20 9
Cumulative 10 25 37 57 66
frequency

n 66
= = 33 ⇒ Median class = 10 – 15
2 2

⇒ Modal class = 15 – 20

Maximum frequency is 20 of the class 15 – 20.

Sum = 10 + 15 = 25

19. (a) As we know that sum of rational and irrational number is always an irrational.

Here 2 is rational and 3 is irrational.


Thus, both A and R are true and R is the correct explanation for A.
20. (a) The x co-ordinate of the point (0, 4) is zero.

∴ Point (0, 4) lies on y-axis.


So, both A and R are correct and R explains A.

SECTION-B
21. Given

3x – 5y = 4 ...(1)
9x – 2y =7 ...(2)
To find ‘x’ and ‘y’
Multiplying (1) × 3, (2) × 1 and subtracting (2) from (1), we get
(3x – 5y) × 3 – (9x – 2y) × 1 = 4 × 3 – 7 × 1

& 9x – 15y – 9x + 2y = 12 – 7
–5
& –13y = 5 & y=
13
–5
Then, Putting y = in (1), ,we get
13

3x = 4 + 5y
–5 52 – 25
& 3x = 4 + 5 × d n & 3x =
13 13

27 9
& 3 x = 13 & x=
13

Sample Papers 95
22. In ∆ABC, we have DE  BC

AD AE
\ = [By Basic Proportionality Theorem]
DB EC

x x+2
⇒ = ⇒ x(x – 1) = (x – 2) (x + 2)
x –2 x –1

⇒ x2 – x = x 2 – 4 ⇒ x=4

23. We have,

∠AOQ = 58°
1 1
+ABQ = +AOQ = × 58° = 29°
2 2

(Angle on the circumference of the circle by the same arc)


∠BAT = 90° ( OA ⊥ AT)

∴ ∠ATQ = 90° – 29° = 61°

cot2 a
24. LHS = 1+
1 + cosec a

cosec2 a – 1
= 1+
1 + cosec a

(cosec a – 1) (cosec a + 1)
= 1+
(1 + cosec a)

= 1 + cosec α –1
= cosec α = RHS Proved
OR
Let us first draw a right ABC in which ∠C = 90°.
D

Now, we know that


Perpendicular BC 3
sin A = = =
Hypotenuse AB 4

Let BC = 3k and AB = 4k, where k is a positive number.


Then, by Pythagoras Theorem, we have
AB2 = BC2 + AC2 ⇒ (4k)2 = (3k)2 + AC2
⇒ 16k2 – 9k2 = AC2 ⇒ 7k2 = AC2

\ AC = 7k
AC 7k 7 BC 3k 3
\ cos A = = = and tan=
A =
AC
=
AB 4k 4 7k 7

25. Let A1 and A2 be the areas of sectors OAB and OCD


respectively. Then, A1 = area of a sector of angle 30° in a


circle of radius 7 cm.

cUsing : A = # rr 2 m
22
A1 = ' # ( 7) 2 1
30° i
⇒ #
360° 7 360°

96 Mathematics–X
22
=' # 7 # 71
1 77
# ⇒ A1 = cm2
12 7 6

A2 = area of a sector of angle 30° in a circle of radius 3.5 cm


22 22 7 7
=' # (3.5) 2 1 ⇒
30° 77
' # # 1=
1
# # cm2
360° 7 12 7 2 2 24

Area of the shaded region = A1 – A2 = c


77 77
\ – m cm2
6 24

77 77
= × (4 – 1) cm2 = cm2 = 9.625 cm2
24 8

OR
Let the radius of the circle be r cm. Then,
Diameter = 2r cm and circumference = 2pr cm
According to question,
\ Circumference = diameter + 16.8 ⇒ 2pr = 2r + 16.8
22
⇒ 2# # r = 2r +16.8 ⇒ 44r = 14r + 16.8 × 7
7

⇒ 44r – 14r = 117.6 or 30r = 117.6


117.6
⇒ r= = 3.92
30

Hence, radius = 3.92 cm.

SECTION-C
26. Prime factors of

404 = 2 × 2 × 101 = 22 × 101 and


96 = 2 × 2 × 2 × 2 × 2 × 3 = 25 × 3

∴ HCF of 404 and 96 = 22 = 4
LCM of 404 and 96 = 101 × 25 × 3 = 9696
HCF × LCM = 4 × 9696 = 38784
Also 404 × 96 = 38784
Hence HCF × LCM = Product of 404 and 96.
11 2
27. Given: p(y) = 7y2 – y–
3 3

1 1 2
= (21y2 –11y –2) = (21y – 14y + 3y – 2)
3 3

1 1
= [7y (3y – 2) + 1 (3y – 2)] = (7y + 1) (3y – 2)
3 3

Equating p(y) = 0
1 –1 2
⇒ (7y + 1) (3y – 2) = 0 ⇒ y= and y =
3 7 3

Sample Papers 97
–c m
–11
–b 3 11 2 1 14 – 3 11
Now, Sum of zeros = = = and – = =
a 7 21 3 7 21 21

2

c 3 –2 –1 2 –2
and product of zeros = = = and × = Hence verified.
a 7 21 7 3 21

28. Let the digit at tens and unit place of the two digit number be x and y respectively.

Therefore required two digit number = 10x + y


According to question
x+y=9 ...(i)
and 9(10x + y) = 2(10y + x) ⇒ 90x + 9y = 20y + 2x
⇒ 90x + 9y – 20y – 2x = 0 ⇒ 88x – 11y = 0
⇒ 8x – y = 0 ...(ii)
Adding equation (i) and (ii), we get
x + y + 8x – y = 9 + 0 ⇒ 9x = 9 ⇒ x=1
Putting x = 1 in (i), we get
1+y=9 ⇒ y=8
Hence, required number = 10 × 1+ 8 = 18
OR
Let the length and breadth of a rectangle be x and y respectively.
Then area of the rectangle = xy
According to question, we have
(x – 5) (y + 3) = xy – 9 ⇒ xy + 3x – 5y – 15 = xy – 9
⇒ 3x – 5y = 15 – 9 = 6 ⇒ 3x – 5y = 6 …(i)
Again, we have
(x + 3) (y + 2) = xy + 67 ⇒ xy + 2x + 3y + 6 = xy + 67
⇒ 2x + 3y = 67 – 6 = 61 ⇒ 2x + 3y = 61 …(ii)
6 + 5y
Now, from equation (i), we express the value of x in terms of y as x = .
3

Substituting the value of x in equation (ii), we have


6 + 5y 12 +10y + 9y
2#d n + 3y = 61 ⇒ = 61
3 3

171
⇒ 19y = 183 – 12 = 171 ⇒ y= =9
19

Putting the value of y in equation (i), we have


3x – 5 × 9 = 6 ⇒ 3x = 6 + 45 = 51
51
\ x= =17
3

Hence, the length of rectangle = 17 units and breadth of rectangle = 9 units.

98 Mathematics–X
29. AC is tangent to circle with centre O.

Thus ∠ACO = 90°

In AO'D and AOC


D D

∠ADO' =∠ACO = 90°


(Tangent is perpendicular to radius.)
∠A = ∠A
(Common)
\ D AO'D ~∠AOC
(By AA similarity)
AO′ DO′
⇒ =
AO CO

DO′ r 1
\ = = ( AO = AO' + O' X+ XO = 3r)
CO 3r 3

OR
Let the circle touches the sides BC, CA and AB of the A
right triangle ABC at D, E and F respectively, where
BC = a, CA = b, and AB = c. (See figure)
Then AE = AF and BD = BF b
Also CE = CD = r F
r
i.e, AF = AE – r, a – r = BF = BD r
c

O E
or AB = c = AF + BF = b – r + a – r
a+b– c r
This gives, r = .
2

B D C
OR a

LHS = (sin θ + 1 + cos θ)(sin θ – 1 + cos θ). sec θ cosec θ


=(sin θ + cos θ + 1)(sin θ + cos θ – 1). sec θ cosec θ
= {(sin θ + cos θ)2 – (1)2}. sec θ cosec θ
= {sin2θ + cos2θ + 2sin θ . cos θ – 1}. sec θ cosec θ
1 1
= ^1 + 2 sin i cos i – 1h × = 2 sin i. cos i× = 2 = RHS
cos i. sin i sin i. cos i
30. LHS
= (sin θ + 1 + cos θ)(sin θ – 1 + cos θ). sec θ cosec θ
=(sin θ + cos θ + 1)(sin θ + cos θ – 1). sec θ cosec θ
= {(sin θ + cos θ)2 – (1)2}. sec θ cosec θ
= {sin2θ + cos2θ + 2sin θ . cos θ – 1}. sec θ cosec θ
1
= ^1 + 2 sin i cos i – 1h ×
cos i. sin i
1
= 2 sin i. cos i× = 2 = RHS
sin i. cos i

Sample Papers 99
31. Here, total number of elementary events (possible outcomes) = 8

(i) We have only one ‘8’ on the spining plant.


\ Favourable number of outcomes = 1


1
Hence, the probability that arrow points at 8 = .
8

(ii) We have four odd points (i.e.,1, 3, 5 and 7)


\ Favourable number of outcomes = 4


4 1
Hence, probability that arrow points at an odd number = = .
8 2

(iii) We have 6 numbers greater than 2, i.e., 3, 4, 5, 6, 7 and 8.


Therefore, favourable number of outcomes = 6


6 3
Probability that arrow points at a number greater than 2 = = .
8 4

(iv) We have 8 numbers less than 9, i.e, 1, 2, 3, ... 8.


\ Favourable number of outcomes = 8
8
Hence, probability that arrow points at a number less than 9 = = 1.
8

SECTION-D
32. Let Shefali’s marks in Mathematics be x.

Therefore, Shefali’s marks in English is (30 – x).


Now, according to question,
(x + 2) (30 – x – 3) = 210 ⇒ (x + 2) (27 – x) = 210
2
⇒ 27x – x + 54 – 2x = 210 ⇒ 25x – x2 + 54 – 210 = 0
⇒ 25x – x2 – 156 = 0 ⇒ – (x2 – 25x + 156) = 0
⇒ x2 – 25x + 156 = 0 ⇒ x2 – 13x – 12x + 156 = 0
⇒ x(x – 13) – 12(x – 13) = 0 ⇒ (x – 13) (x – 12) = 0
Either x – 13 = 0 or
x – 12 = 0
⇒ x = 13 or x = 12

∴ Shefali’s marks in Mathematics = 13, marks in English = 30 – 13 = 17
or Shefali’s marks in Mathematics = 12, marks in English = 30 – 12 = 18.

OR
Let ABCD be the rectangular field. Let the shorter side BC of the rectangle = x metres.
According to question, D C
Diagonal of the rectangle, AC = (x + 60) metres
side of the rectangle, AB =(x + 30) metres
By Pythagoras theorem, AC2 = AB2 + BC2
\ (x + 60)2 = (x + 30)2 + x2
or x2 + 120x + 3600 = x2 + 60x + 900 + x2

\ (2x2 – x2) + (60x – 120x) + 900 – 3600 = 0 A B


2
or x – 60x – 2700
=0

100 Mathematics–X
or (x – 90)(x + 30)
=0
\ Either x – 90 = 0 or x + 30 = 0
⇒ x = 90 or x = –30 (But side cannot be negative)
So, the shorter side of rectangle = 90 m
and longer side of rectangle = 120 m
33.
∆ABC ∼ ∆FEG, [Given]
⇒ ∠A = ∠F
∠B = ∠E
∠C = ∠G
AB BC AC
and = = ...(a)
FE EG FG

(i) In ∆ACD and ∆FGH


∠A = ∠F [Given]
1 1
and ∠1 = ∠2 [ + C = + G]
2 2


∴ ∆ACD ∼ ∆FGH [AA Similarity]
CD AC
⇒ = [Corresponding sides of similar triangles]
GH FG

CD AC
(ii) = [Proved above]
GH FG

AC BC
But = [From (a)]
FG EG C G

1 3 42
CD BC
=
GH EG

A B E F
In ∆DCB ∼ ∆HGE D H

1 1
∠3 = ∠4 [ + C = + G]
2 2

CD BC
and = [Proved above]
GH EG


∴ ∆DCB ∼ ∆HGE [SAS Similarity]

(iii)In ∆DCA and ∆HGF,


1 1
∠1 = ∠2 [ +C = +G ]
2 2

CD AC
and = [Proved above]
GH FG

⇒ ∆DCA ∼ ∆HGF [SAS Similarity]


9
34. Volume of earth dug out = r × ×14 m3
4

3 3 11 3
Radii of circular ring formed = 4 + , m = m, m
2 2 2 2

Sample Papers 101


Let height of embankment = h

rh c – m = r × ×14
121 9 9
4 4 4

9 1
⇒ h = m or 1 m
8 8
   

OR
4
Volume of sphere = p(6)3 cm3
3

 32 
Volume of water rise in cylinder = pr2   cm3
 9 

32 2 4 4 × 2 × 36 × 9
\ rr = r (6) 3 ⇒ r2 = = 81
9 3 32

⇒ r = 9 cm
\ Diameter of the cylindrical vessel = 2r = 2 × 9 = 18.
35. Let the assumed mean A = 50 and h = 20.

Calculation of mean
xi – 50
Class interval Mid-values (xi) Frequency (fi) ui = fi ui
20
0–20 10 17 –2 –34
20–40 30 f1 –1 –f1
40–60 50 32 0 0
60–80 70 f2 1 f2
80–100 90 19 2 38
Total Sfi = 68 + f1 + f2 Sfiui = 4 – f1 + f2

We have, Sfi = 120 (Given)


⇒ 68 + f1 + f2 = 120
⇒ f1 + f2 = 52 ...(i)
Now, mean = 50
Rfi ui
⇒ Xr = A + h f p
Rfi

4 – f1 + f2
⇒ 50 = 50 + 20 × ) 3
120
4 – f1 + f2 4 – f1 + f2
⇒ 50 = 50 + ⇒ 0 =
6 6

⇒ f1 – f2 = 4 ...(ii)
By adding equations (i) and (ii), we get
f1 + f2 = 52
f1 – f2 = 4
2f1 = 56
⇒ f1 = 28

102 Mathematics–X
Putting the value of f1 in equation (i), we get
28 + f2 = 52 ⇒ f2 = 24
Hence, the missing frequencies f1 is 28 and f2 is 24.

SECTION-E
36. (i) We have number of seats which are in AP.

5, 6, 7, ...
` a = 5, d = 6 – 5 = 1 and
a = 15
` a15 = a + (15 – 1)d = 5 + 14 × 1 = 19
Therefore 9 seats are on his right.
n
(ii) Sn = (2a + (a – 1) d)
2

& S22 = 22 2
(2 × 5 + 21×1) = 11 (10 + 21) = 11× 31 = 341

(iii) Required difference in height = 20 × 700 mm – 2 × 700 mm


= (14000 – 1400) = 12600 mm


= 12.6 meter
OR
We have height of the row from the foot of the first row are
700 mm, 1400 mm, 2100 mm, ... to 22 terms which are in AP.
` a = 700, d = 1400 – 700 = 700 mm, n = 22
n
` Sum, Sn = (2a + (n – 1) d)
2

22
= ^2 × 700 + 21× 700h
2

  = 11(1400 + 14700) = 11 × 16,100 = 177100 mm


= 177.1 m
37. (i) If Shaurya's house is at (0, 0), then market is at (–3, –1).

(ii) Kartik's house is at (–2, 4).


Distance between (0, 0) and (–2, 4) = (–2 – 0) 2 + (4 – 0) 2 = 4 + 16 = 20 units


(iii) Let coordinate of the fort be (x , 6).

Then distance between (0, 0) and (x, 6) = 10.


⇒ (x – 0) 2 + (6 – 0) 2 = 10 ⇒ x2 + 36 = 100
2
⇒ x = 64 ⇒ x=±8
OR
Coordinate of school = (–5, 2) and park = (0, 2)
3 × 0 + 2 × ^ –5 h 3 × 2 + 2 × 2
Coordinate of required point = f , p
3+2 3+2

=d , n = _ –2 , 2 i
–10 10
5 5

Sample Papers 103


38. (i)
C

5.7 m

30°
A B

In right ABC, we have


D

BC 5.7 1
tan 30° = ⇒ =
AB 3 AB

` AB = 5.7 × 3 = 5.7 × 1.73 = 9.861 m


BC
(ii) We have, tan 60° = C
AB

5.7 5.7 3
⇒ 3= ⇒ x= ×
x 3 3

5.7 m
5.7 3
= = 1.9 3 = 1.9 × 1.73 m = 3.287 m
3

60°
(iii) In ABC, we have
D
A B
5.7 1 5.7 C
tan 30° =
AB
& =
AB
⇒ AB = 9.861 m
3

and, in A’BC, we have


D

BC 5.7 m
5.7
tan 45° = ⇒ 1= ⇒ A′B = 5.7 m
AB
l Al B
    

30° 45°
` AAl = AB – Al B = 9.861 – 5.7 = 4.161 m
A A‘ B

OR C
D In ABC, we have
5.7 1 5.7
tan 30° =
AB
& =
3 AB

5.7 m
⇒ AB = 5.7 × 3 = 5.7 × 1.73 = 9.861 m
30° 60°
In A'BC, we have
A A‘ B
D

5.7 5.7 5.7 3


tan 60° =
AB
& 3=
Al B
& Al B = ×
l 3 3

5.7 3
=
3

= 1.9 × 1.73 = 3.287 m



∴ AA' = 9.861 – 3.287 = 6.574

z zz

104 Mathematics–X
SAMPLE PAPER
4 [SOLVED]

Time allowed: 3 hours Maximum marks: 80


General Instructions: Same as CBSE Sample Question Paper–2023 (Solved).

SECTION-A
Section A consists of 20 questions of 1 mark each.

1. The largest number which divides 70 and 125 leaving remainders 5 and 8

respectively is
(a) 13
(b) 65
(c) 875
(d) 1750

2. If one of the zeroes of the quadratic polynomial x2 + 3x + k is 2, then the value of k is


(a) 10
(b) – 10
(c) – 7
(d) – 2

3. Consider the equations shown. p + q = 5 and p − q = 2. Which of these are the values

of p and q?
(a) p = 1.5, q = 3.5
(b) p = 3.5, q = 1.5
(c) p = 2, q = 3
(d) p = 3, q = 2

4. If the list price of a toy is reduced by ™ 2, a person can buy 2 toys more for ™ 360. The

original price of the toy is


(a) ™18
(b) ™20
(c) ™19
(d) ™21

AB BC CA
5. If in two triangles ABC and PQR, = = , then
QR PR PQ

(a) DPQR ~ DCAB


(b) DPQR ~ DABC
(c) DCBA ~ DPQR
(d) DBCA~DPQR

6. It is given that DABC ~ DDFE, ∠A = 30°, ∠C = 50°, AB = 5 cm, AC = 8 cm and


DF = 7.5 cm. Then, which of the following is true?


(a) DE = 12 cm, ∠F = 50°

(b)DE = 12 cm, ∠F = 100°
(c) EF = 12 cm, ∠D = 100°

(d)EF = 12 cm, ∠D = 30°


7. The mid point of the line segment joining the points (– 5, 7) and (– 1, 3) is

(a) (– 3, 7)
(b) (– 3, 5)
(c) (– 1, 5)
(d) (5, – 3)

2 tan 30°
8. The value of
1 – tan2 30°

(a) cos 60°


(b) sin 60°
(c) tan 60°
(d) sin 30°

Sample Papers 105


9. If tan x + sin x = m and tan x – sin x = n then m2 – n2 is equal to

(a) 4 mn
(b) mn (c) 2 mn
(d) none of these

10. If two tangents inclined at an angle 60° are drawn to a circle of radius 3 cm, then

length of each tangent is equal to


3
(a) 3 cm (b) 6 cm (c) 3 cm (d) 3 3 cm
2

11. If the area of a circle is 154 cm2, then its perimeter is


(a) 11 cm
(b) 22 cm
(c) 44 cm
(d) 55 cm

2 2
12. If
3 tan q = 3 sin q, (q ≠ 0) then the value of sin q – cos q is
1 2 3
(a) (b) 3 (c) (d)
3 3 2

13. A wire can be bent in the form of a circle of radius 35 cm. If it is bent in the form of a

square, then its area will be


3025
(a) 3025 cm2 (b) cm2 (c) 1225 cm2 (d) 2450 cm2
2

14. In DLMN and DPQR, ∠L = ∠P, ∠N = ∠R and MN = 2QR. Then the two triangles are

(a) congruent but not similar


(b) similar but not congruent

(c) neither congruent nor similar


(d) congruent as well as similar

3
15. The radius of a sphere whose volume is 12p cm is

(a) 3 cm
(b) 3 3 cm
(c) 32/3 cm
(d) 31/3 cm

16. Consider the data:


Class 65–85 85–105 105–125 125–145 145–165 165–185 185–205


Frequency 4 5 13 20 14 7 4

The difference of the upper limit of the median class and the lower limit of the modal
class is
(a) 0 (b) 19
(c) 20
(d) 38

17. The probability that a non-leap year selected at random will contain 53 sunday’s is

1 2 3 5
(a) (b) (c) (d)
7 7 7 7

18. A grouped data is shown below:


Class Interval Frequency Cumulative frequency


0–10 4 4
10–20 a 4+a
20–30 4 8+a
30–40 b 8+a+b
40–50 5 13 + a + b
Total 13 + a + b

106 Mathematics–X
If the median of the grouped data is 22.50 and the total frequency is 20, then what is
the value of a?
(a) 3
(b) 4
(c) 5
(d) 6

DIRECTION: In the question numbers 19 and 20, a statement of assertion (A) is followed by a
statement of Reason (R). Choose the correct option.
(a) Both A and R are true and R is the correct explanation for A.

(b) Both A and R are true and R is not the correct explanation for A.

(c) A is true but R is false.


(d) A is false but R is true.


19. Assertion (A) :


7 is an irrational number.
Reason (R) : A square root of a prime number is always an irrational number.

20. Assertion (A) : (1, 5) is the mid point of the line segment joining the points (5, 6) and (7, 4).

Reason (R) : Mid point of the line segment joining then points (x1, y1) and (x2, y2) is

x1 + x2 y1 + y2
d , n.
2 2

SECTION-B
Section B consists of 5 questions of 2 marks each.

21. Find the value(s) of k for which the pair of equations


kx + 2y = 3
*
3x + 6y = 10

has a unique solution.

22. The perimeters of two similar triangles are 30 cm and 20 cm respectively. If one side

of the first triangle is 9 cm long, find the length of the corresponding side of the second
triangle.
23. From a point Q, the length of the tangent to a circle is 24 cm and the distance of Q from

the centre is 25 cm. Find the radius of the circle.

24. In Fig., there is a sector of circle of radius 10.5 cm. Find the perimeter

22
of the sector. d Take r = n
7 A B
OR
An umbrella has 8 ribs which are equally spaced (Fig.). Assuming umbrella
60°
to be a flat circle of radius 45 cm, find the area between the two consecutive
O
ribs of the umbrella.

Sample Papers 107


1 – cos2 i
25. If tan i =
3
, find the value of e o.
4 1 + cos2 i

OR
3 1 1
If tan A = , find the value of + .
4 sin A cos A

SECTION-C
Section C consists of 6 questions of 3 marks each.
26. Find the largest number which on dividing 1251, 9377 and 15628 leaves remainders 1,

2 and 3 respectively.
2
27. If and –3 are the zeros of the polynomial ax2 + 7x + b, then find the values of a and
3

b.
28. Yash scored 40 marks in a test, getting 3 marks for each right answer and losing 1 mark

for each wrong answer. Had 4 marks been awarded for each correct answer and 2 marks
been deduced for each incorrect answer, then Yash would have scored 50 marks. How
many questions were there in the test?
OR
The area of a rectangle gets reduced by 9 square units, if its length is reduced by 5 units
and breadth is increased by 3 units. If we increase the length by 3 units and the breadth
by 2 units, the area increases by 67 square units. Find the dimensions of the rectangle.
1 1
29. If sec q = x + , prove that sec q + tan q = 2x or .
4x 2x

30. In Fig., PQ is a chord of length 16 cm, of a circle of radius 10 cm. The


tangents at P and Q intersect at a point T. Find the length of TP.


OR
In Fig., PQL and PRM are tangents to the circle with centre O at
the points Q and R, respectively and S is a point on the circle such
that ∠SQL = 50° and ∠SRM = 60°. Find ∠QSR.

31. A bag contains 15 white and some black balls. If the probability

of drawing a black ball from the bag is thrice that of drawing a


white ball, find the number of black balls in the bag.

SECTION-D
Section D consists of 4 questions of 5 marks each.
32. A takes 6 days less than B to do a work. If both A and B working together can do it in

4 days, how many days will B take to finish it?


OR
One-fourth of a herd of camels was seen in the forest. Twice the square root of the herd
had gone to mountains and the remaining 15 camels were seen on the bank of a river.
Find the total number of camels.

108 Mathematics–X
33. In Fig., ABCD is a trapezium with AB|| DC. If AED is similar to BEC, prove that AD = BC.
∆ ∆

34. A heap of rice is in the form of a cone of base diameter 24 m and height 3.5 m. Find the

volume of the rice heap. How much canvas cloth is required to just cover the heap?
OR
Water in a canal, 5·4 m wide and 1·8 m deep, is flowing with a speed of 25 km/hour.
How much area can it irrigate in 40 minutes, if 10 cm of standing water is required for
irrigation?
35. The mean of the following distribution is 18. Find the frequency f of the class 19 – 21.

Class 11–13 13–15 15–17 17–19 19–21 21–23 23–25


Frequency 3 6 9 13 f 5 4

SECTION-E
Case study based questions are compulsory.
36. Your elder brother wants to buy a car and plans to take loan from a bank for his car.

He repays his total loan of ` 1,18,000 by paying every month starting with the first
instalment of ` 1000. If he increases the instalment by ` 100 every month.

On the basis of above information, answer the following questions.


(i) What is the amount paid by him in 20th instalment?

(ii) What is the amount paid by him in 30th instalments?


(iii) What is the amount paid by him upto 20 instalments?


OR
What is the amount paid by him upto 30 instalments?

Sample Papers 109


37. A coach is discussing the strategy of the game with his players. The position of players is

marked with ‘×’ in the figure.


Y
B
A
D

X’ O E F
H X
G

Scale : 1 Square = 1 unit


X’

Based on the above information answer the following questions.


(i) What is the distance between the player C and B?
(ii) Write the position of the player who is 6 units from x-axis and 2 units to the right of

y-axis.
(iii) If (x, y) are the coordinates of the mid-point of the line segment joining A and H, then

write the value of x and y.


OR
According to sudden requirement coach of the team decided to increase one player
in the 4th quadrant without increasing the total number of players, so he decided
to change the position of player F in such a way that F becomes symmetric to D wrt
x-axis, then write the new position of F?
38. A student of class X draw a triangle inside a square of side length 9 cm as shown in figure

given below.

9 cm

9 cm

On the basis of above information, answer the following questions.


(i) When q = 30° then find the length of AE.

(ii) When q = 60° then find the length of AE.


(iii) When q = 60°, then find area of DAED.


OR
Find the area of shaded region when q = 60°.

zzz

110 Mathematics–X

SECTION-A
1. (a) The required largest number is the HCF of (70 – 5) and (125 – 8).

i.e., HCF of 65 and 117 which is 13.


∴ Required largest number is 13.
2. (b) Let given polynomial be p(x)= x2 + 3x + k and one of the zeros of p(x) is 2.

∴ p(2) = 0 ⇒ (2)2 + 3 × 2 + k = 0
⇒ 4+6+k=0 ⇒ k = – 10
3. (b) Given equations,

p+q=5 ...(i)
p–q=2 ...(ii)
On adding (i) and (ii), we get
7
⇒ 2p = 7 ⇒ p= = 3.5
2

Putting the value of p = 3.5 in equation (i), we have


⇒ 3.5 + q = 5 ⇒ q = 5 – 3.5 ⇒
q = 1.5
\ p = 3.5 and q = 1.5
4. (b) Let the original price of toy be `x.
360

Number of toys can buy in `360 = x


Reduced price = `(x – 2)
360
Now number of toys be can purchased in `360 =
( x – 2)

According to equation,
x – x+2
360 e o= 2
360 360
⇒ x – (x – 2) = 2 ⇒
x ( x – 2)

⇒ 720 = 2 [x2 – 2x] ⇒ x2 – 2x – 360 = 0


⇒ x2 – 20x + 18x – 360 = 0 (By mid term splitting)
⇒ x(x – 20) + 18(x – 20) = 0 ⇒ (x – 20)(x + 18) = 0
⇒ x = 20 (As price cant' be negative)
AB BC CA
5. (a)  = =
QR PR PQ

DPQR ∼ DCAB (By SSS similarity)


6. (b)  DABC ∼ DDFE
(Given)
AB BC AC
∴ ∠A = ∠D, ∠B = ∠F, ∠C = ∠E and = =
DF FE DE

Now ∠F = 180° – 30° – 50° = 100°


and
AB
DF
=
AC
DE
& 75.5 = DE 8
⇒ DE =
7. 5 × 8
5
⇒ DE = 12 cm

Sample Papers 111


7. (b) Mid points of line joining the points (–5, 7) and (–1, 3) is given by

–5 –1 7 + 3
d n=< ,
–6 10
, F = ^ –3, 5h
2 2 2 2

1

2 tan 30° 3 2 3
8. (c) = = × = 3 = tan 60°
2 2 2

1 – tan 30° 1 3
o 1– e
3
9. (a) Given tan x + sin x = m and tan x – sin x = n

Now m2 – n2 = (tan x + sin x)2 – (tan x – sin x)2


= tan2x + sin2x + 2 sin x tan x – tan2x – sin2x + 2 sin x tan x = 4 sin x tan x
= 4 sin2 x tan2 x
sin2 x sin2 x (1 – cos2 x)
=4 sin2 x × =4
cos2 x cos2 x

sin2 x – sin2 x cos2 x


=4 = 4 tan2 x – sin2 x
cos2 x

= 4 (tan x – sin x) (tan x + sin x) = 4 mn


1
10. (d) ∠OPA = × 60° = 30°
2

In right DOAP 3 cm
OA
tan 30° =
AP

1 3
= ⇒ AP = 3 3
AP 3 cm

3
11. (a) Given,

Area of circle = 154 cm2


154 154
rr 2 = 154 & r 2 = = × 7 = 49
r 22

⇒ r2 = 49 ⇒ r = 7 cm

` 22

Its perimeter = 2pr = 2 × × 7 = 44 cm


7
12. (a) We have,
3 tan q = 3 sin q
sin i
⇒ 3 = 3 sin q
cos i

3 3 1
⇒ = 3⇒ = cos q ⇒ = cos i
cos i 3

3
1
∴ cos q =

3
Now, sin2 q – cos2 q = 1 – cos2 q – cos2 q = 1 – 2 cos2 q
2
1
=1 – 2 × e o =1– =
2 1
3 3 3

112 Mathematics–X
22
13. (a) Length of wire = circumference of circle = 2 r = 2 × × 35 = 220 cm
7
π

So, the perimeter of square is also 220 cm.


220
Length of one side = = 55 cm
4

Now, area of square = side × side


= 55 × 55 = 3025 cm2
14. (b) In DLMN and DPQR

∠L = ∠P, ∠N = ∠R (Given)
third angle ∠M = ∠Q
` DLMN + DPQR (By AAA similarty)
But MN = 2 QR
` DLMN @ DPQR, i.e DLMN is not congruent to DPQR.
3
15. (c) Given volume of sphere = 12p cm

4 3
rr = 12r & r 3 = 9 = 3 2 & r = 3 2/3 cm
3

16. (c)

Class 65–85 85–105 105–125 125–145 145–165 165–185 185–205


Frequency 4 5 13 20 14 7 4
Cumulative 4 9 22 42 56 63 67
Frequency

Median class = 125 – 145 c = 33.5 m , Modal class = 125 – 145


n 67
=
2 2

Difference = 145 – 125 = 20


17. (a) A non leap year has one day left after 52 weeks.

1
∴ P(53 sundays) =
7

18. (c) ∴ Total frequency = 20


⇒ 13 + a + b = 20
   ⇒ a+b=7
 Median = 22.5
∴ Median class = 20 – 30.
So, l = 2, n = 13 + a + b = 20, cf = 4 + a, f = 4 and h = 10.
JK n N
KK – cf OOO
By formula, median = l + KK 2 O× h
K f OO

L P
JK 20 NO
OO ×10 = 20 + c m ×10
K – (4 + a) OO 10 – 4 – a
⇒ 22.5 = 20 + KK 2
KK 4

4 O
L P
⇒ 2.5 × 4 = 60 – 10a

⇒ 10 – 60 = – 10a
⇒ – 50 = – 10a
50
⇒ a = =5
10
   

Sample Papers 113


19. (a) As we know that square root of every prime number is always an irrational number.

Therefore, 7 is an irrational as 7 is a prime.


5+7 6+4
20. (d) We have mid point of line segment joining the points (5, 6) and (7, 4) be d n

,
2 2

i.e. (6, 5).

SECTION-B
a1 b1 k 2 ⇒k≠1
21. For unique solution a ! ⇒ !
b 3 6

2 2


The pair of equations have unique solution for all real values of k except 1.
22. Let the side of other triangle be x cm.

 Ratio of perimeters of two similar triangles is equal to ratio of their corresponding sides.
9 30

x = 20    ⇒    x = 6 cm

23. Let QT be the tangent and OT be the radius of circle. Therefore


OT ⊥ QT i.e., ∠OTQ = 90°


and OQ = 25 cm and QT = 24 cm
Now, by Pythagoras Theorem, we have
OQ2 = QT2 + OT2

⇒ 252 = 242 + OT2


⇒ OT2 = 252 – 242 = 625 – 576
OT2 = 49 \ OT = 7 cm
24. Given, radius of the sector of the circle, r = 10.5 cm.

i
∴ Perimeter of the sector = ×2rr + 2r
360°

60° 2×22
= × ×10.5 + 2×10.5
360° 7

1
= ×44×1.5 + 21
6

= 11 + 21
= 32 cm
∴ Perimeter of the sector = 32 cm

OR
We have, r = 45 cm
1
Area between two consecutive ribs = # rr 2
8
\

1 22 11 # 45 # 45
= # # 45 # 45 =
8 7 4#7

22275
= = 795.54 cm2
28

114 Mathematics–X
3
25. Given, tan i =
4

Since sec i = 1 + tan2 i


3 2
1 +c m =
9 16 + 9
⇒ sec i = 1+ =
4 16 16

5 4
⇒ sec i = ⇒ cos i =
4 5
   

4 2
1– c m
16
2 1–
1 – cos i 5 25 25 – 16 9
∴ = = = =
1 + cos2 i 2 16 25 + 16 41
1 +c m

4 1+
5 25
OR
3 3k
tan A = =
4 4k

3k 3 4k 4
sin A = = , cos A = =
5k 5 5k 5

1 1 5 5
+ = +
sin A cos A 3 4

20 + 15 35
= =
12 12

SECTION-C
26. 1251 – 1 = 1250, 9377 – 2 = 9375, 15628 – 3 = 15625

Required largest number = HCF (1250, 9375, 15625)


1250 = 2 × 54
9375 = 3 × 55
15625 = 56

\ HCF (1250, 9375, 15625) = 54 = 625

27. ax2 + 7x + b

–b –7
Sum of zeros = a = a
2 7
But sum of given zeros = –3=–
3 3

–7 –7
Now,

a = 3
∴ a=3
c b
Product of zeros = a = a from given polynomial
2 b
But product of given zeros = × (–3) = – 2. Therefore, – 2 = ⇒ b=–6
3 3
   

Sample Papers 115


28. Let x be the number of questions of right answer and y be the number of questions of wrong

answer.

\ According to question,
3x – y = 40 … (i)
and 4x – 2y = 50
or 2x – y = 25 …(ii)
Subtracting (ii) from (i), we have
3x – y = 40
2x – y = 25
– + –
x = 15
Putting the value of x in equation (i), we have
3 × 15 – y = 40 ⇒ 45 – y = 40

\ y = 45 – 40 = 5
Hence, total number of questions is x + y i.e., 5 + 15 = 20.
OR
Let the length and breadth of a rectangle be x and y respectively.
Then area of the rectangle = xy
According to question, we have
(x – 5) (y + 3) = xy – 9 ⇒
xy + 3x – 5y – 15 = xy – 9
⇒ 3x – 5y = 15 – 9 = 6 ⇒
3x – 5y = 6 …(i)
Again, we have
(x + 3) (y + 2) = xy + 67 ⇒
xy + 2x + 3y + 6 = xy + 67
⇒ 2x + 3y = 67 – 6 = 61 ⇒
2x + 3y = 61 …(ii)
Now, from equation (i), we express the value of x in terms of y as
6 + 5y
x=
3

Substituting the value of x in equation (ii), we have


6 + 5y 12 +10y + 9y
2#d n + 3y = 61 ⇒ = 61
3 3

171
⇒ 19y = 183 – 12 = 171 ⇒ y= =9
19

Putting the value of y in equation (i), we have


3x – 5 × 9 = 6 ⇒ 3x = 6 + 45 = 51

51
x= =17
3

\

Hence, the length of rectangle = 17 units and breadth of rectangle = 9 units.

116 Mathematics–X
29. Let sec q + tan q = l
...(i)
2 2
We know that, sec q – tan q = 1
(sec q + tan q) (sec q – tan q) = 1 ⇒ l(sec q – tan q) = 1
1
sec q – tan q = ...(ii)
m

Adding equations (i) and (ii), we get

2c x + m= m+
1 1 1
2 sec q = l + ⇒
m 4x m

1 1
⇒ 2x + =l+
2x m

1
On comparing, we get l = 2x or l =
2x

1
⇒ sec q + tan q = 2x or
2x

Alternative Method:
1
We have sec i = x +
4x

tan2 i = sec2 i – 1
1 1
= x2 + + –1
2 2

16x
1 1 1 2
= x2 +
– = c x – m
16x2 2 4x

⇒ tan i = ! c x – m
1
4x

sec q + tan is given by


θ

1 1 1 1
x+ +x– or x + – x+
4x 4x 4x 4x

1
= 2x or
2x

30. Given: PQ = 16 cm and PO = 10 cm


To find: TP
16
PR = RQ = = 8 cm
2

(Perpendicular from the centre bisects the chord)


In DOPR,
OR = OP2 – PR2 = 102 – 82 = 100 – 64
= 36 = 6 cm
Let ∠POR be q.
PR 8 4
In DPOR,tan q = = =
RO 6 3

Sample Papers 117


We know, OP ⊥ TP (Point of contact of a tangent is perpendicular to the line from the
centre)
OP 4 10
In DOTP, tan q = ⇒ =
TP 3 TP

10 × 3 15
TP = = = 7.5 cm.
4 2

OR
In the given figure PQL and PRM are the tangents to the circle of point Q and R
respectively.
∠OQL = 90° and ∠ORM = 90° (Since tangent is perpendicular to the


radius of the point of tangency)


It is given that ∠SQL = 50° and ∠SRM = 60°
∴ ∠ORS = ∠OQL – ∠SQL = 90° – 50° = 40°
and ∠ORS = ∠ORM – ∠SRM = 90° – 60° = 30° 41
°
In DORS, we have
OR = OR (Radii)

30°
∠ORS = ∠OSR = 30°
Also, in DOQS, we have

OQ = OS (Radii)
∠OQS =∠OSQ = 40°
∴ ∠QSR = ∠OSR + OSQ = 30° + 40° = 70°
∠QSR = 70°
31. Let there be x black balls and 15 white balls.

Total balls = n(S) = 15 + x


P (drawing black ball) = 3 × P (drawing white ball)
x 15
⇒ = 3×
(15 + x) (15 + x)

3×15
⇒ x= ×(15 + x)
(15 + x)

⇒ x = 45
` There are 45 black balls in the bag.

SECTION-D
32. Let B complete a work in x days.

Then A takes x – 6 days to complete it.


Together they complete it in 4 days.
According to work done per day,
1 1 1 x+x–6 1
+ = ⇒ =
x–6 x 4 x (x – 6) 4

4(2x – 6) = x(x – 6)

118 Mathematics–X
   ⇒ 8x – 24 = x2 – 6x
2
` x – 14x + 24 = 0
⇒ x2 – 12x – 2x + 24 = 0
⇒ x(x – 12) – 2(x – 12) = 0
  ` x = 12
As x = 2 is not possible because then x – 6 is (– 4).
` x = 12
So, B takes 12 days to finish the work.
OR
Let x be the total number of camels.
x
Then, number of camels in the forest =
4

number of camels on mountains = 2 x


and number of camels on the bank of river = 15
x
Thus, total number of camels = + 2 x + 15
4

Now by hypothesis, we have


x
+ 2 x + 15 = x ⇒ 3x – 8 x – 60 = 0
4

Let x = y , then x = y2
⇒ 3y2 – 8y – 60 = 0 ⇒
3y2 – 18y + 10y – 60 = 0
⇒ 3y(y – 6) + 10(y – 6) = 0 ⇒ (3y + 10)(y – 6) = 0
10
⇒ y=6 or y=–
3

10 10 2 100
Now, y =– ⇒ x = c– m = ( x = y2)

3 9
But, the number of camels cannot be a fraction.

\ y=6
⇒ x = 62 = 36
Hence, the number of camels = 36
33. In EDC and EBA we have
∆ ∆

∠1 = ∠2 (Alternate angles)

∠3 = ∠4 (Alternate angles)

and ∠CED = ∠AEB (Vertically opposite angles)



\ ∆ EDC ~ EBA ∆ (By AA criterion of similarity)

ED EC ED EB

EB
=
EA
& EC
=
EA
…(i)

It is given that AED ~ BEC.


∆ ∆

ED EA AD
= = ...(ii)
EC EB BC

\

From (i) and (ii), we get

Sample Papers 119


EB EA
= ⇒ (EB)2 = (EA)2 ⇒ EB = EA
EA EB

Substituting EB = EA in (ii), we get


EA AD AD
= ⇒ ⇒ AD = BC
EA BC BC

34. Radius of conical heap = 12 m


1 1 22
Volume of rice = πr 2 h = × × 12 × 12 × 3.5 m3 = 528 m3
3 3 7

Area of canvas cloth required = prl

l= h2 + r 2 = 122 + (3.5)2 = 12.5 m


22
Area of canvas required = × 12 × 12.5 = 471.4 m2
7

OR
40 2
Speed of water in canal = 25 km/hr. in 40 min = = hr.
60 3

2 50 50000
length of water = 25 × = cm = m
3 3 3

Volume of water in canal in 40 minutes = Volume of water for irrigation


54 18 50000 3 10
⇒ × × m = × l × b m3
10 10 3 100

  324 × 5000 = l × b
  1620000 = l × b
area irrigated in 40 minutes = 1620000 m2
1620000
= = 1.62 km2 or 162 hectares.
1000000

35. Here, we take assumed mean a = 18 and class size (h) = 2


xi – a xi – 18
ui = =
h 2

Now, we obtain the data as in table given below:


Daily pocket Number of Mid-point (xi) xi − 18 fiui
allowance (in ™) children (fi) ui =
2
11–13 3 12 –3 –9
13–15 6 14 –2 – 12
15–17 9 16 –1 –9
17–19 13 18 0 0
19–21 k 20 1 k
21–23 5 22 2 10
23–25 4 24 3 12
Sfi = 40 + k Sfiui = k – 8

120 Mathematics–X
Rfi ui
Mean = ] X g = a + h f p
Rfi

k–8
18 = 18 + 2 c m ⇒ k=8
40 + k

SECTION-E
36. (i) Since first installment is of `1000 and he increases the installment by `100 every

month.
Thus AP is formed
1000, 1100, 1200, ...
` a = 1000, d = 1100 – 1000 = 100
` Amount paid by him in 20th installment, a20 = a + 19d

= 1000 + 19 × 100 = `2900.


(ii) We have AP 1000, 1100, 1200, ...

` a = 1000, d = 1100 – 1000 = 100


` Amount paid by him in 30th installment, a30 = a + 29d
= 1000 + 29 × 100 = `3900.
(iii) We have AP 1000, 1100, 1200, ...

` a = 1000, d = 100
Amount paid upto 20th installments
`
n
Sn = (2a + (n – 1) d)
2

20
⇒ S20 = (2 ×1000 + 1900) = 10 × 3900 = `39000
2

OR
Amount paid upto 30 installments
= ^2a + ]n – 1g dh
n
2

30
= ^2 ×1000 + 29 ×100h
2

30
= × 4900 = 15 × 4900 = ` 73500
2

37. (i) Distance between C(–3, 2) and B(1, 6) is


^1 + 3 h2 + (6 – 2) 2 = 16 + 16 = 4 2 units.
(ii) Coordinates of the required player are (2, –6), or (2, 6). J is at (2, – 6).

(iii) Let (x, y) is the mid point of A(– 4, 5) and H(– 4, –1).

–4–4 5 –1
So, x = , y= ⇒ x = – 4, y = 2
2 2

Sample Papers 121


OR
If player F is shifted to IV quadrant symmetric to D wrt x-axis, coordinates of F are
(3, – 4).
38.
(i) Since ABCD is a square

` +A = 90°
DA
` tan i = (In right DDAE)
AE

9 1 9
⇒ tan 30° =
AE
& =
AE
& AE = 9 3 cm.
3

(ii) In right DDAE, we have


DA 9 9
tan i =
AE
& tan 60° =
AE
& 3=
AE
9 3
& AE = ×
3 3

9 3
& AE =
3
= 3 3 cm

1 1 27 3
(iii) Area of DAED = × AE × DA = × 3 3 × 9 = cm 2
2 2 2

OR
Area of shaded region = Area of square ABCD – Area of DAED

27 3
= (9) 2 –
2

27 3
= d 81 – n cm.
2

zzz

122 Mathematics–X
SAMPLE PAPER
5 [SOLVED]
Time allowed: 3 hours Maximum marks: 80
General Instructions: Same as CBSE Sample Question Paper–2023 (Solved).

SECTION-A
Section A consists of 20 questions of 1 mark each.

1. If p and q are prime numbers than the HCF of p3q2 and p2q is

(a) p3q2

(b) p2q
(c) p2q2
(d) pq

2. Which of these is a zero of the polynomials p (y) = 3y 3 – 16y – 8 ?


(a) 2
(b) 8
(c) –2
(d) –8

3. The value of k for which the system of equations x + 2y – 3 = 0 and 5x + ky + 7 = 0


has no solution, is
(a) 5
(b) 10
(c) 7
(d) None of these

4. The quadratic equation 2x2 – 5 x + 1 = 0 has


(a) two distinct real roots


(b) two equal real roots

(c) no real root


(d) more than two real roots

5. The points A(0, 6), B(–5, 3) and C(3, 1) are the vertices of a triangle which is

(a) isosceles
(b) equilateral

(c) scalene
(d) none of these

6. Observe the right triangle ABC, right angled at B as shown below.


A
x
P

5 x+
5

B C
What is the length of PC?
(a) 2.5 cm
(b) 4.5 cm
(c) 6 cm
(d) 7.5 cm

Sample Papers 123


7. In DABC, DE  BC in given figure, then BD is equal to

4 cm 6 cm

D E
3 cm

B C

(a) 2 cm
(b) 8 cm
(c) 3 cm
(d) 9 cm

8. If 3 sec i – 5 = 0 then cot i is equal to


5 4 3 3
(a) (b) (c) (d)
3 5 4 5

2 tan x (sec2 x – 1)
9. Which of these is equivalent to ?
cos 3 x

(a) 2 tan3 x cosec x


(b) 2 cot3 x cosec3 x

3 3
(c) 2 tan x sec x
(d) 2 cot3 x sec3 x

10. From an external point Q, the length of the tangent to a circle is 5 cm and the distance

of Q from the centre is 8 cm. The radius of the circle is


(a) 39 cm
(b) 3 cm
(c) 39 cm (d) 7 cm

11. The difference between the circumference and radius of a circle is 37 cm. The area of

circle is
(a) 111 cm2
(b) 184 cm2
(c) 154 cm2
(d) 259 cm2

12. Volumes of two spheres are in the ratio 64 : 27. The ratio of their surface areas is

(a) 3 : 4
(b) 4 : 3
(c) 9 : 16
(d) 16 : 9

13. If DABC ~ DDEF then which of the following is not true?


(a) BC.DF = AC.EF


(b) AB.DF = AC.DE

(c) BC.DE = AB.EF


(d) BC.DE = AB.FD

sin i
14. is equal to
1 + cos i

1 + cos i 1 + cos i 1– cos i 1– sin i


(a) (b) (c) (d)
sin i cos i sin i cos i

15. The arithmetic mean of 1, 2, 3 ...... n is


n+1 n n –1 n
(a) (b) (c) (d) +1
2 2 2 2

16. A square ABCD is inscribed in a circle of radius ‘r’. The area of


A B
the square in square units is
(a) 3r2
(b) 2r2

O
2
(c) 4r
(d) none of them

D C

124 Mathematics–X
17. The time in seconds, taken by 150 athletes to run a 110 m hurdle race are tabulated

below:
Class 13.8 – 14 14 – 14.2 14.2 – 14.4 14.4 – 14.6 14.6 – 14.8 14.8 – 15

Frequency 2 4 5 71 48 20

The number of atheletes who completed the race in less then 14.6 seconds is
(a) 11 (b) 71 (c) 82
(d) 130

18. If a pair of dice is thrown, then the probability of getting a doublet is


1 1 5 2
(a) (b) (c) (d)
3 6 12 3

DIRECTION: In the question numbers 19 and 20, a statement of assertion (A) is followed by a
statement of Reason (R). Choose the correct option.
(a) Both A and R are true and R is the correct explanation for A.

(b) Both A and R are true and R is not the correct explanation for A.

(c) A is true but R is false.


(d) A is false but R is true.


19. Assertion (A) : A number N when divided by 21 gives the same remainder, when N is

divided by 7.
Reason (R) : p is an irrational number.

20. Assertion (A) :


The distance between the line segment AB whose co-ordinates of end
points are A(3, 2) and B(4, 5) is 10 unit.
Reason (R) : Distance between the line segment joining the points A(x1, y1) and

B(x2, y2) is AB = (x2 – x1) 2 + (y2 – y1) 2 .

SECTION-B
Section B consists of 5 questions of 2 marks each.
21. Solve the following pair of linear equations.

3x + 4y = 10 and 2x – 2y = 2

22. E and F are points on the sides PQ and PR respectively of a PQR. Show that EF ||QR

if PQ = 1.28 cm, PR= 2.56 cm, PE = 0.18 cm and PF = 0.36 cm.


23. PB is a tangent to the circle with centre O to B. AB is a chord of length

O
24 cm at a distance of 5 cm from the centre, if the tangent is of length
20 cm, find the length of PO.
M
24. Prove that (sin a + cos a)(tan a + cot a) = sec a + cosec a.

OR
Show that tan4 q + tan2 q = sec4 q – sec2 q.
25. The length of the minute hand of a clock is 14 cm. Find the area swept by the minute

hand in 5 minutes.

Sample Papers 125


OR
The wheels of a car are of diameter 80 cm each. How many complete revolutions does
each wheel make in 10 minutes when the car is travelling at a speed of 66 km per hour?

SECTION-C
Section C consists of 6 questions of 3 marks each.
a
26. Express the number 0.3178 in the form of rational number .
b

27. Find the quadratic polynomial sum and product of whose zeros are –1 and –20

respectively. Also find the zeros of the polynominal so obtained.


28. Five years ago, Nuri was thrice as old as Sonu. Ten years later, Nuri will be twice as old

as Sonu. How old are Nuri and Sonu?


OR
9
A fraction becomes , if 2 is added to both the numerator and the denominator. If 3
11

5
is added to both the numerator and the denominator, it becomes . Find the fraction.
6
29. In Fig., from an external point P, two tangents PT and PS are

T

drawn to a circle with centre O and radius r. If OP = 2r, show that Q


O P
∠OTS = ∠OST = 30°.
OR S

Prove that, a tangent to a circle is perpendicular to the radius


through the point of contact.
1
30. If 1 + sin2 q = 3 sin q cos q, prove that tan q = 1 or .
2

31. A number x is selected at random from the numbers 1, 2, 3 and 4. Another number y is

selected at random from the numbers 1, 4, 9 and 16. Find the probability that product
of x and y is less than 16.

Section-D
Section D consists of 4 questions of 5 marks each.
32. A plane left 30 minutes late than its scheduled time and in order to reach the destination

1500 km away in time it had to increase its speed by 100 km/h from the speed. Find its
usual speed.
OR
If the roots of the quadratic equation (a – b)x2 + (b – c)x + (c – a) = 0 are equal, prove that
2a = b + c.
33. AD and PM are median of triangles ABC and PQR respectively where ∆ABC ∼ ∆PQR.
∆ ∆

AB AD
Prove that = .
PQ PM

126 Mathematics–X
34. A solid iron pole consists of a cylinder of height 220 cm and base diameter 24 cm, which

is surmounted by another cylinder of height 60 cm and radius 8 cm. Find the mass of
the pole, given that 1 cm3 of iron has approximately 8 g mass. (Use p = 3.14).
OR
A container shaped like a right circular cylinder having diameter 12 cm and height
15 cm is full of ice cream. The ice cream is to be filled into cones of height 12 cm and
diameter 6 cm, having a hemispherical shape on the top. Find the number of such cones
which can be filled with ice cream.
35. If the median of the distribution given below is 28.5, find the values of x and y.

Class interval 0–10 10–20 20–30 30–40 40–50 50–60 Total


Frequency 5 x 20 15 y 5 60

SECTION-E
Case study based questions are compulsory.
36. SUN ROOM

The diagrams show the plans for a sun room. It will be built onto the wall of a house.
The four walls of the sun room are square clear glass panels. The roof is made using
• Four clear glass panels, trapezium in shape, all the same size
• One tinted glass panel, half a regular octagon in shape
Y

A B C D

J I H

B E
P F
A
Top view
B
A 1 cm
P Q R S

Not to scale

Front view
O Scale 1 cm = 1m X

Based on above information answer the following questions.


(i) Find the mid point of the line segment joining the point J(6, 17) and I(9, 16).

(ii) Find the distance of the point P from the y-axis.


(iii) Write the distance between the point A and S.


OR
Find the coordinates of the point which divides the line segment joining school and
park internally in the ratio 3 : 2.

Sample Papers 127


37. Hardik repays his total loan of `1,32,000, by paying every month. Starting with the first

installment of `1,500. He increases the installment by `200 every month.

Based on above information answer the following questions.


(i) If the amount paid in successive installment form an AP then write its first term and

common difference.
(ii) Find the amount paid by Hardik in 15th installment.

(iii) In which installment will he pay `5500?


OR
Write the value of k for which the terms 11, 2k + 1, 3k – 1 form an AP.
38. Kapil is standing in the balcony of his house. He observes two cars, one on either side

(say A and B).


Kapil

A B
Based on above information answer the following questions.
(i) At an instant, the angle of depression of the two cars were found to be 30° and 60°

respectively. If the height at which he is standing is 3 m, then find the distance


between the two cars.
(ii) As the first car approaches towards the building. State whether the angle of

depression will increase or decrease.


(iii) If the ratio of the height of a tower and the length of its shadow is
3 : 1 , then find
the angle of elevation of the sun.
OR
If the height of a building and the distance of the point of observation from its foot,
both are increased by 10%, then state whether the angle of elevation increased by
10% or remains unchanged.

zzz

128 Mathematics–X
SECTION-A
1. (b) We have,

p3q2 = p × p × p × q × q
p2q = p × p × q
∴ HCF = p × p × q = p2q
2. (c) We have,
p(y) = 3y3 – 16y – 8

\ p(–2)= 3 × (–2)3 – 16 × (–2) – 8


= –24 + 32 – 8 = –32 + 32 = 0
⇒ p(–2) = 0

\ y = –2 is the zero of p(y).


i.e., – 2 is the zero of p(y).


3. (b) For no solution; we have

a1 b1 c1 1 2 –3
a2 = b2 ! c2 ⇒ = ! ⇒ k = 10
5 k 7

4. (c) Given quadratic equation 2x2 – 5 x + 1 = 0


Its discriminant D = b2 – 4ac


= _– 5 i – 4 × 2 × 1
2

=5–8=–3<0

⇒ D<0

\ It has no real roots.
5. (a) We have,
AB = (–5 – 0) 2 + (3 – 6) 2 = 25 + 9 = 34
AC = (3 – 0) 2 + (1 – 6) 2 = 9 + 25 = 34
BC = (3 + 5) 2 + (1 – 3) 2 = 64 + 4 = 68
Here, AB = AC ⇒
ABC is an isosceles triangle.
    ∆

6. (d) We have

In DAPB and DABC


+APB = +ABC = 90°
+BAP = +BAC (Common)

` DAPB ~ DABC (By AA similarity criteria)


AB AP
`
AC
=
AB
& AB2 = AC $ AP & 25 = (2x + 5) . x
& 25 = 2x2 + 5x & 2x2 + 5x – 25 = 0 & 2x2 + 10x – 5x – 25 = 0
& 2x (x + 5) – 5 (x + 5) = 0 & (x + 5 ) (2x – 5) = 0
& 2x – 5 = 0 (a x + 5 ! 0 & x ! – 5 and length cannot be negative)

5
` x = = 2.5
2

Length of PC = x + 5 = 2.5 + 5 = 7.5 cm

Sample Papers 129


7. (a) In DABC

A
DE|| BC
AD AE 4 cm 6 cm
Then = [By BPT]
BD EC

4 6 D E
= ⇒ 4 × 3 = BD × 6
BD 3 3 cm
   

4×3
⇒ BD = = 2 cm B C
6
      

5
8. (c)
3 sec i – 5 = 0 & sec i =
3
Hypotenuse A
But sec i =
base
& h = 5 and b = 3
By Pythagoras theorem,
5

h 2 = p 2 + b 2 & ( 5) 2 = p 2 + ( 3) 2
⇒ p2 = 25 – 9 = 16 & p = 4
base b 3 B
3
C
So cot i = = =
perpendicular p 4

9. (c) We have

2 tan x (sec2 x – 1) 2 tan x × tan2 x


= = 2 tan3 x sec3 x
cos3 x cos3 x

10. (c) Let O be the centre of the circle and PQ be the tangent at P

to the circle.
O
∴∠OPQ = 90°
8 cm

Now, in right DOPQ, we have


OQ2= OP2 + OP2 Q 5 cm P
2 2 2 2
  ⇒ (8) = (5) + OP ⇒   OP = 64 – 25 = 39
    ⇒ OP = 39    ⇒ radius of the circle = 39 cm

11. (c) According to question


2rr – r = 37
22 44r – 7r
& 2 × 7 ×r – r = 37 & 7 = 37
37r
& 7 = 37 & r = 7 cm
22
Now area of circle = rr2 = ×(7) 2 = 154 cm2
7

12. (d) Let r1 and r2 be the radii of two spheres. 4 3


rr
3 1
Therefore, Ratio of volume of two spheres =
4 3

rr 2
3 3 3
r 3 r
=f 1 p c m =f 1 p
64 4

27 r2 & 3 r2

130 Mathematics–X
r1 4
⇒ ...(i)
r2 = 3

4rr12
Now, ratio of the surface area of two spheres =
4rr22

2
r1 4 2 16
= fr p =c m =
3 9

∴ Ratio be 16 : 9.
13. (d)  DABC ∼ DDEF

` Ratio of corresponding sides are equal.


AB BC AC
So,

DE
=
EF
=
DF
& BC.DF = AC.EF is true

or AB.DF = AC.DE is true or BC.DE = AB.EF is true


But BC.DE = AB.FD is not correct.

sin i sin i 1 – cos i sin i (1 – cos i)


14. (c) = × =
1 + cos i 1 + cos i 1 – cos i 1 – cos2 i

sin i (1 – cos i) 1 – cos i


= 2
=
sin i

sin i
15. (a) Given data 1, 2, 3, ......, n

1 + 2 + 3 + ..... + n n (n + 1 ) n+1
Arithmetic mean = n = = d n
2n 2

16. (b)
a ∠O = 90°
` AD = AO2 + DO2= r2 + r2 = r 2 square units

So, area of square = side × side = r 2 ×r 2 = 2r2


17. (c) From the given frequency distribution table, we have

Total number of athletes who completed the race in less than 14.6 seconds
= 2 + 4 + 5 + 71 = 82
18. (b) When a pair of dice is thrown then

total number of possible outcomes = 36


⇒ n(S) = 36
∴ Doublets are = {(1, 1), (2, 2), (3, 3), (4, 4), (5, 5), (6, 6)}
n(E) = 6
6 1

∴ Probability of getting a doublet = =


36 6

19. (b) Clearly, both A and R are correct but R does not explain A.

20. (a) We have,


AB = (x2 – x1) 2 + (y2 – y1) 2

  = (4 – 3) 2 + (5 – 2) 2 = 1 + 9 = 10 unit
Both A and R are correct and R is the correct explanation for A.

Sample Papers 131


SECTION-B
21. Given equations:

3x + 4y = 10 … (i)
2x – 2y = 2 … (ii)
Multiply (i) by 2 and (ii) by 3, we get
6x + 8y = 20
6x – 6y = 6
– + –
  

14y = 14 ⇒ y=1

∴ 2x – 2(1) = 2 ⇒ 2x = 4
⇒ x=2
∴ x = 2, y = 1
22. We have, PQ = 1.28 cm, PR = 2.56 cm

PE = 0.18 cm, PF = 0.36 cm


  

Now, EQ = PQ – PE = 1.28 – 0.18 = 1.10 cm


and FR = PR – PF = 2.56 – 0.36 = 2.20 cm


PE 0.18 18 9
Now, = = =
EQ 1.10 110 55

PF 0.36 36 9 PE PF
and, = = = =

FR 2.20 220 55
  \

EQ FR
Therefore, EF  QR [By the converse of Basic Proportionality Theorem]
23. Join OB,

Now, in right angled ∆OMB, we have


OB2 = 52 + 122 = 169 ⇒    OB = 13 cm
O
∴ In right ∆OBP, we have 20
5
OP2 = OB2 + BP2 M
12

24
= 132 + 202 = 169 + 400
⇒ OP2 = 569
∴ OP = 569 = 23.66 cm
24. LHS = (sin + cos )(tan + cot )
α α α α

= ]sin a + cos agc cos a + m


sin a cos a
sin a

= ]sin a + cos agd


sin2 a + cos2 a
n
cos a sin a

= ]sin a + cos ag ×
1
cos a . sin a

sin a cos a
= +
cos a sin a cos a sin a

1 1
= cos a + = sec a + cosec a = RHS
sin a

132 Mathematics–X
OR
4 2

LHS = tan θ + tan θ

= tan2 (tan2 + 1)
θ θ

= (sec2 – 1) (sec2 ) = sec4 – sec2 = RHS


θ θ θ θ

25. Since the minute hand rotates through 6° in one minute, therefore, area swept by the

minute hand in one minute is the area of a sector of angle 6° in a circle of radius 14 cm.
i
Hence, the area swept in 5 minutes = # rr 2 # 5
360°

6° 22 1
= # # (14) 2 # 5 = # 22 # 28 # 5
360° 7 60

154 1
= cm2 = 51 cm2
3 3

OR
The diameter of a wheel = 80 cm
So, radius of the wheel = 40 cm
Now, distance travelled in one complete revolution of wheel = 2p × 40 = 80p
Since, speed of the car is 66 km/h
66 # 100000 # 10
So, distance travelled in 10 minutes = cm
60


= 11 × 100000 cm = 1100000 cm.

1100000 110000
So, number of complete revolutions in 10 minutes = =
80r 22

8#
7
110000 # 7 70000
= = = 4375
8 # 22 16

SECTION-C
26. Let
x = 0.3178
Then x = 0.3178178178... ... (i)
10x = 3.178178178... ... (ii)
10000x = 3178.178178... ... (iii)
On subtracting (ii) from (iii), we get
3175 635
9990x = 3175 ⇒ x= =
9990 1998

635
0.3178 =
1998
\

27. Let a and b be the zeros of the quadratic polynomial.


∴ Sum of zeros, a + b = – 1 and product of zeros, a . b = – 20


Now, quadratic polynomial be
x2 – (a + b) . x + ab
= x2 – (–1) x – 20 = x2 + x – 20

Sample Papers 133


Now, for zeros of this polynominal

x2 + x – 20 = 0 ⇒ x2 + 5x – 4x – 20 = 0
⇒ x (x + 5 ) – 4 (x + 5 ) = 0 ⇒ ( x + 5) ( x – 4) = 0
⇒ x = – 5, 4
∴ Zeros are – 5 and 4.
28. Let the present age of Nuri and Sonu be x and y years respectively.

According to question x – 5 = 3(y – 5)


⇒ x – 5 = 3y – 15

⇒ x – 3y = – 10
...(i)
and x + 10 = 2 (y + 10)

⇒ x + 10 = 2y + 20

⇒ x – 2y = 10
...(ii)
From equation (i), we get
x = 3y – 10
...(iii)
Putting it in (ii), we have,
3y – 10 – 2y = 10

⇒ y = 10 + 10 ⇒ y = 20

Now, from (iii)


x = 3 × 20 – 10
⇒ x = 60 – 10 ⇒ x = 50

Therefore, present age of Nuri is 50 years and of Sonu is 20 years.


OR
x
Let required fraction be y .
According to question
x+2 9
= ⇒ 11x + 22 = 9y + 18 ⇒ 11x – 9y = – 4
y+2 11
     

x+3 5
and = ⇒ 6x + 18 = 5y + 15 ⇒ 6x – 5y = – 3
y+3 6
       

Now, we have a pair of linear equations


11x –9y = – 4 ...(i)
6x – 5y = – 3 ...(ii)
Multiplying (i) by 5 and (ii) by 9, we get
55x – 45y = – 20 ...(iii)
54x – 45y = – 27 ...(iv)
Subtracting (iv) from (iii), we get
55x – 45y – (54x – 45y) = – 20 – (– 27)
⇒ 55x – 45y – 54x + 45y = – 20 + 27 ⇒ x = 7

Putting x = 7 in (i), we have


11 × 7 – 9y = – 4 ⇒ 9y = 77 + 4
⇒ 9y = 81 ⇒ y=9
7
Hence, required fraction is .
9

134 Mathematics–X
29. Given: PT and PS are two tangents drawn from P to circle C(O, r) and OP = 2r.

Let ∠TOP = q
T
OT r 1
cos q = = = ⇒ cos q = cos 60° ⇒ q = 60°
OP 2r 2

\

Q
Hence, ∠TOS = 120° O P

In DOTS, OT = OS (Radii of circle)


60° S
⇒ ∠OTS = ∠OST = = 30°
2

OR
Given: A circle C(O, r) and a tangent AB at a point P.
To Prove: OP ⊥ AB.
Construction: Take any point Q, other than P, on the tangent AB. Join OQ. Suppose OQ
meets the circle at R.
Proof: We know that among all line segments joining the point O to a point on AB, the
shortest one is perpendicular to AB. So, to prove that OP ⊥ AB it is sufficient to prove
that OP is shorter than any other segment joining O to any point of AB.
Clearly, OP = OR (Radii of the same circle)
Now, OQ = OR + RQ
⇒ OQ > OR
⇒ OQ > OP ( OP = OR)
Thus, OP is shorter than any other segment joining O to any
point on AB.
Hence, OP ⊥ AB.

30. Given, 1 + sin2 = 3 sin .cos


θ θ θ

2
Divide both sides by cos , we have
θ

1 sin2 i 3 sin i. cos i


2
+ 2
=
cos2 i

cos i cos i
⇒ sec2 i + tan2 i = 3 tan i
⇒ 1 + tan2 i + tan2 i = 3 tan i
⇒ 2 tan2 i – 3 tan i + 1 = 0
⇒ 2 tan2 i – 2 tan i – tan i + 1 = 0
2 tan i ]tan i – 1g –1 ]tan i –1g = 0


]tan i – 1g]2 tan i –1g = 0


⇒ tan θ –1=0 or 2 tan – 1 = 0
θ

1
⇒ tan =1 or 2 tan =1 ⇒ tan i =
2
θ θ

1
⇒ tan =1 or
2
θ

Sample Papers 135


31. x can be any one of 1, 2, 3 or 4 and y can be any one of 1, 4, 9 or 16

Total number of cases of product of x and y = 16


Product less than 16 = (1×1, 1×4, 1×9, 2×1, 2×4, 3×1, 3×4, 4×1)
Number of cases, where product is less than 16 = 8
8 1
Required probability = or
16 2

\

SECTION-D
32. Given, distance is 1500 km.

Usual speed = S.
distance
& time = distance

We know, speed =
time speed

1500 1 1500
From question, + = [half an hour late i.e., 30 min = 0.5 h.]
100 + S 2 S
  

1500 1500 1
= = –
100 + S S 2

1500 3000 – S
= =
100 + S 2S

Cross multiplying,
3000S = 300000 – 100S + 3000S – S2
  ⇒ S2 + 100 S – 300000 = 0
2
⇒ S + 600 S – 500 S – 300000 = 0
  ⇒ S(S + 600) – 500(S + 600) = 0
     ⇒ (S – 500)(S + 600) = 0
⇒ S – 500 = 0 or S + 600 = 0
⇒ S = 500 km/h     S =– 600 km/h
⇒ S = 500 or – 600 km/h.
But speed cannot be negative.
The usual speed of the plane is 500 km/h.
OR
2
Since the equation (a – b) x + (b – c) x + (c – a) = 0 has equal roots, therefore discriminant
D = (b – c)2 – 4(a – b) (c – a) = 0

= b2 + c2 – 2bc – 4(ac – a2 – bc + ab) = 0


= b2 + c2 – 2bc – 4ac + 4a2 + 4bc – 4ab = 0
= 4a2 + b2 + c2 – 4ab + 2bc – 4ac = 0
= (2a)2 + (– b)2 + (– c)2 + 2(2a) (– b) + 2(– b) (– c) + 2 (– c) 2a = 0


= (2a – b – c)2 = 0    ⇒    2a – b – c = 0
   ⇒   2a = b + c.
Hence proved
33. In ABD and PQM we have
∆ ∆

∠B = ∠Q ( ABC ~ PQR)
∆ ∆ …(i)

136 Mathematics–X
AB BC ( ABC ~ PQR)

= ∆ ∆

PQ QR

1
BC
AB 2
⇒ = (Since AD and PM are
PQ 1 QR


the medians of ABC


2

and PQR respectively)


AB BD
⇒ = …(ii)
PQ QM

From (i) and (ii), it is proved that ∆ABD ~ PQM


∆ (By SAS criterion of similarity)


AB BD AD AB AD
⇒ = = ⇒ =
PQ QM PM PQ PM

34. Let r1 and h1 be the radius and height of longer cylinder, respectively, and r2, h2 be the

respective radius and height of smaller cylinder mounted on the longer cylinder. Then
we have, r
r1 = 12 cm, h1 = 220 cm

r2 = 8 cm, h2 = 60 cm

Now, volume of solid iron pole


= volume of the longer cylinder + volume of smaller cylinder
= rr12 h1 + rr22 h2
= 3.14 × (12)2 × 220 + 3.14 × (8)2 × 60
= 3.14 × 144 × 220 + 3.14 × 64 × 60
= 99475.2 + 12057.6
= 111532.8 cm3
Hence, the mass of the pole = (111532.8 × 8) grams

111532.8 # 8
= kg
1000

= 892.2624 kg
OR
Volume of the cylinder = pR2H
22
× (6)2 × 15
=
7

22
= × 36 × 15 cm3
7

Now, volume of cone having hemispherical shape on top


1 2
= rr 2 h + rr 3
3 3

Sample Papers 137


1 2
=rr (h + 2r)
3

1 22
= × × (3)2 (12 + 2 × 3)
3 7

1 22 22
= # # 9 # 18 = # 54 cm3
3 7 7


\ The number of cones that can be filled with ice cream
Volume of cylinder
=
Volume of cone having hemispherical shape on top
22
# 36 # 15
7
= = 10
22

# 54
7
35. Here, median = 28.5 and n = 60

Now, we have
Class interval Frequency (fi) Cumulative frequency (cf)
0–10 5 5
10–20 x 5+x
20–30 20 25 + x
30–40 15 40 + x
40–50 y 40 + x + y
50–60 5 45 + x + y
Total Sfi = 60

Since the median is given to be 28.5, thus the median class is 20 – 30.
n
= 30, l = 20, h = 10, cf = 5 + x and f = 20
2

\

n
f p# h
– cf 30 – (5 + x)
28.5 = 20 + < F # 10
2
Median = l + ⇒
f 20

\

25 – x
⇒ 28.5 = 20 + # 10
20

25 – x
⇒ 28.5 = 20 + ⇒ 57 = 40 + 25 – x
2

⇒ 57 = 65 – x ⇒ x = 65 – 57 = 8
Also, n = Sfi = 60
⇒ 45 + x + y = 60
⇒ 45 + 8 + y = 60 ( x = 8)

\ y = 60 – 53 ⇒ y=7
Hence, x = 8 and y = 7.

138 Mathematics–X
SECTION-E
6 + 9 17 + 16
36. (i) Mid-point of JI = e o=e ,
15 33
, o
2 2 2 2

(ii) The distance of the point P from the y-axis is 4 unit.


(iii) Distance between A and S = 16 boxes.


OR
Coordinates of A and B are (1, 8) and (5, 10) respectively.
Coordinates of point dividing AB in the ratio 1 : 3 internally are:
1× 5 + 3 ×1 1×10 + 3 × 8
x= , y=
+
1 3 1+ 3

8 34
⇒ x = = 2 and y = = 8 .5
4 4

Co-ordinates of required points be (2, 8.5).


37.
(i) The amount paid in each successive installment is given by `1500, `1700, `1900 ...

It forms an AP with a = 1500, d = 200


(ii) a15 = 1500 + (15 –1)200 = 1500 + 2800 = `4300

(iii) an = 5500
⇒ 1500 + (n – 1)200 = 5500

& n = 4000
200
+ 1 = 21

OR
Since 11, 2k +1, 3k – 1 are in AP.
∴    2k + 1 – 11 = (3k –1) – (2k + 1)
⇒ 2k – 10 = k – 2 ⇒ 2k – k = – 2 + 10 = 8

⇒ k=8
38. (i) Let the distance of two cars from the tower be x and y.

3 3
Then tan 30° = and tan 60° =
x y
1 3 3 3m
⇒ = and 3 =
3 x y

3 30° 60°
⇒ x = 3 3 and y = = 3
3 x y

Distance between the cars = 3 +3 3 =4 3m

Sample Papers 139


(ii) Angle of depression will increase as the car approaches the building.

p 3
(iii) Given, =
b 1

Tower

Shadow

⇒ tan θ = 3 ⇒ q = 60°
OR
If both height and base are increased by 10%.

1 11p
p+ p p
Then, tan '= 10 = 10 = 10% of p
1 11b b
θ

b+ b
10 10
p
⇒ il = i
⇒ Angle of elevation remains unchanged. θ' θ
10%
of b b

zzz

140 Mathematics–X
SAMPLE PAPER
6 [UNSOLVED]

Time allowed: 3 hours Maximum marks: 80


General Instructions: Same as CBSE Sample Question Paper–2023 (Solved).

SECTION-A
Section A consists of 20 questions of 1 mark each.
1. The LCM of smallest two digit composite number and smallest composite number is

(a) 12
(b) 4
(c) 20
(d) 44

2
2. The zeroes of the polynomial x – 3x – m(m + 3) are

(a) m, m + 3
(b) –m, m + 3
(c) m, – (m + 3)
(d) –m, – (m + 3)

3. The value of k for which the lines represented by the following pair of linear equations

are coincident is
2x + 3y + 7 = 0
8x + 12y + k = 0
(a) all real values except 14
(b) 8

(c) 28
(d) 14

2
4. Nature of roots of quadratic equation 2x – 4x + 3 = 0 is

(a) real
(b) equal
(c) not real
(d) none of them

5. Point on y-axis has coordinate


(a) (–a, b)
(b) (a, 0)
(c) (0, b)
(d) (– a, – b)

6. In figure, ∠BAC = 90° and AD ⊥ BC. Then,


(a) BD . CD = BC2
(b) AB . AC = BC2

2 2
(c) BD . CD = AD
(d) AB . AC = AD

Sample Papers 141


7. If DABC ~ DEDF and DABC is not similar to DDEF, then which of the following is not

true?

(a) BC. EF = AC. FD


(b) AB. EF = AC. DE

(c) BC. DE = AB. EF


(d) BC. DE = AB. FD

8. If DPRQ ~ DXYZ, then


PR RQ PQ PR PQ QR QR PR
(a) = (b) = (c) = (d) =
XZ YZ XY XZ XZ YZ XZ XY

9. In figure ABCD is a cyclic quadrilateral and if ∠BAC = 50° and ∠DBC = 60° then the

value of ∠BCD is

(a) 50°
(b) 60° (c) 70°
(d) 100°

tan 30°
10. The value of is
cot 60°


1 1
(a) (b) (c) 3 (d) 1

2 3
11. The value of (sin 45° + cos 45°) is


1 3
(a) (b) 2 (c) (d) 1
2

2
1
12. If sin A = , then the value of cot A is
2


1 3
(a) 3 (b) (c) (d) 1
2

3
13. The perimeter of a circle is equal to that of a square, then the ratio of their areas is

(a) 22 : 7
(b) 14 : 11
(c) 7 : 22
(d) 11 : 14

14. If the circumference of a circle and the perimeter of a square are equal, then

(a) Area of the circle = Area of the square


(b) Area of the circle > Area of the square


(c) Area of the circle < Area of the square


(d) Nothing definite can be said about the relation between the areas of the circle and

square.
15. Two identical solid hemispheres of equal base radius are stuck along their bases. The

total surface area of the combination is


(a) πr2
(b) 2πr2
(c) 3πr2
(d) 4πr2

16. A medicine-capsule is in the shape of a cylinder of radius


0.25 cm with two hemispheres stuck to each of its ends. 0.25 0.25
The length of the entire capsule is 2 cm. What is the total
surface area of the capsule? (Take as 3.14) p

1.50 cm
(a) 0.785 cm2

(b) 0.98125 cm2


2 cm
(c) 2.7475 cm2

(d) 3.14 cm2


142 Mathematics–X
17. Manish wants to polish the object, which is composed of a

cylinder surmounted by a hemisphere. If the whole length of 3.5 m

the solid is 5 m and the diameter of the hemisphere is 7 m, what


is the cost of polishing the surface area of the solid at the rate of
1.5 m
22
50 paise per sq m? e Use r= o
7
(a) `55
(b) ` 110

(c) ` 550
(d) ` 1,100

18. A card is selected from a deck of 52 cards. The probability of being a red face card is

3 3 2 1
(a) (b) (c) (d)
26 13 13 2

DIRECTION: In the question numbers 19 and 20, a statement of assertion (A) is followed by a
statement of Reason (R). Choose the correct option.
(a) Both A and R are true and R is the correct explanation for A.

(b) Both A and R are true and R is not the correct explanation for A.

(c) A is true but R is false.


(d) A is false but R is true.


19. Assertion (A) : 6n ends with the digit zero, where n is natural number.

Reason (R) : Any number ends with digit zero, if its prime factor is of the form

2m × 5n, where m, n are natural numbers.


20. Assertion (A) : The point (0, 4) lies on y-axis.

Reason (R) : The x co-ordinate of the point on y-axis is zero.


SECTION-B
Section B consists of 5 questions of 2 marks each.
21. Is the following pair of linear equations consistent? Justify your answer.

   2ax + by = a, 4ax + 2by – 2a = 0; a, b ≠ 0


22. In triangles PQR and TSM, ∠P = 55°, ∠Q = 25°, ∠M = 100°, and ∠S = 25°. Is

∆QPR ~ ∆TSM? Why?


23. In figure PA and PB are tangents to the circle drawn

from an external point P. CD is the third tangent


touching the circle at Q. If PA = 15 cm, find the
perimeter of ∆PCD.
24. If sin2 A = 2 sin A then find the value of A.

OR
1
Find maximum value of , 0° ≤ q ≤ 90°.
sec i

Sample Papers 143


25. Find the area of the sector of a circle with radius 4 cm and of angle 30°.

Also, find the area of the corresponding major sector. (Use = 3.14) p

O
OR
What is the angle subtended at the centre of a circle of radius 10 cm by
an arc of length 5 cm? p

SECTION-C
Section C consists of 6 questions of 3 marks each.
26. An army contingent of 616 members is to march behind an army band of 32 members

in a parade. The two groups are to march in the same number of columns. What is the
maximum number of columns in which they can march?
27. Find the zeros of the following quadratic polynomials and verify the relationship between

the zeros and coefficients:


4u2 + 8u
28. Solve the following linear equations:

   152x – 378y = –74 and –378x + 152y = –604


OR
For which values of a and b does the following pair of linear equations have an infinite
number of solutions?
2x + 3y = 7
(a – b)x + (a + b)y = 3a + b – 2
29. In figure, if TP and TQ are the two tangents to a circle with centre O so that

∠POQ = 110°, then find ∠PTQ.     

OR
Prove that the tangents drawn at the ends of a diameter of a circle are parallel.

30. If sin
θ + cos θ = 3 , then prove that tan θ + cot θ = 1.

144 Mathematics–X
31. Harpreet tosses two different coins simultaneously (say, one is of `1 and other of `2).

What is the probability that she gets at least one head?

SECTION-D
Section D consists of 4 questions of 5 marks each.
32. Using quadratic formula, solve the following equation for x: abx2 + (b2 – ac) x – bc = 0.

OR
Find the value of p for which the quadratic equation
(2p + 1)x2 – (7p + 2)x + (7p – 3) = 0 has equal roots. Also find these roots.

33. ABCD is a trapezium in which AB|| DC and its diagonals intersect each other at the point

AO CO
O. Show that = .
BO DO
34. From a solid cylinder whose height is 2.4 cm and diameter 1.4 cm, a conical cavity of the

same height and same diameter is hollowed out. Find the total surface area of the
remaining solid to the nearest cm2.
OR
Rasheed got a playing top (lattu) as his birthday present, which
surprisingly had no colour on it. He wanted to colour it with his
crayons. The top is shaped like a cone surmounted by a hemisphere.
The entire top is 5 cm in height and the diameter of the top is 3.5 cm.
22
Find the area he has to colour. c Take r = m
7
35. The following distribution shows the daily pocket allowance of

children of a locality. The mean pocket allowance is ` 18. Find the missing frequency f.
Daily pocket allowance (in `) 11–13 13–15 15–17 17–19 19–21 21–23 23–25
Number of children 7 6 9 13 f 5 4

Section-E
Case study based questions are compulsory.
36. Kriti saves ™24 during the first month ™30 in the second month and ™36 in the third

month. She continues to save in this manner.

Sample Papers 145


On the basis of above information answer the following questions.
(i) Whether the monthly savings of Kriti form an AP or not? If yes then write the first

term and common difference.


(ii) What is the amount that she will save in 15th month?

(iii) In which month, will she save `66?


OR
What is the common difference of an AP whose nth term is 8 – 5n?
37. The top of a table is shown in the figure given below:

Y B C

M N

A T D
O

S P
H E

R Q

G F
O X
Scale : 1 Square = 1 unit

On the basis of above information answer the following questions.


(i) Find the distance between points A and B.

(ii) Write the co-ordinates of the mid point of line segment joining points M and Q.

(iii) If G is taken as the origin, and x, y axis put along GF and GB, then find the point

denoted by coordinate (4, 2).


OR
Find the coordinates of H, G and also find the distance between them.
38. A person/observer on the sea coast observes two ship in the sea, both the ships are in

same straight path one behind the other. O


45°
60°

20 m

60° 45°
C B
A
If the observer is on his building of height 20 meters (including observer) and he
observes the angle of depression of two ships as 45° and 60° respectively.
On the basis of above information answer the following questions.
(i) If a person observes a ship whose angle of depression is 60° then how much distance

is the ship away from him?


(ii) If a person observes another ship whose angle of depression is 45° then how much

distance that ship is away from him?

146 Mathematics–X
(iii) If a person observes the ship whose angle of depression changes from 60° to 30°

then how far be ship from the observer if the observer is at 20 m of height (including
him)?
OR  

At a time when a person observes two ships whose angle of depressions are 60° and
45° the distance between the ships is (in meter).

Answers
1. (c)
2. (b) 3. (c)
4. (c)
5. (c)
6. (c)

7. (c)
8. (c) 9. (b)
10. (d)
11. (b)
12. (a)

13. (b)
14. (b) 15. (d)
16. (d)
17. (a)
18. (a)

19. (d)
20. (a) 21. Yes
22.

QPR is not similar to TSM.

23. 30 cm
24. ∠A = 0° OR 1
25. 46.1 cm2 (approx.) OR 90° 26. 8.

27. 0, –2
28. x = 2, y = 1 OR a = 5 and b = 1.
29. ∠PTQ = 70°

3 c –b –4
31. 32. x = b or x = a OR ; x = 5, 5
4 7 3 3

2 2
34. 18 cm OR 39.6 cm (approx)
35. 20.

36. (i) 6 (ii) ™108 (iii) 8th month OR –5


37. (i) 4 2 units (ii) (7, 7) (iii) Q OR H(1, 5), G(5, 1); 4 2 units

20 3
m (ii) 20 m (iii) 20 3 m OR d20 – nm
20 3
38. (i)
3 3

zzz

Sample Papers 147


SAMPLE PAPER
[UNSOLVED] 17
Time allowed: 3 hours Maximum marks: 80
General Instructions: Same as CBSE Sample Question Paper–2023 (Solved).

SECTION-A
Section A consists of 20 questions of 1 mark each.
1. The total number of factors of a prime number is

(a) 1
(b) 0
(c) 2 (d) 3

2. The degree of polynomial having zeroes – 3 and 4 only is


(a) 2
(b) 1
(c) more than 3
(d) 3

3. The value of a for which the pair of equations 10x + 5y = a – 5, 20x + 10y – a = 0 has

infinitely many solutions is


(a) 5 (b) –10 (c) 10 (d) 20

4. The product of three consecutive integers is equal to 6 times the sum of three integers.

If the smallest integer is x, which of the following equations represent the above
situation?
(a) 2x2 + x − 9 = 0
(b) 2x2 − x + 9 = 0

2
(c) x + 2x + 18 = 0
(d) x2 + 2x − 18 = 0

5. The perimeter of a triangle with vertices (0, 4), (0, 0) and (3, 0) is

(a) 5 units
(b) 12 units
(c) 11 units
(d) 10 units

6. In triangles ABC and DEF, ∠B = ∠E, ∠F = ∠C and AB = 3DE. Then, the two triangles

are
(a) congruent but not similar
(b) similar but not congruent

(c) neither congruent nor similar


(d) congruent as well as similar

7. In figure, two line segments AC and BD intersect each



A
6 cm
D
m 30°
other at the point P such that PA= 6 cm, PB = 3 cm, 5c
P
PC = 2.5 cm, PD = 5 cm, ∠APB = 50° and ∠CDP = 30°. 50°
2.5 c
Then, ∠PBA is equal to 3c
m m
C
B
(a) 50°
(b) 30°

(c) 60°

(d) 100°

148 Mathematics–X
ar (DABC) 9
8. If DABC ~ DQRP, = , AB = 18 cm and BC = 15 cm, then PR is equal to
ar (DPQR) 4

20
(a) 10 cm (b) 12 cm (c) cm (d) 8 cm

9. A figure is shown below.


C
K

A
L

E F

Which of the following is true?


(a) Lines AB and CD are the tangents and line EF is a secant to the circle.

(b) Lines AB and CD are the secants and line EF is a tangent to the circle.

(c) Line AB is a tangent and lines EF and CD are the secants to the circle.

(d) Line AB is a secant and lines EF and CD are the tangents to the circle.

10. If sin A + sin2 A = 1, then the value of the expression (cos2A + cos4A) is

1
(a) 1 (b) (c) 2 (d) 3
2

4 sin i – cos i
11. If 4 tan θ = 3, then d n is equal to
4 sin i + cos i

2 1 1 3
(a) (b) (c) (d)
3 3 2 4

12. Value of sin 30° tan 45° is


3 1
(a) (b) 2 (c) (d) 0
2 2

13. The circumference of two circles are in the ratio 2 : 3. The ratio of their areas is

(a) 2 : 3
(b) 4 : 9
(c) 9 : 4
(d) None of these

14. If the perimeter of a semicircular protractor is 36 cm; its diameter is


(a) 14 cm
(b) 16 cm
(c) 18 cm
(d) 12 cm

15. Which of these is equivalent to p?


Circumference Circumference
(a) (b)
Radius Diameter

(c) Circumference × Diameter


(d) Circumference × Radius

16. Two identical solid cubes of side k units are joined end to end. What is the volume, in

cubic units, of the resulting cuboid?


(a) 2k3

(b) 3k3
(c) 4k3

(d) 6k3

Sample Papers 149


17. A vendor sells glasses of juice, which is in the form of a cylinder

mounted by a hollow hemisphere. The diameter of the 4.2 cm


hemisphere and the cylinder is 8.4 cm. If the total height of the
glass is 15 cm, which of these is closest to the volume, in cubic 4.2 cm
15 cm
22
centimeters, of 8 such glasses? e Use r = o h=10.8 cm
7
(a) 754 cm3

(b) 6032 cm3


3
(c) 29083 cm

(d) None of these


18. One card is drawn from a well shuffled deck of 52 cards. The probability that it is black

queen is
1 1 1 2
(a) (b) (c) (d)
26 13 52 13

DIRECTION: In the question numbers 19 and 20, a statement of assertion (A) is followed by a
statement of Reason (R). Choose the correct option.
(a) Both A and R are true and R is the correct explanation for A.

(b) Both A and R are true and R is not the correct explanation for A.

(c) A is true but R is false.


(d) A is false but R is true.


19. Assertion (A) : For any two positive integers a and b, HCF (a, b) × LCM (a, b) = a × b.

Reason (R) : The HCF of two numbers is 5 and their product is 150. Then their LCM

is 40.
20. Assertion (A) : The value of y is 6, for which the distance between the points P(2, –3) and

Q (10, y) is 10.
Reason (R) : Distance between two given points A (x1, y1) and B (x2, y2) is given by

AB = (x2 – x1) 2 + (y2 – y1) 2 .

SECTION-B
Section B consists of 5 questions of 2 marks each.
21. Find c if the system of equations

cx + 3y + (3 – c) = 0; 12x + cy – c = 0 has infinitely many solutions.


22. If ABC and DEF are similar triangles such that ∠A = 47° and ∠E = 63°, then what is the

measures of ∠C.
23. In Fig., PA and PB are tangents to the circle from an external point

A
C

P. CD is another tangent touching the circle at Q. If PA =12 cm, O Q P

QC = QD = 3 cm, then find PC + PD. D


a B

24. Given that sin = , find the value of tan .


b
q q

OR
If sin = cos , then find the value of 2 tan + cos2 .
q q q q

150 Mathematics–X
25. Difference between the circumference and radius of a circle is 37 cm. Find the area of

circle.
OR
The radii of two circles are 8 cm and 6 cm respectively. Find the radius of the circle
having area equal to the sum of the areas of the two circles.

SECTION-C
Section C consists of 6 questions of 3 marks each.

26. If HCF of 65 and 117 is expressible in the form 65n – 117, find the value of n.

27. If a and b are the zeros of the quadratic polynomial f(x) = 2x2 – 5x + 7, find a polynomial

whose zeros are 2a + 3b and 3a + 2b.


28. For which value of k will the following pair of linear equations have no solution?

   3x + y = 1
   (2k – 1)x + (k – 1)y = 2k + 1
OR
Find whether the following pair of linear equations has a unique solution. If yes, find the
solution.
   7x – 4y = 49 and 5x – 6y = 57
29. The length of a tangent from a point A at distance 5 cm from the centre of the circle is

4 cm. Find the radius of the circle.


OR
Two concentric circles are of radii 5 cm and 3 cm. Find the length of the chord of the
larger circle which touches the smaller circle.
1 − sin θ
30. Prove that : = (sec – tan )2
1 + sin θ
q q

31. A game consists of tossing a one-rupee coin 3 times and noting the outcome each time.

Ramesh wins the game if all the tosses give the same result (i.e. three heads or three tails)
and loses otherwise. Find the probability of Ramesh losing the game.

SECTION-D
Section D consists of 4 questions of 5 marks each.
32. The sum of the reciprocals of Rehman’s age (in years) 3 years ago and 5 years from now is

1
. Find his present age.
3
OR
1
The difference of two natural numbers is 5 and the difference of their reciprocals is .
10

Find the numbers.

Sample Papers 151


33. If AD and PM are medians of triangles ABC and PQR respectively, where ABC ~ PQR,
∆ ∆

AB AD
prove that = .
PQ PM
34. The decorative block shown in figure is made of two solids—a cube

and a hemisphere. The base of the block is a cube with edge 5 cm,
and the hemisphere fixed on the top has a diameter of 4.2 cm. Find
22
the total surface area of the block. cUse r = m
7
OR
2 cm = h2
Rachel, an engineering student, was asked to make a model shaped like a
cylinder with two cones attached at its two ends by using a thin aluminium
sheet. The diameter of the model is 3 cm and its length is 12 cm. If each
8 cm = h1
cone has a height of 2 cm, find the volume of air contained in the model
3 cm
that Rachel made. (Assume the outer and inner dimensions of the model to
be nearly the same.)
35. A survey was conducted by a group of students as a part of their

2 cm = h2

environment awareness programme, in which they collected the following


data regarding the number of plants in 20 houses in a locality. Find the mean number
of plants per house.
Number of plants 0–2 2–4 4–6 6–8 8–10 10–12 12–14
Number of houses 1 2 1 5 6 2 3
Which method did you use for finding the mean and why?

SECTION-E
Case study based questions are compulsory.
36. A number of bricks were stacked in the following manner.

25 bricks in the bottom row,


24 in the next row, 23 in the row next to it and so on, forming rows.

Based on above information answer the following questions.


(i) Write the row number that has 14 bricks.

(ii) What is the number of bricks in the last row?


(iii) Which among the two, cannot be the nth term of an AP? Why?

(a) 4n – 6 (b) 3 – n2

OR
Which is the 8th term from the end of the AP 9, 13, 17, . . . . 65?

152 Mathematics–X
37. Shaurya made a map of his locality on a coordinate plane

Y
Kartik’s Temple
House

Post Office
School Park
O
X
Shaurya’s
Market House Railway
Station
Scale : 1 Square = 1 unit

Based on above information answer the following questions.


(i) If house is considered as origin, then find coordinates of market.

(ii) What is the distance from his house to Kartik’s house?


(iii) The distance of fort from his house is 10 units. If its ordinate is 6, then find its

abscissa.

OR
What is the coordinates of the point which divides the line segment joining school
and park internally in the ratio 3 : 2 ?
38. A Kapil is standing in the balcony of his house. He observes two cars, one on either side

(say A and B).

Kapil

A B

Based on above information answer the following questions.


(i) If the height at which he is standing is 3 m and the distance of one of the cars from

the foot of the building is also 3m, then what is the angle of depression of this car
as observed by him?
(ii) If the ratio of the height of a tower and the length of its shadow is
3 :1, then what
is the angle of elevation of sun?
(iii) At an instant, the angle of depression of two cars were found to be 30° and 60°

respectively. If the height at which he is standing is 3 m, find the distance between


the two cars.
OR
If the height of a building and the distance of the point of observation from its foot
both are increased by 10%, then what is the angle of elevation of its top?

Sample Papers 153


Answers
1. (c)
2. (a)
3. (c)
4. (d)
5. (b)
6. (b)

7. (d)
8. (a)
9. (a)
10. (a)
11. (c)
12. (c)

13. (b)
14. (a)
15. (b)
16. (a)
17. (b)
18. (a)

19. (c)
20. (d)
21. c = 6
22. 70°
23. 18 cm

a 5
24. OR 25. 154 cm2 OR 10 cm 26. n = 2
b2 – a2 2

25
27. k(x2 – x + 41) 28. k = 2 OR yes, x = 3, y = –7 29. 3 cm OR 8 cm
2

3
31. 32. 7 years OR 10 and 5 34. 163.86 cm2 OR 66 cm
4

35. 8.1; By Direct Method because the value of xi and fi are small.

36. (i) 12 (ii) 17 (iii) (b), as for being the nth term the expression should be linear. OR 37

37. (i) (–3, –1) (ii)


20 units (iii) ± 8 OR (–2, 2)

38. (i) 45° (ii) 60° (iii) 4 3 m OR Angle of elevation remains unchanged

zzz

154 Mathematics–X
SAMPLE PAPER
8 [UNSOLVED]

Time allowed: 3 hours Maximum marks: 80


General Instructions: Same as CBSE Sample Question Paper–2023 (Solved).

SECTION-A
Section A consists of 20 questions of 1 mark each.

1. If 6370 = 2m . 5n . 7k . 13p, then the value of m + n + k + p is


(a) 2
(b) 3
(c) 4
(d) 5

2
2. The zeros of the quadratic polynomial ax + bx + c, c ≠ 0 are equal, then

(a) c and a have opposite signs


(b) c and b have opposite signs

(c) c and a have the same sign


(d) c and b have the same sign

3. The value of k, for which the pair of linear equations kx + y = k2 and x + ky = 1 have

infinitely many solutions is


(a) ±1
(b) 1
(c) –1
(d) 2

4. The quadratic polynomial p(x) with –24 and 4 as a product and one of the zeros

respectively is
(a) x2 – 2x – 24
(b) x2 + 2x – 24

2
(c) x + 2x + 24
(d) Can’t be determined

1 5
5. If is a root of the equation x2 + kx – = 0, then the value of k is
2 4

1 1
(a) 2 (b) – 2 (c) (d)
4 2

6. The point which divides the line segment joining the points (8, – 9) and (2, 3) in ratio

1 : 2 internally lies in the

(a) I quadrant
(b) II quadrant

(c) III quadrant


(d) IV quadrant

A

AD AE
7. In the given figure = and +ADE = 70° and +ACB = 50° then
BD EC

+BAC is equal to D E

(a) 70°
(b) 50°

(c) 80°
(d) 60°

B C

Sample Papers 155


8. If ∆ABC is circumscribing a circle in the Fig. The length of AB is

A

(a) 12 cm
(b) 13 cm

(c) 9 cm
(d) 14 cm

12 cm
F E
1 1
9. Given that sin a = and cos b = , then the value of (a + b) is
2 2

B 6 cm D 4 cm C
(a) 0°
(b) 30°

(c) 60°
(d) 90°

10. A pole 6 m high casts a shadow 2 3 m long on the ground, then the Sun’s elevation is

(a) 60°
(b) 45°
(c) 30°
(d) 90°

11. The area of the shaded region in fig, where arcs drawn with

P
A
Q

centres P, Q, R and S intersect in pairs at mid points A, B, C and


D of the sides PQ, QR, RS and SP respectively of a square PQRS,

12 cm
is D B

(a) 30.86 cm2


(b) 30.86 m2

(c) 30.86 mm2


(d) 30.86 km2

S R
12. The radii of two cylinders are in the ratio 5 : 7 and their

C

heights are in the ratio 3 : 5. The ratio of their curved surface area is
(a) 3 : 7
(b) 7 : 3
(c) 5 : 7
(d) 3 : 5

13. For the following distribution:


Class 0–8 8–16 16–24 24–32 32–40


Frequency 12 26 10 9 15
The sum of upper limits of the median class and modal class is
(a) 24
(b) 40
(c) 32
(d) 16

14. If a bag contains 3 red and 7 black balls, the probability of getting a black ball is

3 4 7 5
(a) (b) (c) (d)
10 10 10 10

15. If two tangents inclined at an angle 60° are drawn to a circle of radius 3 cm, then

length of each tangent is equal to


3
(a) 3 cm (b) 6 cm (c) 3 cm (d) 3 3 cm
2

3
16. If tan A = , then the value of cos A is
2

3 2 2 13
(a) (b) (c) (d)
3 2

13 13
17. A racetrack is in the form of a ring whose inner circumference is

352 m and outer circumference is 396 m. The width of the track is


(a) 4 m
(b) 6 m

(c) 8 m
(d) 7 m

156 Mathematics–X
18. When a die is thrown, the probability of getting an even number less than 4 is


1 1 1
(a) (b) 0 (c) (d)
4 2 6

Direction: In the question numbers 19 and 20, a statement of assertion (A) is followed by a statement
of Reason (R). Choose the correct option.
(a) Both A and R are true and R is the correct explanation for A.

(b) Both A and R are true and R is not the correct explanation for A.

(c) A is true but R is false.


(d) A is false but R is true.


19. Assertion (A) : 10th term of an AP is 41 whose first team is 5 and common difference

is 4.
Reason (R) : nth term of an AP is an = a + (n –1) d, where a = first term, d = common

difference.

20. Assertion (A) : When two coins are tossed simultaneously then the probability of getting

1
no tail is .
4
1
Reason (R) : The probability of getting a head (i.e., no tail) in one toss of a coin is .
2

SECTION-B
Section B consists of 5 questions of 2 marks each.
21. If one root of the polynomial p(y) = 7y2 + 14y + m is reciprocal of other, then find the value

of m.
1 + sin A
22. Prove that: = sec A + tan A
1 – sin A

23. Prove that the points (3, 0), (6, 4) and (–1, 3) are the vertices of a right isosceles triangle.

OR
If the point P(x, y) is equidistant from the points A(a + b, b – a) and B(a – b, a + b). Prove
that bx = ay.
24. Prove that: (sin A + cosec A)2 + (cos A + sec A)2 = 7 + tan2 A + cot2 A

25. An integer is chosen at random between 1 and 100. Find the probability that it is :

(i) divisible by 8.
(ii) not divisible by 8.
OR
Find the probability that a leap year selected at random will contain 53 Sundays and 53
Mondays.

Sample Papers 157


SECTION-C
A
Section C consists of 6 questions of 3 marks each.

26. In the given figure, ABCDE is a pentagon with BE || CD and


BC || DE. BC is perpendicular to CD. AB = 5 cm, AE = 5 cm, BE= 7 cm, B E


BC= x – y and CD = x + y. If the perimeter of ABCDE is 27 cm. Find
the value of x and y, given x, y ≠ 0.
27. Solve for x: x2 + 5x – (a2 + a – 6) = 0

C D

28. The digit of a positive number of three digits are in AP and their sum is 15. The number

obtained by reversing the digits is 594 less than the original number. Find the number.
OR
If m times the mth term of an AP is equal to n times its nth term, show that the (m + n)th
term of the AP is zero.
29. Find the coordinates of the points which divide the line segment joining A (–2, 2) and

B (2, 8) into four equal parts.

30. Two concentric circles are of radii 5 cm and 3 cm. Find the length of the chord of the

larger circle which touches the smaller circle. ×

31. Find the area of the shaded region in Fig., where arcs drawn with

centres A, B, C and D intersect in pairs at mid-points P, Q, R and S of × ×


the sides AB, BC, CD and DA respectively of a square ABCD of side
12 cm. [Use π = 3.14] ×
OR
Two circles touch internally. The sum of their areas is 116 cm2 and distance between
p

their centres is 6 cm. Find the radii of the circles.

SECTION-D
Section D consists of 4 questions of 5 marks each.

32. Prove that 3 + 2 5 is an irrational number.


33. ABCD is a trapezium in which AB||DC and its diagonals intersect each other at the point O.

AO CO
Show that = .
BO DO
34. As observed from the top of a light house, 100 m high above sea level, the angles of

depression of a ship, sailing directly towards it, changes from 30° to 60°. Find the distance
travelled by the ship during the period of observation. (Use 3 =1.73)
OR
From a point on the ground, the angles of elevation of the bottom and the top of a tower
fixed at the top of a 20 m high building are 45° and 60° respectively. Find the height of
the tower.

158 Mathematics–X
35. The distribution below gives the weights of 30 students of a class. Find the median

weight of the students.


Weight (in kg) 40–45 45–50 50–55 55–60 60–65 65–70 70–75
Number of students 2 3 8 6 6 3 2

OR
The table below shows the salaries of 280 persons:
Salary
5–10 10–15 15–20 20–25 25–30 30–35 35–40 40–45 45–50
(In thousand `)
No. of persons 49 133 63 15 6 7 4 2 1
Calculate the median salary of the data.

SECTION-E
Case study based questions are compulsory.
36. Vijay is trying to find the average height of a tower near his house. He is using the

properties of similar triangles. The height of Vijay’s house if 20 m when Vijay’s house
casts a shadow 10 m long on the ground. At the same time, the tower casts a shadow
50 m long on the ground and the house of Ajay casts 20 m shadow on the ground.
Based on the above information answer the following questions.
(i) What is the height of the tower?

(ii) What will be the length of the shadow of the tower when Vijay’s house casts a

shadow of 12m?
(iii) What is the height of Ajay's house?

OR
When the tower casts a shadow of 40 m, same time what will be the length of the
shadow of Ajay's house?
37. It is common that governments revise travel fares from time to time based on various

factors such as inflation (a general increase in prices and fall in the purchasing value of
money) on different types of vehicles like auto, rickshaws, taxis, radio cab etc. The auto
charges in a city comprise of a fixed charge together with the charge for the distance
covered. Study the following situations:

Sample Papers 159


Name of the city Distance travelled (km) Amount paid (`)
City A 10 75
15 110
City B 8 91
14 145

Based on the above information answer the following questions.


Situation 1: In city A, for a journey of 10 km, the charge paid is `75 and for a journey of
15 km, the charge paid is `110.
Situation 2: In a city B, for a journey of 8 km, the charge paid is `91 and for a journey
of 14 km, the charge paid is `145.
Refer situation 1
(i) If the fixed charges of auto rickshaw be ` x and the running charges be ` y per km,

then write the pair of linear equations representing the situation.


(ii) A person travels a distance of 50 km. How much amount he has to pay?

Refer situation 2
(iii) What will be a person have to pay for travelling a distance of 30 km?

OR
Out of both the city, which one has cheaper fare?
38. The Great Stupa at Sanchi is one of the oldest stone structure in India, and an important

monument of Indian Architecture. It was originally commissioned by the emperor


Ashoka in the 3rd century BC. Its nucleus was a simple hemispherical brick structure
built over the relics of the Buddha. It is a perfect example of combination of solid figures.
22
A big hemispherical dome with a cuboidal structure mounted on it. c Take r = m
7

Based on the above information answer the following questions.

(i) Find the volume of the hemispherical dome if the height of the dome is 21 m.

(ii) Find the area of cloth required to cover the hemispherical dome if the radius of its

base is 14 m.

160 Mathematics–X
(iii) Find the total surface area of the combined figure i.e., hemispherical dome with

radius 14 m and cuboidal shaped top with dimensions 8 m × 6 m × 4 m.


OR
Write the volume of the cuboidal shaped top with dimensions 8 m × 6 m × 4 m.

Answers
1. (d)
2. (b) 3. (b)
4. (b)
5. (a)
6. (d)

7. (d)
8. (d) 9. (d)
10. (a)
11. (a)
12. (a)

13. (c)
14. (c) 15. (d)
16. (b)
17. (d)
18. (d)

6 43 1
19. (a) 20. (a) 21. m = 7 25. (i) (ii) OR
49 49 7

26. x = 6, y = 1
27. –(a + 3), (a – 2)
28. 852 OR 258

13
29. 1, 30. 8 31. 30.96 cm2 OR 10 cm, 4 cm
2

34. 115.33 m OR 14.6 meters 35. 56.67 kg OR 13.421 thousand rupee



36. (i) 100 m (ii) 60 m (iii) 40 m OR 16 m


37. (i) x + 10y =75, x + 15y = 110 (ii) `355


(iii) `289 OR City A

3 2
38. (i) 19404 m (ii) 1232 m
(iii) 1392 m OR 192 m3

2

zzz

Sample Papers 161


SAMPLE PAPER
[UNSOLVED] 19
Time allowed: 3 hours Maximum marks: 80
General Instructions: Same as CBSE Sample Question Paper–2023 (Solved).

SECTION-A
Section A consists of 20 questions of 1 mark each.
1. Arnav has 40 cm long red and 84 cm long blue ribbon. He cuts each ribbon into pieces

such that all pieces are of equal length. What is the length of each piece?

(a) 4 cm as it is the HCF of 40 and 84


(b) 4 cm as it is the LCM of 40 and 84


(c) 12 cm as it is the LCM of 40 and 84


(d) 12 cm as it is the HCF of 40 and 84


2. If one of the zeros of the quadratic polynomial (k – 1) x2 + k x + 1 is – 3, then the value


of k is
4 –4 2 –2
(a) (b) (c) (d)
3 3 3 3

m
2
3. Consider the expression x(m – 1) + 3x 2 , where m is a constant. For what value of m, will

the expression be a cubic polynomial?

(a) 1
(b) 2
(c) –1
(d) –2

4. Shipra gave a note of ™ 2,000 for a pair of jeans worth ™ 500. She was returned 11

notes in denominations of ™ 200 and ™ 100. Which pair of equations can be used to
find the number of ™ 200 notes, x, and the number of ™ 100 notes y? How many notes
of ™ 200 did she get?
(a) x+y = 11 and 200x+100y = 1500; 4

(b) x = y+11 and 200x+100y = 2000; 4


(c) x+y = 15 and 200x+100y = 1800; 10


(d) x+y = 15 and 100x+200y = 1800; 12


5. The centre of a circle whose end points of a diameter are (–6, 3) and (6, 4) is

(c) c0, m (d) c 4, m


7 7
(a) (8, –8) (b) (4, 7)
2 2

162 Mathematics–X
6. Consider the figure below.

C

cm
2.4
A
1.6 c
70° m
70°
O
0.8 c
m D

4.8 cm

Which of the following statement is correct about the triangles in the figure?
AO BO
(a) DAOB ~ DDOC because =
DO CO

(b) DAOB ~ DDOC because +AOB = +DOC


AO BO
(c) DAOB ~ DDOC because = and +BAO = +CDO
DO CO

AO BO
(d) DAOB ~ DDOC because = and +AOB = +DOC
DO CO

7. In Fig., if PA and PB are tangents to the circle with centre O


such that ∠APB = 50°, then ∠OAB is equal to


(a) 25°
(b) 30°

(c) 40°
(d) 50°

5+ 5
8. The quadratic equation with real co-efficients whose one root is , is
2

(a) x2 – 5x + 5 = 0
(b) x2 + 5x – 5 = 0

(c) x2 – 5x – 5 = 0
(d) x2 – 5x + 1 = 0

1
9. If sin i = , the value of (2cot2 q + 2) is
3

(a) 16
(b) 20
(c) 12
(d) 18

10. At one end A of a diameter AB of a circle of radius 5 cm, tangent



X
C
XAY is drawn to the circle. The length of the chord CD parallel to
XY and at a distance 8 cm from A is
m
5c

A B
(a) 4 cm
(b) 5 cm
5 cm O 3 cm

(c) 6 cm
(d) 8 cm

D
11. (sec A + tan A) (1 – sin A) is equal to

Y
(a) sec A

(b) sin A

(c) cosec A
(d) cos A

Sample Papers 163


12. David draws a circle with diameter 6 units. He draws another circle by increasing

the radius of the previously drawn circle by 4 units. What would be the quotient if he
divides the circumference of the newly formed circle by its diameter?
(a) 8 (b) 12
(c) p (d) 2p

13. A coin is tossed 1000 times and 640 times a ‘head’ occurs. The empirical probability

of occurrence of a head in this case is


(a) 0.6
(b) 0.64
(c) 0.36
(d) 0.064

14. The class marks of the class 18–22 is


(a) 4 (b) 18
(c) 22 (d) 20

2 2
15. 9 sec A – 9tan A is equal to

(a) 1 (b) 9
(c) 8 (d) 0

16. A cubical ice cream brick of edge 22 cm is to be distributed among some children

by filling ice cream cones of radius 2 cm and height 7 cm upto its brim. How many
children will get the ice cream cones?
(a) 163
(b) 263
(c) 363
(d) 463

17. A pendulum swings through an angle of 30° and describes an arc 8.8 cm is length. The

length of the pendulum is


(a) 16 cm
(b) 16.8 cm
(c) 16.4 cm
(d) none of them

18. If a pair of dice is thrown, the probability of getting a sum of 10 is


1 1 1 1
(a) (b) (c) (d)
12 36 9 4

DIRECTION: In the question numbers 19 and 20, a statement of assertion (A) is followed by a
statement of Reason (R). Choose the correct option.
(a) Both A and R are true and R is the correct explanation for A.

(b) Both A and R are true and R is not the correct explanation for A.

(c) A is true but R is false.


(d) A is false but R is true.


19. Assertion (A) : In an AP with a = 15, d = –3 then 6th term will be zero.

Reason (R) : a – d, a, a + d are three numbers in AP.


20. Assertion (A) : The probability of winning a game is 0.4, then the probability of losing

it, is 0.6.
Reason (R) : P(E) + P (not E) = 1

(a) Both A and R are true and R is the correct explanation for A.

(b) Both A and R are true and R is not the correct explanation for A.

(c) A is true but R is false.


(d) A is false but R is true.


164 Mathematics–X
Section-B
Section B consists of 5 questions of 2 marks each.

21. Find the zeros of the polynomial p(x) = 4x2 – 12x + 9.


22. Find the ratio in which the point (–3, k) divides the line-segment joining the points

(–5, –4) and (–2, 3). Also find the value of k.

OR
If two adjacent vertices of a parallelogram are (3, 2) and (– 1, 0) and the diagonals
intersect at (2, – 5), then find the coordinates of the other two vertices.
1
23. Prove that: (cosec A – sin A) (sec A – cos A) =
tan A + cot A

24. A child has a die whose six faces show the letters as shown below:

A B C D E A

The die is thrown once. What is the probability of getting (i) A (ii) D ?
OR
Two dice are thrown at the same time and the product of numbers appearing on them
is noted. Find the probability that the product is a prime number.
cos i – sin i 1 – 3
25. Find an acute angle q, when = .
cos i + sin i 1 + 3

Section-C
Section C consists of 6 questions of 3 marks each.

26. Solve: ax + by = a – b and bx – ay = a + b


27. Find the nature of the roots of the following quadratic equation. If the real roots exist, find

them:
3x2 – 4 3 x + 4 = 0
OR
Find the value of k for the following quadratic equation, so that they have two equal
roots.
kx (x – 2) + 6 = 0

28. Determine the AP whose third term is 16 and 7th term exceeds the 5th term by 12.

29. In what ratio does the point c , y m divide the line segment joining the point P (2, –2)
24
11

and Q (3, 7)? Also find the value of y.


30. In Fig., XY and X'Y' are two parallel tangents to a circle with

centre O and another tangent AB with point of contact C


intersecting XY at A and X'Y' at B. Prove that ∠AOB = 90°.


Sample Papers 165


31. A chord PQ of a circle of radius 10 cm subtends an angle of 60° at the centre of circle.

Find the area of major and minor segments of the circle.


OR
In Fig., three section of a circle of radius 7 cm, making angles of 60°,
80°
80° and 40° at the centre are shaded. Find the area of the shaded
60°

region.
40°

Section-D
Section D consists of 4 questions of 5 marks each.

32. Prove that


3 is an irrational number.
33. In Fig., ∆FEC  ∆GDB and ∠1 = ∠ 2. Prove that ∆ADE ~ ∆ABC.

34. The shadow of a tower standing on a level ground is found to be 40 m longer when the

Sun’s altitude is 30° than when it was 60°. Find the height of the tower. (Use 3 = 1.732] )
OR
A man standing on the deck of a ship, which is 10 m above the water level, observes the
angle of elevation of the top of a hill as 60° and the angle of depression of the base of the
hill as 30°. Calculate the distance of the hill from the ship and the height of the hill.
35. The lengths of 40 leaves of a plant are measured correctly to the nearest millimetre, and

the data obtained is represented in the following table:


Length
118–126 127–135 136–144 145–153 154–162 163–171 172–180
(in mm)
Number
3 5 9 12 5 4 2
of leaves
Find the median length of the leaves.
OR
The following data gives the information on the observed lifetimes (in hours) of 225
electrical components:

Lifetime (in hours) 0–20 20–40 40–60 60–80 80–100 100–120


Frequency 10 35 52 61 38 29

Determine the modal lifetimes of the components.

166 Mathematics–X
Section-E
Case study based questions are compulsory. y
x 2m
36. Amit is planning to buy a house and the layout is

Bedroom 1 Bath Kitchen


5m room
given figure. The design and the measurement
2m
has been made such that areas of two bedrooms Living Room
and kitchen together is 95 sq.m. 5m Bedroom 2
On the basis of above information answer the 15 m
following questions.
(i) Write the pair of linear equations in two variables from this situation.

(ii) Find the length of the outer boundary of the layout.


(iii) Find the area of each bedroom and kitchen in the layout.

OR
Find the area of living room in the layout.
37. Two trees are standing parallel to each other. The bigger tree 8 m high, casts a shadow

of 6 m.
B

E
A C

Based on the above information answer the following questions.


(i) If AB and CD are the two trees and AE is the shadow of the longer tree, then state

whether TAEB + TCED , give reason.


(ii) If the ratio of the height of two trees is 3 : 1, then write the shadow of the smaller

tree.
(iii) Find the distance of point B from E.

OR
If the ratio of the height of two trees is 4 : 1, then find the shadow of the smaller tree.
38. A child is playing with clay. He has a clay ball of diameter 24 cm. He first formed three

identical cylindrical pillars out of this ball.


When he finished playing with the pillars, he again mixed all the clay and formed some
identical small balls, each of diameter 8 cm.

Sample Papers 167


Based on the above information answer the following questions.
(i) If the diameter of each cylinder formed by him is 16 cm, then find the height of

each of them.
(ii) Write the number of small balls formed by the child.

(iii) A solid sphere of radius r is melted and recast into the shape of a solid cone of

height equal to the diameter of the sphere. What will be the radius of the cone?
OR
A solid cylinder of radius r and height h is placed over other cylinder of same
height and radius. Find the total surface area of the shape so formed.

Answers
1. (a)
2. (a)
3. (b)
4. (a)
5. (c)
6. (d)

7. (a)
8. (a)
9. (d)
10. (d)
11. (d)
12. (c)

13. (b)
14. (d)
15. (b)
16. (c)
17. (b)
18. (a)

3 3 2
19. (b) 20. (a) 21. , 22. 2 : 1, OR (1, –12) and (5, –10)
2 2 3

1 1
24. (i) (ii) 25. 60° 26. x = 1, y = 1
3 6

2 3 2 3
27. Real and equal roots, , OR k = 6 28. AP is 4, 10, 16, ...
3 3

–4 25 25
29. 2 : 9, y = 31. (44 – 21 3 ) cm2; (220 + 21 3 ) cm2 OR 77 cm2
11 21 21

34. 34.6 m OR 11.39 m; 40 m 35. 146.75 mm OR 65.625 hours


36. (i) x + y = 13
(ii) 54 m (iii) 30 m2; 35 m2 OR 75 m2

37. (i) Yes (ii) 2 m (iii) 10 m OR 1.5 m


38. (i) 12 cm (ii) 27 (iii) 2 r OR 4rrh + 2rr 2


zzz

168 Mathematics–X
SAMPLE PAPER
10 [UNSOLVED]

Time allowed: 3 hours Maximum marks: 80


General Instructions: Same as CBSE Sample Question Paper–2023 (Solved).

SECTION-A
Section A consists of 20 questions of 1 mark each.

1. 5 is the prime factor of


(a) 78
(b) 240
(c) 1001
(d) 1547

2. The pair of equations x = a and y = b graphically represents lines which are




(a) parallel
(b) intersecting at (b, a)

(c) coincident
(d) intersecting at (a, b)

3. The two consecutive odd positive integers, sum of whose squares is 290 are

(a) 13, 15
(b) 11, 13
(c) 7, 9
(d) 5, 7

2
4. If 2 and α are zeros of 2x – 6x + 4 then the value of α is equal to

(a) 2
(b) 3
(c) 1
(d) 5

5. A quadratic polynomial, whose zeros are 5 and – 8 is


(a) x2 + 13x – 40
(b) x2 + 4x – 3
(c) x2 – 3x + 40
(d) x2 + 3x – 40

6. The point P(1, 2) divides the line joining of A(–2, 1) and B(7, 4) in the ratio

(a) 1:2
(b) 2:1
(c) 3:2
(d) 2:3

7. It is given that DABC ~ DDFE, ∠A = 30°, ∠C = 50°, AB = 5 cm, AC = 8 cm and


DF = 7.5 cm. Then, which of the following is true? 

(a) DE = 12 cm, ∠F = 50°


(b) DE = 12 cm, ∠F = 100°

(c) EF = 12 cm, ∠D = 100°


(d) EF = 12 cm, ∠D = 30°

2 2 4
8. If cos A + cos A = 1, then sin A + sin A is equal to

(a) –1
(b) 0 (c) 1 (d) None of these

9. In Fig., AB is a chord of the circle and AOC is its diameter such


C

that ∠ACB = 50°. If AT is the tangent to the circle at the point


A, then ∠BAT is equal to

(a) 65°
(b) 60°

(c) 50°
(d) 40°

T

Sample Papers 169


10. Which of the following option makes the statement below true?

1
sec x + sec x
cos2 x – 1 – tan2 x

(a) −cosec x tan x (b) −sec x tan x (c) −cosec x cot x (d) −sec x cot x
11. From a point X, the length of the tangent to a circle is 20 cm and the distance of X from

the centre is 25 cm. The radius of the circle is


(a) 10 cm
(b) 5 41 cm
(c) 15 cm
(d) 20 cm

12. A ladder 18 m long makes an angle of 60° with a wall. The height of the point where

the ladder reaches the wall is


(a) 9 3 m
(b) 18 3 m
(c) 18 m
(d) 9 m

13. A chord AB of a circle of radius 10 cm makes a right angle at the


centre of the circle. The area of major segment is O

cm
(a) 210 cm2 (b) 285.7 cm2 90°

10

A B
(c) 185.5 cm2

(d) 258.1 cm2

14. The runs scored by a batsman in 35 different matches are given below:

Runs scored 0–15 15–30 30–45 45–60 60–75 75–90


Frequency 5 7 4 8 8 3
The number of matches in which the batsman scored less than 60 runs are
(a) 16
(b) 24
(c) 8 (d) 19

15. A solid piece of iron in the form of a cuboid of dimensions 49cm × 33cm × 24cm, is

moulded to form a solid sphere. The radius of the sphere is


(a) 21 cm
(b) 23 cm
(c) 25 cm
(d) 19 cm

16. The probability of getting a number from 1 to 100, which is divisible by 7 is


1 14 7 23
(a) (b) (c) (d)
7 100 100 98

17. A drain cover is made from a square metal of side 40 cm having 441 holes of diameter

1 cm each drilled in it. The area of remaining square plate is


(a) 1250.5 cm2
(b) 1256.5 cm2
(c) 1253.5 cm2
(d) none of them

18. Two coins are tossed simultaneously. The probability of getting atmost one head is

1 1 3
(a) (b) (c) (d) 1
4 2 4

DIRECTION: In the question numbers 19 and 20, a statement of assertion (A) is followed by a
statement of Reason (R). Choose the correct option.
(a) Both A and R are true and R is the correct explanation for A.

(b) Both A and R are true and R is not the correct explanation for A.

(c) A is true but R is false.


(d) A is false but R is true.


170 Mathematics–X
19. Assertion (A) : Card numbered as 1, 2, 3 ... 15 are put in a box and mixed thoroughly,

one card is then drawn at random. The probability of drawing an even


1
number is .
2
Reason (R) : For any event E, we have 0 # P (E) # 1 .

20. Assertion (A) : The 6th term from the end of the AP 5, 2, –1, –4, ..., –31 is –16.

Reason (R) : General term (nth term) from the beginning is given by an = a + (n – 1) d.

SECTION-B
Section B consists of 5 questions of 2 marks each.

21. If
a and b are zeros of polynomial p(x) = x2 – 5x + 6, then find the value of a + b – 3 b.
a

22. If the point C (–1, 2) divides internally the line segment joining the points A(2, 5) and

B(x, y) in the ratio of 3 : 4, find the value of x2 + y2.


OR
If (1, 2), (4, y), (x, 6) and (3, 5) are the vertices of a parallelogram taken in order, find
x and y.
cos 3 i + sin 3 i cos 3 i – sin 3 i
23. Prove that : + =2
cos i + sin i cos i – sin i

24. Rahim tosses two different coins simultaneously. Find the probability of getting at least

one tail.
OR
12 defective pens are accidentally mixed with 132 good ones. It is not possible to just
look at a pen and tell whether or not it is defective. One pen is taken out at random from
this lot. Determine the probability that the pen taken out is a good one.
25. If cos i + sin i = 2 cos i, show that cos i – sin i = 2 sin i .

SECTION-C
Section C consists of 6 questions of 3 marks each.

26. Solve for x and y


b a 2 2

a x + b y = a + b ; x + y = 2ab
27. A two digit number is four times the sum of the digits. It is also equal to 3 times the

product of digits. Find the number.


OR
Find the roots of the following quadratic equation by factorisation:
1
2x 2 – x + = 0
8

28. An AP consists of 50 terms of which 3rd term is 12 and the last term is 106. Find the 29th

term.
29. Find the ratio in which the line segment joining the points A(3, –3) and B(–2, 7) is

divided by x-axis. Also find the coordinates of the point of division.

Sample Papers 171


30. In Fig., two equal circles, with centres O and O', touch each

other at X.OO' produced meets the circle with centre O' at A.


AC is tangent to the circle with centre O, at the point C. O'D
DOl
is perpendicular to AC. Find the value of .
CO
31. In Fig., O is the centre of a circle such that diameter AB =13 cm

and AC = 12 cm. BC is joined. Find the area of the shaded region.


(Take p = 3.14)
OR
On a square handkerchief, nine circular designs, each of radius
7 cm are made (see Fig.). Find the area of the remaining portion of
the handkerchief.

SECTION-D
Section D consists of 4 questions of 5 marks each.

32. Prove that


5 is an irrational number.
33. Sides AB and AC and median AD of a triangle ABC are respectively proportional to sides

PQ and PR and median PM of another triangle PQR. Show that ABC ~ PQR.∆ ∆

34. A statue, 1.6 m tall, stands on the top of a pedestal. From a point on the ground, the

angle of elevation of the top of the statue is 60° and from the same point, the angle of
elevation of the top of the pedestal is 45°. Find the height of the pedestal.
OR
A 1.2 m tall girl spots a balloon moving
with the wind in a horizontal line at a
height of 88.2 m from the ground. The
angle of elevation of the balloon from the
eyes of the girl at any instant is 60°. After
some time, the angle of elevation reduces
to 30° (Fig.). Find the distance travelled
by the balloon during the interval.
35. The lengths of 40 leaves of a plant

are measured correctly to the nearest


millimetre, and the data obtained is
represented in the following table:
Length
(in mm) 118–126 127–135 136–144 145–153 154–162 163–171 172–180

Number
of leaves 3 5 9 12 5 4 2

Find the median length of the leaves.

172 Mathematics–X
OR
The following table gives the literacy rate (in percentage) of 35 cities. Find the mean literacy
rate.
Literacy rate (in %) 45–55 55–65 65–75 75–85 85–90
Number of cities 3 10 11 8 3

SECTION-E
Case study based questions are compulsory.
36. A test consists of ‘True’ or ‘False’ questions. One mark is awarded for every correct

answer while ¼ mark is deducted for every wrong answer. A student knew correct
answers of some of the questions. Rest of the questions he attempted by guessing. He
answered 120 questions and got 90 marks.
Marks given for Marks deducted for
Type of Question
correct answer wrong answer

True/False 1 0.25
On the basis of above information answer the following questions.
(i) If answer to all questions he attempted by guessing were wrong, then write the

number of questions did he answer correctly.


(ii) If answer to all questions he attempted by guessing were wrong and answered 80

correctly, then how many marks will he get?


(iii) If answer to all questions he attempted by guessing were wrong, then how many

questions were answered correctly to score 95 marks?


OR
What is the maximum marks that a student can score?
37. The ratio of two corresponding sides in similar figures is called scale factor.

Length of image
Scale factor =
Actual length of object

Based on the above information answer the following questions.


(i) A model of a car is made on the scale 1 : 8. The model is 40 cm long and 20 cm

wide. Find the actual length of car.


(ii) If two similar triangles have a scale factor of 2 : 5, then write the ratio of their

median.
(iii) The shadow of a statue 8 m long has length 5 m. At the same time what is the

shadow of a pole of 5.6 m height?

Sample Papers 173


OR
Two similar triangles have a scale factor of 1 : 2. Then write the ratio of their
corresponding altitudes.
38. In a medical laboratory a lab technician composed a capsule in the shape of a cylinder

with two hemispheres stuck to each of its ends. The length of entire capsule is 15 mm
and the diameter of the cylindrical portion is 7 mm.

Based on the above information answer the following questions.


(i) Write the formula to calculate the volume of the capsule.
(ii) Write the volume of 10 such capsules (approximately).

(iii) Find the surface area of 2 such capsules.


OR
A hemisphere and a cylinder stand on equal bases and have the same height. Then
find the ratio of their volume.

Answers
1. (d)
2. (a)
3. (b)
4. (c)
5. (d)
6. (a)

7. (b)
8. (c)
9. (c)
10. (c)
11. (c)
12. (a)

13. (b)
14. (b)
15. (a)
16. (b)
17. (c)
18. (c)

19. (d)
20. (b)
21. –13
22. 29 OR x = 6 and y = 3

3 11 1 1
24. OR 26. x = ab, y = ab 27. 24 OR and
4 12 4 4

3
29. d , 0 n
1
28. a29 = 64 30. 31. 36.33 cm2 OR 378 cm2
2 3

34. 0.8 ^ 3 + 1 h m OR 58 3 metres


35. 146.75 mm OR 69.43%

36. (i) 96 (ii) 70 (iii) 100 OR 120


37. (i) 320 cm (ii) 2 : 5 (iii) 3.5 m OR 1 : 2

4
38. (i) rr 2 d h + r n (ii) 4876.7 mm (iii) 660 mm OR 2 : 3.
3 2
3

zzz

174 Mathematics–X
SAMPLE PAPER
11 [UNSOLVED]

Time allowed: 3 hours Maximum marks: 80


General Instructions: Same as CBSE Sample Question Paper–2023 (Solved).

SECTION-A
Section A consists of 20 questions of 1 mark each.
1. The product of the zeros of the polynomial 4x2 + 3x + 7 is

3 3 7 –7
(a) (b) – (c) (d)
4 4 4 4

2. Two numbers are in the ratio 1 : 3. If 5 is added to both the numbers, the ratio becomes

1 : 2. The numbers are


(a) 4 and 12
(b) 5 and 15
(c) 6 and 18
(d) 7 and 21

3. If HCF (26, 169) = 13 then LCM (26, 169) is


(a) 26
(b) 52
(c) 338
(d) 13

2
4. If the difference of roots of the quadratic equation x + kx + 12 = 0 is 1, the positive

value of k is
(a) – 7
(b) 7
(c) 4
(d) 8

5. A point P divides the join of A(5, –2) and B(9, 6) in the ratio 3:1. The co-ordinates of

P are
11
(a) (4, 7) (b) (8, 4) (c) e , 5 o (d) (12, 8)
2

6. Prashant claims that the polynomial p(x) = mxa + x2b (a > 2b) has 4b zeroes. For

Prashant’s claim to be correct, which of these must be true?


(a) a = 2 or a = 4b
(b) a = 4b
(c) m = 2b
(d) m = 4b

7. Observe the two triangles shown below.


P 4.8
F E
y
y

3.2 4 6

Q R
D

Sample Papers 175


Which statement is correct?
(a) Triangles are similar by SSA.

(b) Triangles are similar by SAS.


(c) Triangles are not similar as sides are not in proportion.


(d) No valid conclusion about similarity of triangles can be made as angle measures are not.

8. In the given figure, ∠CBD = 56° and ∠ABC = 65°. What is the
C
measure of ∠ACB?
(a) 56°

(b) 90°

(c) 59°

D
(d) 121°

7
9. If sin q = , what are the values of tan q, cos q and cosec q?
85

6 7 85
(a) tan i = , cos i = and cosec i =
7 85 7

7 7 85
(b) tan i = , cos i = and cosec i =
6 85 7

7 6 85
(c) tan i = , cos i = and cosec i =
6 85 7

7 6 85
(d) tan i = , cos i = and cosec i =
6 85 6

10. The number of tangents that can be drawn to a circle from a point inside it is

(a) one (b) two


(c) infinite
(d) none

1
and x = tan i , then the value of d x – 2 n is
2 2
11. If 2x = sec
x
θ

1 1
(a) 4 (b) (c) 2 (d)
4 2

12. Which of the following options represents the shaded region as the major sector and

unshaded region as the minor sector?

(a) (b)

(c) (d)

176 Mathematics–X
13. A bag contains three green marbles, four blue marbles and two orange marbles. If a

marble is picked at random, then the probability that it is not an orange marble is
7 2 4
(a) (b) (c) (d) none of these
9 9 9

14. The height of a pole is 10 m. What is the length of the shadow when Sun’s altitude is

30°?
10
(a) m
(b) 10 3 m
(c) 10 m
(d) 15 m

3
15. Two concentric circles of radii 8 cm and 5 cm are shown

below, and a sector forms an angle of 60° at the centre O.


What is the area of the shaded region? B
5 c m
38r 77r C
(a) cm2 (b) cm2 O
2 2 60°

195r 295r 8c
(c) cm2 (d) cm2 m D
6 6

f1 – fo A
16. In the formula, Mode = l + f p × h, f2 is
2f1 – f0 – f2

(a) frequency of the modal class


(b) frequency of the second class


(c) frequency of the class preceding the modal class


(d) frequency of the class succeeding the modal class


17. The length of the diagonal of a cube is 8 3 cm. Its total surface area is

(a) 321 cm2


(b) 350 cm2
(c) 384 cm2
(d) 256 cm2

18. A card is selected at random from a well shuffled deck of 52 playing cards. The

probability of its being a face card is


3 4 6 9
(a) (b) (c) (d)
13 13 13 13

DIRECTION: In the question numbers 19 and 20, a statement of assertion (A) is followed by a
statement of Reason (R). Choose the correct option.
(a) Both A and R are true and R is the correct explanation for A.

(b) Both A and R are true and R is not the correct explanation for A.

(c) A is true but R is false.


(d) A is false but R is true.


19. Assertion (A) : 12th term of the AP 30, 27, 24, ... is the first negative term of the AP.

Reason(R) : Sum of first terms of an AP a, a + d, a + 2d, ... is given by


n
Sn = {2a + (n – 1) d} .
2
20. Assertion (A) : In a simultaneously throw of a pair of dice. The probability of getting a

1
double is .
6
Reason (R) : Probability of an event may be negative.

Sample Papers 177


SECTION-B
Section B consists of 5 questions of 2 marks each.
2
21. If and –3 are the zeros of the polynomial ax2 + 7x + b, then find the values of a and b.
3

22. Find the ratio in which the line segment joining A(1, –5) and B(–4, 5) is divided by the

x-axis. Also find the coordinates of the point of division.


OR
Find the point on the x-axis which is equidistant from (2, – 5) and (– 2, 9).
23. If sin
θ + cos θ = 3 , then prove that tan θ + cot θ = 1.
24. Subhash enjoys walking every morning. As it is very cold outside, he usually take his

sweaters when he goes out for walking. He has 3 red sweaters, 7 black sweaters and 10
white sweaters. One day, while he was about to take his sweater from the cupboard, the
power goes off.
(i) If he picks one sweater at random, how likely is it that he will get a white sweater?

(ii) If he picks one sweater at random, how likely is it that he will get either red or black

sweater?
OR
A child has a die whose six faces show the letters as given below:
A B C A A B
The die is thrown once. What is the probability of getting (i) A? (ii) B?
1 – sin i
25. Prove that = (sec – tan )2.
1 + sin i
θ θ

SECTION-C
Section C consists of 6 questions of 3 marks each.

26. Solve the following pair of equations for x and y.


a2 b2 a2 b b2 a

x – y = 0 ; x + y = a + b, x ! 0, y ! 0
27. If the roots of the quadratic equation (x – a) (x – b) + (x – b) (x – c) + (x – c) (x – a) = 0 are

equal, then show that a = b = c.

OR
2x – 1 x+3
Solve for x: 2 c m – 3c m = 5; x ! – 3,
1
Y
x+3 2x – 1 2

C
28. In the given figure, ∆ABC is an equilateral

triangle of side 3 units. Find the coordinates of


the other two vertices.

X' X
O A (2,0) B
Y'

178 Mathematics–X
29. A circle is touching the side BC of DABC at P and touching AB and AC produced at Q

1
and R respectively. Prove that AQ = (Perimeter of DABC).
2
30. The ratio of the sums of m and n terms of an AP is m2 : n2. Show that the ratio of the mth

and nth terms is (2m – 1) : (2n – 1).


31. In figure, ABCD is a trapezium of area 24.5 sq. cm. In it, AD

|| BC, ∠DAB = 90°, AD = 10 cm and BC = 4 cm. If ABE is


a quadrant of a circle, find the area of the shaded region.
22
(Take p = )
7
OR
In figure, are shown two arcs PAQ and PBQ. Arc PAQ is a part
of circle with centre O and radius OP while arc PBQ is a semi-
circle drawn on PQ as diameter with centre M.

If OP = PQ = 10 cm show that area of shaded region is


25 c 3 – m cm2.
r
6

SECTION-D
Section D consists of 4 questions of 5 marks each.

32. Prove that


5 is an irrational number and hence show that 3 + 5 is also an irrational
number.
5
33. At a point, the angle of elevation of a tower is such that its tangent is . On walking
12

3
240 m nearer to the tower, the tangent of the angle of elevation becomes . Find the
4
height of the tower.
OR
A TV tower stands vertically on a bank of a canal. From
a point on the other bank directly opposite the tower,
the angle of elevation of the top of the tower is 60°.
From another point 20 m away from this point on the
line joining this point to the foot of the tower, the angle
of elevation of the top of the tower is 30° (Figure). Find
the height of the tower and the width of the canal.
34. In figure, ABD is a triangle right-angled at A and

AC⊥ BD. Show that


(i) AB2 = BC . BD

(ii) AD2 = BD . CD

2
(iii) AC = BC . DC

Sample Papers 179


35. The mean of the following frequency table is 50. But the frequencies f1 and f2 in class

20–40 and 60–80 respectively are missing. Find the missing frequencies.
Classes 0–20 20–40 40–60 60–80 80–100 Total
Frequency 17 f1 32 f2 19 120
OR
If the median of the distribution given below is 28.5, find the values of x and y.

Class interval 0–10 10–20 20–30 30–40 40–50 50–60 Total

Frequency 5 x 20 15 y 5 60

SECTION-E
Case study based questions are compulsory.
36. A cricket bat manufacturer’s revenue is the function used to calculate the amount of

money that comes into the business. It can be represented by the equation R = xp,
where x = quantity and p = price.
The revenue function is shown in orange colour in the figure. The cost function is the
function used to calculate the costs of doing business. It includes fixed costs, such as rent
and salaries, and variable costs such as utilities. The cost function is shown in blue colour
in figure.
Y
r
lou
co

140
ge

Blue colour
an
Or

120
y = 100x + 30000
Money (in thousands rupees)

100

80

60
0x
y = 15

40

20

X
O 5 10 15 20 25 30
Quantity (in hundred)

The x-axis represents quantity (in hundreds) of units, and the y-axis represents either
cost or revenue (in thousand of rupees).

180 Mathematics–X
The profit function is the difference of revenue function and the cost function, written
as
P(x) = R(x) – C(x)
Now, let C(x) = 100x + 30000 and R(x) = 150x
If we replace function by y, we get
Linear equations are y = 100x + 30000
and y = 150x
On the basis of above information answer the following questions.
(i) Write the number of bat manufactured (x) so that there is no profit or loss for the

manufacturer.
(ii) State whether the system of linear equations 3x + 2y = 12, 5x – 2y = 4 represents the

intersecting lines or not. Give reason.


(iii) Find the solution of the system of linear equations x + y = 1, 2x – 3y = 7.

OR
Find the value of K for which the system of linear equation 4x + 5y = 3 and
Kx + 15y = 9 has infinitely many solutions.
37. A farmer had a triangular piece of land. He put a fence, parallel to one of the sides of

the field as shown in the figure.

D E

B C

On the basis of above information answer the following questions.


(i) If the point D is 20 m away from A, where as AB and AC are 80 m and 100 m

respectively. Find the length of AE.


(ii) If AD = x + 1, DB = 3x – 1, AE = x + 3, EC = 3x + 4, then find the value of x.

BD AE
(iii) State whether = or not give reason.
AD EC

OR
If P and Q are the mid points of sides YZ and XZ respectively in DZYX, then state
whether PQ || YX or not. Give reason.

Sample Papers 181


38. In festive season Deepali bought cylindrical jars and cubical boxes to fill her home made

cakes and chocolates. Each jar is 14 cm high and has a diameter of 11 cm.

On the basis of above information answer the following questions.


(i) Find the volume of cake that can be kept in each jar.

(ii) If the volume of each jar is equal to the volume of box, then find the length of edge

of the jar.
(iii) Each cubical box was packed with a wrapping paper. The area of sheet used in

folding was 24 cm2. Find the area of sheet required to pack each box.
OR
If two cubes of edge ‘a’ units are joined end to end, then find the surface area of the
resulting cuboid.

Answers
1. (c)
2. (b)
3. (c)
4. (b)
5. (b)
6. (b)

7. (b)
8. (c)
9. (c)
10. (d)
11. (b) 12. (b)

13. (a)
14. (b)
15. (c)
16. (d)
17. (c) 18. (a)

22. 1 : 1, c – , 0 m OR (– 7, 0).
3
19. (b) 20. (c) 21. a = 3, b = –6
2

1 1 1 1
24. (i) (ii) OR (i) (ii) 26. x = a2, y = b2
2 2 2 3

7 3 3
28. B (5, 0), C e , o
1
27. – OR – 10 31. 14.875 cm2
5 2 2

33. 225 metres OR 10 3 m, 10 m


35. f1 = 28 and f2 = 24 OR x = 8 and y = 7

36. (i) 600


(ii) Yes, it is intersecting lines
(iii) (2, –1) OR 12

37. (i) 25 cm
(ii) 7
(iii) No OR Yes , PQ || YX

38. (i) 1331 cm3(ii) 11 cm


(iii) 750 cm2 OR 10 a2

zzz

182 Mathematics–X
SAMPLE PAPER
12 [UNSOLVED]

Time allowed: 3 hours Maximum marks: 80


General Instructions: Same as CBSE Sample Question Paper–2023 (Solved).

SECTION-A
Section A consists of 20 questions of 1 mark each.
1. Three bulbs red, green and yellow flash at intervals of 80 seconds, 90 seconds and 110

seconds. All three flash together at 8:00 am. At what time will the three bulbs flash
altogether again?

(a) 8:12 am
(b) 9:12 am

(c) 10:12 am
(d) 11:12 am

2. The solution of the equations x + 2y = 1.5 and 2x + y = 1.5 is


(a) x = 1 and y = 1
(b) x = 1.5 and y = 1.5

(c) x = 0.5 and y = 0.5


(d) None of these

3. 3. 27 is

(a) an integer
(b) a rational number

(c) a natural number


(d) an irrational number

4. If the list price of a toy is reduced by ™ 2, a person can buy 2 toys more for ™ 360. The

original price of the toy is


(a) ™18
(b) ™ 20

(c) ™ 19
(d) ™ 21

5. In an AP if a = 1, an = 20 and Sn = 399, then n is


(a) 19
(b) 21

(c) 38

(d) 42

6. The point on the x-axis which is equidistant from (–4, 0) and (10, 0) is

(a) (7, 0)
(b) (5, 0)

(c) (0, 0)
(d) (3, 0)

Sample Papers 183


7. Consider the figure alongside.

A
(7x + 1)°
B
Which of the following statement help proving that
triangle ABO is similar to triangle DOC? 60°
(i) ∠B = 70° and (ii) ∠C = 70°

O
(a) Statement (i) alone is sufficient, but statement (ii)

alone is not sufficient.


(b) Statement (ii) alone is sufficient, but statement (i)

alone is not sufficient.


(c) Each statement alone is sufficient.

(d) Both statement together is sufficient, but neither (3x + 29)°


D
C

statement alone is sufficient. Q


8. In figure, PQ is a chord of a circle and PT is the

tangent at P such that ∠QPT = 60°. Then ∠PRQ is


equal to O
(a) 135°
(b) 150°

R

(c) 120°
(d) 110°

1 + tan2 A P
9. is equal to T
1 + cot2 A

(a) sec2A
(b) –1
(c) cot2 A
(d) tan2 A

10. Which of these is equivalent to p?


Circumference Circumference
(a) (b)
Radius Diameter

(c) Circumference × Diameter


(d) Circumference × Radius

11. An observer 1.6 m tall is 20 3 m away from a tower. The angle of elevation from his

eye to the top of the tower is 30°. The height of the tower is
(a) 21.6 m
(b) 23.2 m
(c) 24.72 m
(d) None

12. In figure, if O is the centre of a circle, PQ is a chord and the


tangent PR at P makes an angle of 50° with PQ, then ∠POQ is


equal to
(a) 100°
(b) 80°

(c) 90°

(d) 75°

13. cos4 A – sin4 A is equal to


(a) 2cos2A + 1
(b) 2cos2A – 1
(c) 2sin2A – 1
(d) 2sin2 A + 1

14. The area of a square inscribed in a circle of diameter p cm is


p p2 p
(a) p2 cm2 (b) cm2 (c) cm2 (d) cm2
4 2 2

15. The length of the longest rod that can be placed in a 12m × 9m × 8m room is

(a) 15 m
(b) 17 m
(c) 21 m
(d) 25 m

184 Mathematics–X
16. The class mark of the class 15.5 – 20.5 is

(a) 15.5
(b) 20.5
(c) 18
(d) 5

17. If a letter of English alphabet is chosen at random, than the probability that the letter

is a consonant is
21 10 21 5
(a) (b) (c) (d)
23 13 26 26

18. The volume of the largest right circular cone that can be cut out from a cube of edge

4.2 cm is
(a) 9.7 cm3
(b) 77.6 cm3
(c) 58.2 cm3
(d) 19.4 cm3

DIRECTION: In the question numbers 19 and 20, a statement of assertion (A) is followed by a
statement of Reason (R). Choose the correct option.
(a) Both A and R are true and R is the correct explanation for A.

(b) Both A and R are true and R is not the correct explanation for A.

(c) A is true but R is false.


(d) A is false but R is true.


19. Assertion (A) : sin2 67° + cos2 67° = 1


Reason (R) : For any value of θ, sin2 θ + cos2 θ = 1.


20. Assertion (A) : In a circle of radius 6 cm, the angle of a sector is 60°. Then the area of

6
the sector is 18 cm2.
7
Reason (R) : Area of the circle with radius r is pr2.

SECTION-B
Section B consists of 5 questions of 2 marks each.
a1 b1 c1
21. On comparing the coefficients a , , c , find out whether the following pair of linear
2 b

2 2
equations is consistent or inconsistent.
4
x + 2y = 8
3

2x + 8y = 12
OR
Find the solution of the pair of equations:
3 8 1 2

x + y = – 1 , x – y = 2 , x, y ! 0
22. The mid point of the line segment joining A (2a, 4) and B(–2, 3b) is (1, 2a+1). Find the

values of a and b.
23. ∆ABC ~ ∆DEF. If AB = 4 cm, BC = 3.5 cm, CA = 2.5 cm and DF = 7.5 cm, then find

perimeter of ∆DEF.
24. In figure, there are two concentric circles with centre O and

of radii 5 cm and 3 cm. From an external point P, tangents


PA and PB are drawn to these circles. If AP = 12 cm, find the
length of BP.

Sample Papers 185


OR
In figure, BOA is a diameter of a circle with centre O and the tangent at a point P meets
BA extended at T. If ∠ABP = 40°, then find the value of ∠PTA.

25. A bag contains lemon flavoured candies only. Malini takes out one candy without looking

into the bag. What is the probability that she takes out
(i) an orange flavoured candy?
(ii) a lemon flavoured candy?

SECTION-C
Section C consists of 6 questions of 3 marks each.

26. A part of monthly hostel charges is fixed and the remaining depends on the number of

days one has taken food in the mess. When a student A takes food for 20 days, she has to
pay `1000 as hostel charges whereas a student B, who takes food for 26 days, pays `1180
as hostel charges. Find the fixed charges and the cost of food per day.
27. Find the 31st term of an AP whose 11th term is 38 and the 16th term is 73.

OR
In an AP, given l = 28, S = 144 and there are total 9 terms. Find a.
28. If Q(0, 1) is equidistant from P(5, –3) and R(x, 6) find the value of x. Also, find the

distances of QR and PR.


29. In figure, ABC is a right triangle, right angled at B such that BC = 6 cm and AB = 8 cm.

Find the radius of its incircle.

30. If x sin3
q + y cos3 q = sin q cos q and x sin q = y cos , prove x2 + y2 = 1.
q

OR
If tan q + sin q = m and tan q – sin q = n, show that (m2 – n2) = 4 mn .
31. The table below shows the daily expenditure on food of 25 households in a locality.

Daily Expenditure (in `) 100–150 150–200 200–250 250–300 300–350

No. of households 4 5 12 2 2

Find the mean daily expenditure on food by a suitable method.

186 Mathematics–X
SECTION-D
Section D consists of 4 questions of 5 marks each.
32. Solve the following quadratic equation:

9x2 – 9 (a + b) x + [2a2 + 5ab + 2b2] = 0


33. A straight highway leads to the foot of a tower. A man standing at the top of the tower

observes a car at an angle of depression of 30°, which is approaching the foot of the
tower with a uniform speed. Six seconds later, the angle of depression of the car is found
to be 60°. Find the time taken by the car to reach the foot of the tower from this point.
OR
A person standing on the bank of a river observes that the angle of elevation of the top
of a tree standing on the opposite bank is 60°. When he moves 40 metres away from the
bank, he finds the angle of elevation to be 30°. Find the height of the tree and the width
of the river.
34. CD and GH are respectively the bisectors of ∠ACB and ∠EGF such that D and H lie on

sides AB and FE of ∆ABC and ∆EFG. If ∆ABC ~ ∆FEG, show that


CD AC
(i) = (ii) ∆DCB ~ ∆HGE (iii) ∆DCA ~ ∆HGF
GH FG

35. A gulab jamun, contains sugar syrup about 30% of its volume. Find

approximately how much syrup would be found in 45 gulab jamun, each


shaped like a cylinder with two hemispherical ends with length 5 cm and
diameter 2.8 cm.
OR
A solid toy is in the form of a hemisphere surmounted by a right circular cone. The height
of the cone is 2 cm and the diameter of the base is 4 cm. Determine the volume of the toy.
If a right circular cylinder circumscribes the toy, find the difference of the volumes of the
cylinder and the toy. (Take p = 3.14)

SECTION-E
Case study based questions are compulsory.
36. A Mathematics Exhibition is being conducted in your School and one of your friends

is making a model of a factor tree. He has some difficulty and asks for your help in
completing a quiz for the audience.
Observe the following factor tree and answer the following:
x

5 2783

y 253

11 z

Sample Papers 187


Based on the above information, answer the following questions:
(i) What will be the value of x?

(ii) What will be the value of y?


(iii) What will be the value of z?


OR
Find the prime factorisation of 13915.
37. An asana is a body posture, originally and still a general term for a sitting meditation pose,

and later extended in hatha yoga and modern yoga as exercise, to any type of pose or
position, adding reclining, standing, inverted, twisting, and balancing poses. In the figure,
one can observe that poses can be related to representation of quadratic polynomial.

Y
2
1
–3 –2 –1 1 2 3 4 5

X' O X
–1
–2
–3

–4
–5

–6

–7
–8
Y'

Based on the above information, answer the following questions:


(i) If the graph of parabola x2 = 4ay opens downward than what should be the value

of a.
(ii) In the graph, how many zeros are there for the polynomial?

(iii) Write the two zeroes in the above shown graph.


OR
Find the zeros of the quadratic polynomial 4 3 x2 + 5x – 2 3 .

188 Mathematics–X
38. On a weekend Rani was playing cards with her family .The deck has 52 cards. If her

brother drew one card.

Based on the above information, answer the following questions:


(i) What is the probability of getting a King of red colour?

(ii) What is the probability of getting a face card?


(iii) What is the probability of getting a jack of hearts?


OR
What is the probability of getting a spade?

Answers
1. (c)
2. (c)
3. (b)
4. (b)
5. (c)
6. (d)

7. (c)
8. (c)
9. (d)
10. (b)
11. (a)
12. (a)

13. (b)
14. (c) 15. (b)
16. (c)
17. (c)
18. (d)

19. (a)
20. (b) 21. Consistent with infinitely many solutions. OR x= 1, y = –2

22. a = 2, b = 2
23. 30 cm
24. 4 10 cm OR 10°

25. (i) 0 (ii) 1


26. Fixed charge = `400 and cost of food per day = `30

27. 178 OR a = 4
28. x = ±4 ,QR =
41 , PR = 82 , 9 2 29. 2 cm

2a + b a + 2b
31. `211 32. x =and x =
3 3

33. 3 seconds OR Height of the tree = 34.64 m; Width of the river = 20 m


3 3
35. 338 cm (approx.) OR 25.12 cm

36. (i) 13915


(ii) 11
(iii) 23 OR 5 × 112 × 23

2 3
37. (i) a < 0 (ii) 2 (iii) –2 and 4 OR – ,
3 4

1 3 1 1
38. (i) (ii) (iii) OR
26 13 52 4

zzz

Sample Papers 189


SAMPLE PAPER
[UNSOLVED] 13
1
Time allowed: 3 hours Maximum marks: 80
General Instructions: Same as CBSE Sample Question Paper–2023 (Solved).

SECTION-A
Section A consists of 20 questions of 1 mark each.

1. If 3 is the least prime factor of number a and 7 is the least prime factor of number b,

then the least prime factor of (a + b) is


(a) 2
(b) 3
(c) 5
(d) 10

2. If two positive integers a and b are written as a = x3y2 and b = xy3; x, y are prime

numbers, then LCM (a, b) is


(a) xy
(b) xy2
(c) x3y3

(d) x2y2

3. The value of k for which the lines (k + 1) x + 3ky + 15 = 0 and 5x + ky + 5 = 0 are


coincident is
(a) 14
(b) 2
(c) –14
(d) –2

4. Value(s) of k for which the quadratic equation 2x2 – kx + k = 0 has equal roots is

(a) 0
(b) 4
(c) 8
(d) 0 and 8

5. Which term of the AP 21, 42, 63, 84, ... is 210?


(a) 9th
(b) 10th
(c) 11th
(d) 12th

6. If (a, b) is the mid-point of the line segment joining the points A(10, –6) and B(k, 4) and

a – 2b = 18, the value of k is


(a) 30
(b) 22
(c) 4
(d) 40

7. Observe the right triangle ABC, right angled at A


B
as shown on the figure. If BP ⊥ AC, then which of
the following is not correct?
(a) ΔAPB ~ ΔABC (b) ΔAPB ~ ΔBPC
2
(c) BC = CP . AC
(d) AC2 = AB . CB

A P C

190 Mathematics–X
8. At point A on a diameter AB of a circle of radius 10 cm, tangent XAY is drawn to the

circle. The length of the chord CD parallel to XY at a distance 16 cm from A is


(a) 8 cm
(b) 10 cm
(c) 16 cm
(d) 18 cm

9. Consider the triangle shown below.


X 15 Y
θ

8
17

What are the values of tan q, cosec q and sec q?


8 17 17 8 17 17
(a) tan i = , cosec i = , sec i = (b) tan i = , cosec i = , sec i =
15 8 15 15 15 8

17 8 17 8 17 8
(c) tan i = , cosec i = , sec i = (d) tan i = , cosec i = , sec i =
15 15 8 15 15 17

10. If angle between two radii of a circle is 125°, then the angle between the tangents at the

ends of the radii is


(a) 90°
(b) 75° (c) 55°
(d) 125°

a
11. Given that sin i = , then tan i is equal to
b

b b2 – a2 a b2 – a2
(a) (b) (c) (d)
b2 – a2 b b2 – a2 a

12. If the angle of elevation of top of a tower from a point on the ground which is 20 3 m

away from the foot of the tower is 30°, then the height of the tower is
(a) 60 m
(b) 30 m
(c) 25 m
(d) 20 m

13. Observe the figure alongside:


R
What is the area of the segment PQR, if the radius of the circle
22
is 7 cm? e Use r = o Q
7

O
2 2
(a) 14 cm
(b) 17.3 cm

(c) 28 cm2
(d) 91 cm2

P
14. If the height of right circular cylinder is equal to the diameter

of the base, then its whole surface area is equal to


4 2 2 2 3 2
(a) rh (b) rh (c) rh (d) 4πh2
3 3 2

15. A circle with centre O of diameter 28 cm and a chord BC of length



C

14 cm is shown in figure:
What is the length of the major arc of the circle, to the nearest tenth?
14 cm

O
(a) 14.7 cm
(b) 73.3 cm

(c) 146.7 cm
(d) 216.3 cm

Sample Papers 191


16. A cylinder, a cone and a hemisphere are of the same base and of the same height. The

ratio of their volumes is


(a) 1 : 2 : 3
(b) 2 : 1 : 3
(c) 3 : 1 : 2
(d) 3 : 2 : 1

17. If the mean of data is 27 and mode is 45, the median is


(a) 30
(b) 27
(c) 32 (d) 33

18. If a number x is chosen from the numbers 1, 2, 3 and a number y is selected from the

numbers 1, 4, 9. Then P(xy < 9) is


3 4 1 5
(a) (b) (c) (d)
9 9 9 9

DIRECTION: In the question numbers 19 and 20, a statement of assertion (A) is followed by a
statement of Reason (R). Choose the correct option.
(a) Both A and R are true and R is the correct explanation for A.

(b) Both A and R are true and R is not the correct explanation for A.

(c) A is true but R is false.


(d) A is false but R is true.


3 3
19. Assertion (A) : In a right angled triangle, if tan = then sin i = .
4 5
θ

1
Reason (R) : sin 60° =
2

20. Assertion (A) : If the circumference of two circles are in the ratio 2 : 3, then ratio of

their areas is 4 : 9.
Reason (R) : The circumference of a circle of radius r is 2pr and its area is pr2.

SECTION-B
Section B consists of 5 questions of 2 marks each.
21. For what value of k, will the following pair of equations have infinitely many solutions?

2x + 3y = 7 and (k + 2) x – 3 (1 – k) y = 5k + 1
OR
Solve the following pair of linear equations as:
3x – 5y = 4
2y + 7 = 9x

22. The x-coordinate of a point P is twice its y-coordinate. If P is equidistant from Q(2, –5)

and R(–3, 6), find the coordinates of P.


OA OD
23. In figure, = . Prove that ∠A = ∠C and ∠B = ∠D.
OC OB

192 Mathematics–X
24. If the angle between two tangents drawn from an external point P to a circle of radius a

and centre O, is 60°, then find the length of OP. C


OR
In figure, ABC is a triangle in which ∠B = 90°, BC = 48 cm and 48 cm
AB = 14 cm. A circle is inscribed in the triangle, whose centre is
r r
O. Find radius r of in-circle. O
25. A card is drawn at random from a pack of 52 playing cards. Find B A
14 cm

the probability that the card drawn is neither a red card nor a
black king.

SECTION-C
Section C consists of 6 questions of 3 marks each.

26. Ankita travels 14 km to her home partly by rickshaw and partly by bus. She takes half an

hour if she travels 2 km by rickshaw, and the remaining distance by bus. On the other
hand, if she travels 4 km by rickshaw and the remaining distance by bus, she takes 9
minutes longer. Find the speed of the rickshaw and of the bus.
27. Which term of the arithmetic progression 3, 15, 27, 39 .... will be 120 more than its 21st

term?
OR
The sum of the 5th and the 9th terms of an AP is 30. If its 25th term is three times its 8th
term, find the AP.
28. Find the ratio in which the line x – 3y = 0 divides the line segment joining the points

(– 2, – 5) and (6, 3). Find the coordinates of the point of intersection.


29. If tangents PA and PB from a point P to a circle with centre O are inclined to each other

at angle of 80°, then find ∠POA.

30. Prove that: (sinq + 1 + cos q)(sin q – 1 + cos q). sec q cosec q = 2

OR
tan A tan A
Prove that: – = 2 cosec A
1 + sec A 1 – sec A

31. Find the value of p, if the mean of the following distribution is 20.

x 15 17 19 20+p 23
f 2 3 4 5 6

SECTION-D
Section D consists of 4 questions of 5 marks each.
32. A motorboat whose speed is 18 km/h in still water takes 1 h 30 minutes more to go 36 km

upstream than to return downstream to the same spot. Find the speed of the stream.

Sample Papers 193


33. Two poles of equal heights are standing opposite to each other on either side of the road,

which is 80 m wide. From a point between them on the road, the angles of elevation of the
top of the poles are 60° and 30°, respectively. Find the height of the poles and the distances
of the point from the poles.

OR
The angle of elevation of the top of a building from the foot of the tower is 30° and the
angle of elevation of the top of the tower from the foot of the building is 60°. If the tower
is 50 m high, find the height of the building.
34. In figure, DEFG is a square and ∠BAC = 90°. Prove that:

(i) ∆AGF ~ ∆DBG


(ii) ∆AGF ~ ∆EFC
(iii) ∆DBG ~ ∆EFC
(iv) DE2 = BD × EC

35. A right cylindrical container of radius 6 cm and height 15 cm is full of ice-cream, which

has to be distributed to 10 children in equal cones having hemispherical shape on the


top. If the height of the conical portion is four times its base radius, find the radius of
the ice-cream cone. 4.2 cm
OR
In figure, a decorative block is shown which is made of two
solids, a cube and a hemisphere. The base of the block is a
cube with edge 6 cm and the hemisphere fixed on the top has
a diameter of 4.2 cm. Find 6 cm

(a) the total surface area of the block. 6 cm


22

(b) the volume of the block formed. (Take r = ) 6 cm


7

SECTION-E
Case study based questions are compulsory.
36. To enhance the reading skills of grade X students, the school nominates you and two of

your friends to set up a class library. There are two sections- section A and section B of
grade X. There are 32 students in section A and 36 students in section B.

194 Mathematics–X
Based on the above information answer the following questions.
(i) What is the minimum number of books you will require for the class library, so that

they can be distributed equally among students of Section A or Section B?


(ii) If the product of two positive integers is equal to the product of their HCF and

LCM is true then, write HCF (32, 36).


(iii) Express 36 as a product of its primes.

OR
If p and q are positive integers such that p = ab2 and q = a2b, where a, b are prime
numbers, then write the LCM (p, q).
37. The wall of room is decorated with beautiful garlands, each garland forming a parabola.

X' X
O

Y'
Based on the above information answer the following questions.
(i) What type of polynomial does a parabola represent?

(ii) Write a quadratic polynomial with the sum and product of its zeroes as 1 and –2 .

(iii) If one of the zeroes of the quadratic polynomial (k – 2) x2 – 2x – 5 is –1, then find

the value of k.
OR
If a, b are the zeros of the polynomial f (x) = x2 – 7x + 12 then find the value of
1 1

a+b.
38. Rahul and Ravi planned to play Business ( board game) in which they were supposed to

use two dice.

Based on the above information, answer the following questions:


(i) Ravi got first chance to roll the dice. What is the probability that he got the sum of

the two numbers appearing on the top face of the dice as 8?

Sample Papers 195


(ii) Rahul got next chance. What is the probability that he got the sum of the two

numbers appearing on the top face of the dice as 13?

(iii) Now it was Ravi’s turn. He rolled the dice. What is the probability that he got the sum

of the two numbers appearing on the top face of the dice less than or equal to 12?
OR
Rahul got next chance. What is the probability that he got the sum of the two
numbers appearing on the top face of the dice equal to 7?

Answers
1. (a)
2. (c)
3. (a)
4. (d)
5. (b)
6. (b)

7. (d)
8. (c)
9. (a)
10. (c)
11. (c)
12. (d)

13. (a)
14. (c)
15. (b)
16. (c)
17. (d)
18. (d)

9 –5
19. (c) 20. (a)21. k = 4 OR x = , y = 22. (16, 8)
13 13

6
24. 2a OR r = 6 cm 25. 26. 10 km/h, 40 km/h
13

27. 31st term OR 3, 5, 7, 9.... 28. 13 : 3, c , m 29. ∠POA = 50°


9 3
31. p = 1
2 2

2
32. 6 km/h
33. 20 3 ; 20 m, 60 m OR 16
m 35. 3 cm OR 235.40 cm3

3
36. (i) 288
(ii) 4 (iii) 22 × 32 OR a2 b2

7
37. (i) quadratic polynomial (ii) x2 + x – 2 (iii) k = 5 OR
12

5 1
38. (i) (ii) 0 (iii) 1 OR
36 6

zzz

196 Mathematics–X
SAMPLE PAPER
14 [UNSOLVED]

Time allowed: 3 hours Maximum marks: 80


General Instructions: Same as CBSE Sample Question Paper–2023 (Solved).

SECTION-A
Section A consists of 20 questions of 1 mark each.
1. HCF of 96 and 404 is

(a) 4
(b) 2
(c) 3
(d) 101

2. If the HCF of 65 and 117 is expressible in the form 65m – 117, then the value of m is

(a) 4
(b) 2
(c) 11
(d) 3

3. If x = a, y = b is the solution of the equations x – y = 2 and x + y = 4, then the values


of a and b are, respectively


(a) 3 and 5
(b) 5 and 3
(c) 3 and 1
(d) –1 and –3

4. Which of the following is a quadratic equation?


(a) (4 – x) (3x + 1) = 2 – 3x2


(b) (x + 3) 2 – 5 = x2 + 9

(c) (k – 4) x2 – 3x = 9k – 4 , (k = 4)
(d) (x + 1)3 = x3 – 5

2
5. If the sum of n terms of an AP is 2n + 5n, then its nth term is

(a) 4n – 3
(b) 3n – 4
(c) 4n + 3
(d) 3n + 4

6. Two poles are to be installed on an elevated road as shown in the diagram. The diagram

also shows the starting and ending points of the road.


(8, 8) Q R (14, 11)

Which of the following are the coordinates of the poles?


(a) Q (10, 9) and R (12, 8)
(b) Q (10, 9) and R (12, 10)

(c) Q (10, 8) and R (12, 11)


(d) Q (–10, 9) and R (0, 11)

Sample Papers 197


7. In the given figure, QR || AB, RP || BD, CQ = x + 2, QA = x,

A
x
CP = 5x + 4, PD = 3x. Q
x+
2
The value of x is _____________.
B C
R
(a) 1 (b) 6
+4

5x
(c) 3 (d) 9

P
5x
D
8. In Fig., if PA and PB are tangents to the circle with centre O

such that ∠APB = 50°, then ∠PAB is equal to A


(a) 35°

(b) 65°

O P

(c) 40°

(d) 70°
B

9. (cosec θ – sin θ) (sec θ – cos θ) (tan θ + cot θ) is equal to


(a) 0 (b) 1
(c) –1
(d) none of these

10. If radii of two concentric circles are 4 cm and 5 cm, then the length of each chord of

one circle which is tangent to the other circle is


(a) 3 cm
(b) 6 cm
(c) 9 cm

(d) 1 cm

4 2
11. sec A – sec A is equal to

(a) tan2A – tan4A


(b) tan4A – tan2A
(c) tan4A + tan2A
(d) None of these

12. To form a circle of radius r, four minor sectors of equal measure are joined. Which of
..
.. ....... ... ...

.............
these options completes the sentence below? ............................
..........................
............................
............................
............................
............................
............................ ............................
............................
............................
............................
............................

. . .. . .
............................
............................
The sum of the area of the four minor sectors is equal to the ............................

.....r..........o
............................
............................
............................
............................
............................
............................
............................

............................
............................
............................
............................
___________. ............................
............................
............................
............................
............................
r
(a) area of the semicircle of diameter 2r.

(b) area of the circle of diameter 2r.


(c) circumference of the circle of radius r.


(d) circumference of the circle of diameter r.


13. The length of the ladder making an angle of 45° with a wall and whose foot is 7 m away

from the wall is


7 2
(a) m (b) 7 2 m (c) 14 2 m (d) 14 m
2

/ fi di
14. In the formula x = a + for finding the mean of a grouped data, di's are deviations
/ fi

from a of
(a) lower limits of the classes
(b) upper limits of the classes

(c) mid-points of the classes


(d) frequencies of the class marks

198 Mathematics–X
15. Which of these is equivalent to the sum of the lengths of arc corresponding to the

minor and major segment of a circle of radius 12 cm?


(a) 24p cm
(b) 48p cm
(c) 12p cm
(d) 144p cm

16. Kirti has a box containing four cards labelled A, B, C and D. She randomly picks a

card from the box, records the label on the card and put it back in the box. She repeats
this experiment 80 times and records her observation in the table shown below.

Card A Card B Card C Card D


11 16 25 28

Which of the following shows the empirical probability and theoretical probability of
picking Card C the next time?
5 5
(a) Empirical probability = (b) Empirical probability =
11 11

1 1
Theoretical probability = Theoretical probability =
2 4

5 5
(c) Empirical probability = (d) Empirical probability =
16 16

1 1
Theoretical probability = Theoretical probability =
2 4

17. A piece of paper is in the shape of a semi circular region of radius 10 cm. It is rolled

to form a right circular cone. The slant height is


(a) 5 cm
(b) 10 cm
(c) 15 cm
(d) 20 cm

18. A cylinder and a cone are of the same base radius and same height. Find the ratio of

the volumes of the cylinder to that of the cone.


(a) 1 : 3
(b) 1 : 2
(c) 3 : 1
(d) 2 : 1

DIRECTION: In the question numbers 19 and 20, a statement of assertion (A) is followed by a
statement of Reason (R). Choose the correct option.
(a) Both A and R are true and R is the correct explanation for A.

(b) Both A and R are true and R is not the correct explanation for A.

(c) A is true but R is false.


(d) A is false but R is true.


1 3
19. Assertion (A) : In a right angled triangle, if cos i = and sin i = , then tan θ = 3.
2 2

sin i
Reason (R) : tan i =
cos i

20. Assertion (A) : The length of the minute hand of a clock is 7 cm. Then the area swept

5
by the minute hand in 5 minutes is 12 cm2.
6
Reason (R) : The length of an arc of a sector of angle θ and radius r is given by

i
l= × 2 rr .
360

Sample Papers 199


SECTION-B
Section B consists of 5 questions of 2 marks each.
21. Find the relation between p and q if x = 3 and y = 1 is the solution of the pair of equations

x – 4y + p = 0 and 2x + y – q – 2 = 0.
OR
For which value(s) of k will the pair of equations kx + 3y = k – 3, 12x + ky = k have no
solution?
x y +1 o
22. If the mid-point of a segment joining A e , and B(x + 1, y – 3) is C(5, –2), find x, y.
2 2

23. In the figure given below, GCBD and GEBF. If AC = 3 cm and CD = 7 cm, then find

AE
the value of .
AF D
m
7c
C
m
3c
A B
G

24. In figure, CP and CQ are tangents from an external


point C to a circle with centre O. AB is another tangent


which touches the circle at R. If CP = 11 cm and
BR = 4 cm, find the length of BC.

OR
In figure, BA and BC are tangents to the circle drawn from
an external point B. PQ is a third tangent touching the circle
at R. If BC = 12 cm and PR = 3 cm, what is the perimeter of
DBPQ?
25. A card is drawn at random from a pack of 52 playing cards. Find the probability of

drawing a card which is neither a spade nor a king.

SECTION-C
Section C consists of 6 questions of 3 marks each.
26. A part of monthly hostel charges in a college hostel are fixed and the remaining depends

on the number of days one has taken food in the mess. When a student A takes food for
25 days, he has to pay ™ 4,500, whereas a student B who takes food for 30 days, has to
pay ™ 5,200. Find the fixed charges per month and the cost of food per day.

200 Mathematics–X
1 1 3
27. Which term of the progression 20, 19 , 18 , 17 , ... is the first negative term?
4 2 4

OR
If the sum of first 7 terms of an AP is 49 and that of 17 terms is 289, find the sum of first
n terms.
28. The points A(4, –2), B(7, 2), C(0, 9) and D(–3, 5) form a parallelogram. Find the length

of the altitude of the parallelogram on the base AB.


29. Find the actual lengths of sides of DOTP.

3
30. Find A and B if sin (A + 2B) = and cos(A + 4B) = 0, where A and B are acute angles.
2

OR
1 1 – cos2 i 3
If cot =
, show that = .
2 – sin2 i 5
q

3
31. The following data gives the information on the observed lifetimes (in hours) of 225

electrical components:
Lifetimes (in hours) 0–20 20–40 40–60 60–80 80–100 100–120
No. of components 10 35 52 61 38 29
Determine the modal lifetimes of the components.

SECTION-D
Section D consists of 4 questions of 5 marks each.
1
32. Two taps running together can fill a tank in 3 hours. If one tap takes 3 hours more
13

than the other to fill the tank, then how much time will each tap take to fill the tank?
33. In the figure given below, OB is the perpendicular bisector of the line segment DE,

1 1 2
FA ⊥ OB and FE intersects OB at the point C. Prove that: + = .
OA OB OC

Sample Papers 201


34. From the top of a hill, the angles of depression of two consecutive kilometre stones due

east are found to be 45° and 30° respectively. Find the height of the hill.
OR
The angle of elevation of a aeroplane from a point A on the ground is 60°. After a flight of
30 seconds, the angle of elevation changes to 30°. If the aeroplane is flying at a constant
height of 3600 3 metres then find the speed of the aeroplane.
35. A solid iron pole consists of a cylinder of height 220 cm and base diameter 24 cm, which

is surmounted by another cylinder of height 60 cm and radius 8 cm. Find the mass of
the pole, given that 1 cm3 of iron has approximately 8 gm mass. [Use π = 3.14]
OR
A well of diameter 4 m is dug 14 m deep. The earth taken out is spread evenly all around
the well to form a 40 cm high embankment. Find the width of the embankment.

SECTION-E
Case study based questions are compulsory.
36. A seminar is being conducted by an Educational Organisation, where the participants


will be educators of different subjects. The number of participants in Hindi, English and
Mathematics are 60, 84 and 108 respectively.

Based on of above information answer the following questions.


(i) If in each room the same number of participants are to be seated and all of them

being in the same subject, then find the maximum number of participants that can
be accommodated in each room.
(ii) What is the maximum number of rooms required during the event?

OR
Find the LCM of 60, 84 and 108.
(iii) Write the product of HCF and LCM of 60, 84 and 108.

202 Mathematics–X
37. Water flowing in a fountain follows trajectory as shown below:

X' X
O

Y'

Based on the above information answer the following questions.


(i) If the trajectory is represented by x2 – 3x – 18, then write its zeros.
1
(ii) If – is one of the zeros of 9x2 – kx – 5, then find the value of k.
3

OR
If a and b are the zeros of the polynomial 2x2 – 3x – 5 then what is the value of a + b.
(iii) If one zero of the polynomial p(x) = 3x2 – 10x – m reciprocal of other, then write the

value of m.
38. Akriti and Sukriti have to start the game of ludo. They are fighting for who will start the

game. They found two coins and decided to toss them simultaneously to know who will
start the game.

Based on above information answer the following questions.


(i) How many possible outcomes are there?

(ii) Akriti says if I get atleast one head, I will win and start the game. What is the

probability that Akriti will start the game?


OR
Sukriti says if I get at most one tail, I will start the game. What is the probability that
Sukriti will start the game?
(iii) If the probability of success is 73%. Then what is the probability of failure?

Sample Papers 203


Answers
1. (a)
2. (b) 3. (c)
4. (d)
5. (c)
6. (b)

7. (a)
8. (b) 9. (b)
10. (b)
11. (c)
12. (b)

13. (b)
14. (c) 15. (a)
16. (d)
17. (b)
18. (c)

19. (a)
20. (b) 21. q = 5p OR k = – 6
22. x = 6, y = – 1

9
23. 3 : 10 24. 7 cm OR 24 cm 25.
13

26. Fixed charge = ™ 1,000; Cost of food per day = ™ 140


27. 28th term OR n2

28. 9.8 units 29. 6 cm, 8 cm, 10 cm


30. A = 30°, B = 15°

31. 65.63 hours


32. 5 hours; 8 hours

34. 500 ( 3 + 1) m OR 864 km/h


35. 892.262 kg OR 10 m

36. (i) 12
(ii) 21 OR 3780
(iii) 45360
3
37. (i) 6, –3 (ii) 12 OR (iii) 3
2

3 3
38. (i) 4 (ii) OR (iii) 27% = 0.27
4 4

zzz

204 Mathematics–X
SAMPLE PAPER
15 [UNSOLVED]

Time allowed: 3 hours Maximum marks: 80


General Instructions: Same as CBSE Sample Question Paper–2023 (Solved).

SECTION-A
Section A consists of 20 questions of 1 mark each.
1. The largest number which divides 615 and 963 leaving remainder 6 in each case is

(a) 82
(b) 95
(c) 87
(d) 93

2. The value of k for which the pair of equation kx – y = 2 and 6x – 2y = 3 has unique

solution
(a) k = 3
(b) k ≠ 3
(c) k ≠ 0
(d) k = 0

3. The exponent of 2 in prime factorisation of 144 is


(a) 4
(b) 5 (c) 6 (d) 3

4. lf the point P(k, 0) divides the line segment joining the points A(2, – 2) and B(– 7, 4) in

the ratio 1 : 2, then the value of k is


(a) 1
(b) 2 (c) – 2
(d) – 1

5. In the given figure, QR || AB, RP || BD, CQ = x + 2, QA = x, CP = 5x + 4, PD = 3x.


A
x
Q
x+
2

B C
R

+4
5x
P
5x
D

The value of x is _____________.


(a) 1
(b) 6 (c) 3 (d) 9

Sample Papers 205


1 + cos i
6. The value of is
1 – cos i

(a) cot i – cosec i


(b) cosec i + cot i

(c) cosec2 i + cot2 i


(d) (cot i + cosec i) 2

7. If x = a cos q and y = b sin q, then the value of b x + a2y2 is



2 2

a2
(a) a2 + b2 (b) (c) a2b2 (d) None of these
b2

8. The ratio of outer and inner perimeters of circular path is 23:22. If the path is 5 m

wide, the inner circle is


(a) 55 m
(b) 110 m
(c) 220 m
(d) 230 m

9. A fountain is enclosed by a circular fence of circumference 11 m and is surrounded


by a circular path. The circumference of the outer boundary of the path is 16 m. A


gardener increased the width of the pathway by decreasing the area enclosed by the
fence such that the length of the fence is decreased by 3 m. The path is to be covered
by the bricks which cost ™125 per m2. What will be the total cost, to the nearest whole
22
number, required to cover the area by the bricks? e Use r = o
7
(a) ™ 1,910
(b) ™ 9,878
(c) ™ 39,772
(d) ™ 79,545

10. Smita has a bag containing 1 red, 1 green, 1 yellow, 1 black and 1 blue ball. She

randomly picks the ball from the bag notes it colour and keeps it back in the bag. She
repeats this 40 times. The table shows the number of times each colour ball she gets.
The number of times the black ball is picked is missing in the table.

Red ball Green ball Yellow ball Black ball Blue ball
10 6 5 ? 10
She then repeats the experiment 10 more times and gets red ball twice, green ball
once, yellow ball thrice, black ball once and blue ball thrice.
Which of these is a valid conclusion as the number of trials of the experiment increases?
(a) The empirical probability of picking red ball becomes equal to its theoretical

probability.
(b) The empirical probability of picking red ball does not get closer to its theoretical

probability.
(c) The empirical probability of picking yellow ball gets closer to its theoretical

probability.
(d) The empirical probability of picking yellow ball gets further away from its theoretical

probability.
11. Which of the following is not a quadratic equation?

(a) 2(x – 1)2 = 4x2 – 2x + 1 (b) 2x – x2 = x2 + 5


(c) ^ 2 x + 3 h + x2 = 3x2 – 5x

2
(d) (x2 + 2x)2 = x4 + 3 + 4x3

206 Mathematics–X
12. The 11th and 13th terms of an AP are 35 and 41 respectively, its common difference

is
(a) 38
(b) 32
(c) 6
(d) 3

13. Two circles touch each other externally at C and AB is common tangent of circles, then

∠ACB is

A B

(a) 70°
(b) 60°
(c) 100°
(d) 90°

14. Twelve solid spheres of the same size are made by melting a solid metallic cylinder of

base diameter 2 cm and height 16 cm. The diameter of each sphere is


(a) 4 cm
(b) 3 cm
(c) 2 cm
(d) 6 cm

15. A tangent is drawn from a point at a distance of 17 cm of circle (O, r) of radius 8 cm.

The length of tangent is


(a) 5 cm
(b) 9 cm
(c) 15 cm
(d) 23 cm

16. The radius of the largest right circular cone that can be cut out from a cube of edge

4.2 cm is
(a) 2.1 cm
(b) 4.2 cm
(c) 3.1 cm
(d) 2.2 cm

17. A 1.6 m tall girl stands at distance of 3.2 m from a lamp post and casts shadow of

4.8 m on the ground, then the height of the lamp post is


8
(a) 8 m (b) 4 m (c) 6 m (d) m
3

18. The runs scored by a batsman in 35 different matches are given below:

Runs Scored 0–15 15–30 30–45 45–60 60–75 75–90


Frequency 5 7 4 8 8 3
Number of matches in which the batsman scored less than 60 runs are
(a) 16
(b) 24
(c) 8
(d) 19

DIRECTION: In the question numbers 19 and 20, a statement of assertion (A) is followed by a
statement of Reason (R). Choose the correct option.
(a) Both A and R are true and R is the correct explanation for A.

(b) Both A and R are true and R is not the correct explanation for A.

(c) A is true but R is false.


(d) A is false but R is true.


19. Assertion (A) : If cos A + cos2 A = 1 then sin2 A + sin4 A = 2.


Reason (R) : 1 – sin2 A = cos2 A, for any value of A.


Sample Papers 207


20. Assertion (A) : A bicycle wheel makes 5000 revolutions in covering 11 km. Then

diameter of the wheel is 35 cm.


i 1
Reason (R) : Area of segment of a circle is ×rr2 – r2 sin i .
360 2

SECTION-B
Section B consists of 5 questions of 2 marks each.
21. If the system of equations 2x + 3y = 7 and (a + b)x + (2a – b)y = 21 has infinitely many

solutions, then find a and b.


OR
Sumit is 3 times as old as his son. Five years later, he shall be two and a half time as old
as his son. How old is Sumit at present?
22. Write the coordinates of a point on x-axis which is equidistant from the points (–3, 4)

and (2, 5).


23. X is a point on the side BC of DABC. XM and XN are drawn parallel to AB and AC

respectively meeting AB in N and AC in M. MN produced meets CB produced at T.


Prove that TX2 = TB × TC.
24. Find the length of the tangent from an external point P at a distance of 20 cm from the

centre of a circle of radius 12 cm.


OR
Two concentric circles are of radii 8 cm and 5 cm. Find the length of the chord of the
larger circle which touches the smaller circle.
25. The probability of selecting a blue marble at random from a jar that contains only blue,

1
black and green marbles is . The probability of selecting a black marble at random
5
1
from the same jar is . If the jar contains 11 green marbles, find the total number of
4
marbles in the jar.

SECTION-C
Section C consists of 6 questions of 3 marks each.
26. Two numbers are in the ratio of 1 : 3. If 5 is added to both the numbers, the ratio

becomes 1 : 2. Find the numbers.


27. How many terms of the AP 9, 17, 25, ... must be taken to give a sum of 636?

OR
How many terms of the series 54, 51, 48, ... be taken so that, their sum is 513? Explain
the double answer.
28. Find the coordinates of the points which divide the line segment joining A (–2, 2) and

B (2, 8) into four equal parts.

208 Mathematics–X
29. In the given figure, AB is a chord of length 8 cm of a circle of radius 5 cm. The tangents

to the circle at A and B intersect at P. Find the length of AP.


A

5 cm

O 8 cm P

15 (2 + 2 sin i) (1 – sin i)
30. If cot q =
]1 + cos ig (2 – 2 cos i)
, then evaluate .
8

OR
a a sin i – b cos i a2 – b2
If tan q = , prove that = 2 .
b a sin i + b cos i a + b2

31. If the mean of the following frequency distribution is 62.8, then find the missing

frequency x:
Class 0–20 20–40 40–60 60–80 80–100 100–120
Frequency 5 8 x 12 7 8

SECTION-D
Section D consists of 4 questions of 5 marks each.
32. A motorboat whose speed in still water is 9 km/h, goes 15 km downstream and comes

back to the same spot, in a total time of 3 hours 45 minutes. Find the speed of the
stream.
33. In the given figure, E is a point on side AD produced of a parallelogram ABCD and BE

intersects CD at F. Prove that ∆ABE ~ ∆CFB.

34. The lower window of a house is at a height of 2 m above the ground and its upper window

is 4 m vertically above the lower window. At certain instant, the angles of elevation of a
balloon from these windows are observed to be 60° and 30°, respectively. Find the height
of the balloon above the ground.
OR
As observed from the top of a lighthouse, 75 m high from the sea level, the angles of
depression of two ships are 30° and 45°. If one ship is exactly behind the other on the
same side of the lighthouse, find the distance between the two ships.

Sample Papers 209


35. A toy is in the form of a hemisphere surmounted by a right circular cone of the same

base radius as that of the hemisphere. If the radius of the base of the cone is 21 cm and
2
its volume is of the volume of the hemisphere, calculate the height of the cone and
3
the surface area of the toy.
OR
A vessel full of water is in the form of an inverted cone of height 8 cm and the radius of
its top, which is open, is 5 cm. 100 spherical lead balls are dropped into the vessel. One-
fourth of the water flows out of the vessel. Find the radius of a spherical ball.

SECTION-E
Case study based questions are compulsory.
36. Seema is an artificial jewellery seller. She buys them from a dealer at a price based on the

presence or absence of stones as shown in the table below:


Item With Stones Without Stones
Ring 200 100
Earring pair 170 90
Bangle pair 90 120

Based on above information answer the following questions.


(i) For every ring with stones, two rings without stones are purchased by Seema’s

customers. Seema buys rings accordingly from the dealer for ™ 10,000. How many
rings does she buy?
(ii) For every earring pair with stones, two earrings without stones are purchased by

Seema’s customers. Seema buys rings accordingly from the dealer for ™ 7,000.
How many rings does she buy?
(iii) For every bangle pair with stones half bangle pair without stones are purchased

by Seema’s customers. Seema buys bangle pair accordingly from the dealer for
™ 4,500. How many bangle does she buy?
OR
For every ring with stones, the same number of bangle pair without stones are
purchased by Seema’s customers. Seema buys rings and bangle pair without stones
accordingly from the dealer for ™ 16,000. Write the number of rings with stones and
bangle pair without stone she buy.
37. A child was flying a kite, and its string got struck into a tree and touched ground as

shown in figure.
Y

X' X
O

Y'

210 Mathematics–X
Based on the above information answer the following questions.
(i) If one zero of the polynomial x2 – 12x + (3k – 1) is five times than other, then write
the value of k.
(ii) Write the zeros of the polynomial 6x2 – 3 – 7x.

(iii) If both the zeros of a quadratic polynomial ax2 + bx + c are equal and opposite in

sign then write the value of b.


OR
If the polynomial x2 + kx – 15 represents such a curve, with one of its zeros as 3,
then write the value of k.
38. Aisha took a pack of 52 cards. She kept aside all the face cards and shuffled the remaining

cards well.

Based on the above information answer the following questions.


(i) Write the number of total possible outcomes.

(ii) She draw a cars from the well-shuffled pack of remaining cards. What is the

probability that the card is a red card?


(iii) Write the probability of drawing a black card.

OR
What is the probability of getting neither a black card nor a red card?

Answers
1. (c)
2. (b)
3. (a)
4. (d)
5. (a)
6. (b)

7. (c)
8. (c)
9. (a)
10. (c)
11. (c)
12. (d)

13. (d)
14. (c)
15. (c)
16. (a)
17. (d)
18. (b)

22. c , 0 m
2
19. (d) 20. (d) 21. a = 5, b = 1 OR 45
5

24. 16 cm OR 2 39 cm
25. 20 marbles 26. 5 and 15

28. c – 1, m, ^0, 5 h and c1, m


7 13
27. n = 12 OR 18th term or 19th term
2 2

20 225
29. cm 30. 31. x = 10 32. 3 km/h 34. 8 m OR 75 ( 3 – 1) m
3 64

1
35. 28 cm, 5082 cm2 OR cm 36. (i) 75 (ii) 60 (iii) 45 OR 50
2

3 1 1 9
37. (i) 7 (ii) , – units (iii) 0 OR 2 38. (i) 40 (ii) (iii) 0 OR
2 3 2 20

zzz

Sample Papers 211


CBSE SAMPLE QUESTION
PAPER (BASIC)-2023 [SOLVED]

Time allowed: 3 hours Maximum marks: 80


General Instructions:

(i) This Question Paper has 5 Sections A, B, C, D, and E.


(ii) Section A has 20 Multiple Choice Questions (MCQs) carrying 1 mark each.

(iii) Section B has 5 Short Answer-I (SA-I) type questions carrying 2 marks each.

(iv) Section C has 6 Short Answer-II (SA-II) type questions carrying 3 marks each.

(v) Section D has 4 Long Answer (LA) type questions carrying 5 marks each.

(vi) Section E has 3 Case Based integrated units of assessment (4 marks each) with sub-parts of


the values of 1, 1 and 2 marks each respectively.


(vii) All Questions are compulsory. However, an internal choice in 2 Qs of 2 marks, 2 Qs of


3 marks and 2 Questions of 5 marks has been provided. An internal choice has been provided
in the 2 marks questions of Section E.
(viii) Draw neat figures wherever required. Take π = 22/7 wherever required if not stated.

SECTION-A
Section A consists of 20 questions of 1 mark each.

1. If two positive integers p and q can be expressed as p = ab2 and q = a3b; a, b being

prime numbers, then LCM (p, q) is 1


(a) ab
(b) a2b2

3 2
(c) a b
(d) a3b3

2. What is the greatest possible speed at which a man can walk 52 km and 91 km in an

exact number of hours?


(a) 17 km/hours
(b) 7 km/hours

(c) 13 km/hours
(d) 26 km/hours

2
3. If one zero of the quadratic polynomial x + 3x + k is 2, then the value of k is
1
(a) 10
(b) – 10

(c) 5
(d) – 5

212 Mathematics–X
4. Graphically, the pair of equations given by

6x – 3y + 10 = 0
2x – y + 9 = 0
represents two lines which are 1
(a) intersecting at exactly one point.

(b) parallel.

(c) coincident.

(d) intersecting at exactly two points.


5. If the quadratic equation x2 + 4x + k = 0 has real and equal roots, then


1
(a) k < 4
(b) k > 4

(c) k = 4
(d) k ≥ 4

6. The perimeter of a triangle with vertices (0, 4), (0, 0) and (3, 0) is
1
(a) 5 units
(b) 12 units

(c) 11 units
(d) (7 + √5) units

AB BC
7. If in triangles ABC and DEF, = , then they will be similar, when 1
DB FD

(a) ∠B = ∠E
(b) ∠A = ∠D

(c) ∠B = ∠D
(d) ∠A = ∠F

8. In which ratio the y-axis divides the line segment joining the points (5, – 6) and

(–1, – 4)? 1
(a) 1 : 5
(b) 5 : 1

(c) 1 : 1
(d) 1 : 2

9. In the figure, if PA and PB are tangents to the circle with centre O such that

∠APB = 50°, then ∠OAB is equal to 1


A

P 50° O

(a) 25°
(b) 30°

(c) 40°
(d) 50°

Sample Papers 213


1
10. If sin A = , then the value of sec A is: 1
2

2 1
(a) (b)

3 3
(c) 3 (d) 1

11. 3 cos2 A + 3 sin2 A is equal to 1


1
(a) 1 (b)

3
(c) 3 (d) 0

12. The value of cos 1° cos 2° cos 3° cos 4° ........ cos 90° is
1
(a) 1 (b) 0

(c) – 1
(d) 2

13. If the perimeter of a circle is equal to that of a square, then the ratio of their areas is: 1

(a) 22 : 7
(b) 14 : 11

(c) 7 : 22
(d) 11: 14

14. If the radii of two circles are in the ratio of 4 : 3, then their areas are in the ratio of:
1
(a) 4 : 3
(b) 8 : 3

(c) 16 : 9
(d) 9 : 16

15. The total surface area of a solid hemisphere of radius 7 cm is:


1
2 2
(a) 447 cm
p (b) 239 cm
p

2
(c) 174 cm
p (d) 147 cm2
p

16. For the following distribution:


1
Class 0-5 5-10 10-15 15-20 20-25
Frequency 10 15 12 20 9
the upper limit of the modal class is 1
(a) 10 (b) 15

(c) 20 (d) 25

17. If the mean of the following distribution is 2.6, then the value of y is
1
Variable (x) 1 2 3 4 5
Frequency 4 5 y 1 2
(a) 3 (b) 8

(c) 13 (d) 24

214 Mathematics–X
18. A card is selected at random from a well shuffled deck of 52 cards. The probability of

its being a red face card is  1


3 3
(a) (b)
26 13

2 1
(c) (d)
13 2

DIRECTION: In the question numbers 19 and 20, a statement of Assertion (A) is followed by a
statement of Reason (R). Choose the correct option.
(a) Both Assertion (A) and Reason (R) are true and Reason (R) is the correct explanation for


Assertion (A).
(b) Both Assertion (A) and Reason (R) are true and Reason (R) is not the correct explanation for


Assertion (A).
(c) Assertion (A) is true but Reason (R) is false.

(d) Assertion (A) is false but Reason (R) is true.


19. Statement (A) : If HCF of 510 and 92 is 2, then the LCM of 510 and 92 is 32460.

Statement (R) : As HCF (a, b) × LCM (a, b) = a × b.


 1
20. Statement(A) : The ratio in which the line segment joining (2, –3) and (5, 6) internally

divided by x axis is 1 : 2.
mx + nx1 my2 + ny1
Statement(R) : As formula for the internal division is e 2 , o. 1
m+ n m+ n


SECTION-B
Section B consists of 5 questions of 2 marks each.
21. For what values of k will the following pair of linear equations have infinitely many

solutions? 2
kx + 3y – (k – 3) = 0 C
12x + ky – k = 0 D
22. In the figure, altitudes AD and CE of ΔABC intersect

P
each other at the point P. Show that:
(i) ΔABD ~ ΔCBE

(ii) ΔPDC ~ ΔBEC


A B

E
A
OR
In the figure, DE || AC and DF || AE.
D
BF BE
Prove that = . 2
FE EC

B C
F E

Sample Papers 215


23. Two concentric circles are of radii 5 cm and 3 cm. Find the length of the chord of the

larger circle which touches the smaller circle. 2


7 (1 + sin i) (1 – sin i)
24. If cot i = , evaluate . 2
8 (1 + cos i) (1 – cos i)

25. Find the perimeter of a quadrant of a circle of radius 14 cm.


 2
OR
Find the diameter of a circle whose area is equal to the sum of the areas of the two circles
of radii 24 cm and 7 cm.

SECTION-C
Section C consists of 6 questions of 3 marks each.
26. Prove that
5 is an irrational number. 3
2
27. Find the zeroes of the quadratic polynomial 6x – 3 – 7x and verify the relationship

between the zeroes and the coefficients. 3


28. A shopkeeper gives books on rent for reading. She takes a fixed charge for the first

two days, and an additional charge for each day thereafter. Latika paid ™ 22 for a book
kept for six days, while Anand paid ™ 16 for the book kept for four days. Find the fixed
charges and the charge for each extra day. 3
OR
Places A and B are 100 km apart on a highway. One car starts from A and another from
B at the same time. If the cars travel in the same direction at different speeds, they meet
in 5 hours. If they travel towards each other, they meet in 1 hour. What are the speeds
of the two cars?
29. In the figure, PQ is a chord of length 8 cm of a circle of radius 5 cm. The tangents at P

and Q intersect at a point T. Find the length TP.     3

5 cm
8 cm
T O
R

30. Prove that


tan i cot i
+ = 1 + sec i cosec i 3
1 – cot i 1 – tan i

OR
If sin q + cos q = 3 , then prove that tan q + cot q = 1

216 Mathematics–X
31. Two dice are thrown at the same time. What is the probability that the sum of the two

numbers appearing on the top of the dice is 3


(i) 8?

(ii) 13?

(iii) less than or equal to 12?


SECTION-D
Section D consists of 4 questions of 5 marks each.
32. An express train takes 1 hour less than a passenger train to travel 132 km between Mysore

and Bangalore (without taking into consideration the time they stop at intermediate
stations). If the average speed of the express train is 11km/h more than that of the
passenger train, find the average speed of the two trains.  5
OR
A motor boat whose speed is 18 km/h in still water takes 1 hour more to go 24 km
upstream than to return downstream to the same spot. Find the speed of the stream.
33. Prove that If a line is drawn parallel to one side of a triangle to intersect the other two

sides in distinct points, the other two sides are divided in the same ratio. In the figure,
AD AE
find EC if = using the above theorem. 5
DB EC

A
2 cm 3 cm
D E

6 cm

B C

34. A pen stand made of wood is in the shape of a cuboid with four conical depressions to

hold pens. The dimensions of the cuboid are 15 cm by 10 cm by 3.5 cm. The radius of
each of the depressions is 0.5 cm and the depth is 1.4 cm. Find the volume of wood in
the entire stand. 5

Sample Papers 217


OR
Ramesh made a bird-bath for his garden in the shape of a cylinder with a hemispherical
depression at one end. The height of the cylinder is 1.45 m and its radius is 30 cm. Find
the total surface area of the bird-bath.
30 cm

1.45 m

35. A life insurance agent found the following data for distribution of ages of 100 policy

holders. Calculate the median age, if policies are given only to persons having age
18 years onwards but less than 60 years. 5
Age (in years) Number of policy holders
Below 20 2
20–25 4
25–30 18
30–35 21
35–40 33
40–45 11
45–50 3
50–55 6
55–60 2

SECTION-E
Case study based questions are compulsory.
36. Case Study–1

In the month of April to June 2022, the exports of passenger cars from India increased
by 26% in the corresponding quarter of 2021–22, as per a report. A car manufacturing
company planned to produce 1800 cars in 4th year and 2600 cars in 8th year. Assuming
that the production increases uniformly by a fixed number every year.

218 Mathematics–X

Based on the above information answer the following questions.


(i) Find the production in the 1st year.
1
(ii) Find the production in the 12th year.
1
(iii) Find the total production in first 10 years.
2
OR
In which year the total production will reach to 15000 cars?
37. Case Study–2

In a GPS, The lines that run east-west are known as lines of latitude, and the lines
running north-south are known as lines of longitude. The latitude and the longitude of
a place are its coordinates and the distance formula is used to find the distance between
two places. The distance between two parallel lines is approximately 150 km. A family
from Uttar Pradesh planned a round trip from Lucknow (L) to Puri (P) via Bhuj (B) and
Nashik (N) as shown in the given figure below.

Sample Papers 219


Based on the above information answer the following questions using the coordinate
geometry.
(i) Find the distance between Lucknow (L) to Bhuj(B).
1
(ii) If Kota (K), internally divide the line segment joining Lucknow (L) to Bhuj (B) into

3 : 2 then find the coordinate of Kota (K). 1


(iii) Name the type of triangle formed by the places Lucknow (L), Nashik (N) and Puri

(P). 2
OR
Find a place (point) on the longitude (y-axis) which is equidistant from the points
Lucknow (L) and Puri (P).
38. Case Study–3

Lakshaman Jhula is located 5 kilometers north-east of the city of Rishikesh in the


Indian state of Uttarakhand. The bridge connects the villages of Tapovan to Jonk.
Tapovan is in Tehri Garhwal district, on the west bank of the river, while Jonk is in Pauri
Garhwal district, on the east bank. Lakshman Jhula is a pedestrian bridge also used by
motorbikes. It is a landmark of Rishikesh. A group of Class X students visited Rishikesh
in Uttarakhand on a trip. They observed from a point (P) on a river bridge that the
angles of depression of opposite banks of the river are 60° and 30° respectively. The
height of the bridge is about 18 meters from the river.

220 Mathematics–X
Q

P 30°
60°
30° R

A C
B

Based on the above information answer the following questions.


(i) Find the distance PA.
1
(ii) Find the distance PB.
1
(iii) F ind the width AB of the river.
2
OR
Find the height BQ if the angle of the elevation from P to Q be 30°.

zzz

Sample Papers 221


SECTION-A
1. (c) a3b2
1
2. (c) 13 km/hours
1
3. (b) –10
1
4. (b) Parallel
1
5. (c) k = 4
1
6. (b) 12 units
1
7. (c) ∠B = ∠D
1
8. (b) 5 : 1
1
9. (a) 25°
1
2
10. (a)
1
3
11. (c) 3
1
12. (b) 0
1
13. (b) 14: 11
1
14. (c) 16 : 9
1
2
15. (d) 147 π cm

1
16. (c) 20
1
17. (b) 8
1

3
18. (a) 1
26

19. (d) Assertion (A) is false but Reason (R) is true.


1
20. (a) Both Assertion (A) and Reason (R) are true and Reason (R) is the correct explanation


of Assertion (A). 1

SECTION-B
21. For a pair of linear equations to have infinitely many solutions :

a1 b1 c1 k 3 k–3
a2 = b = c2 ⇒ = = ½
12 k k

2
k 3
= ⇒ k2 = 36 ⇒ k=±6 ½
12 k

3 k–3
Also = ⇒ k2 – 6k = 0 ⇒ k = 0, 6 ½
k k

Therefore, the value of k, that satisfies both the conditions, is k = 6.


½

222 Mathematics–X
22. (i) In ΔABD and ΔCBE

C
∠ADB = ∠CEB = 90° ½
D
∠ABD = ∠CBE (Common angle)

⇒ ΔABD ~ ΔCBE (AA criterion) P ½


(ii) In ΔPDC and ΔBEC

∠PDC = ∠BEC = 90° ½


∠PCD = ∠BCE (Common angle) A
E
B

⇒ ΔPDC ~ ΔBEC (AA criterion) ½


OR

B C
F E

In ΔABC, DE || AC
BD BE
= ...(i) (Using BPT) ½
AD EC

In ΔABE, DF || AE
BD BF
= ...(ii) (Using BPT) ½
AD FE

From (i) and (ii)


BD BE BF
= = ½
AD EC FE

BF BE
Thus, = ½
FE EC

23. Let O be the centre of the concentric circle of radii 5 cm and 3 cm respectively. Let
AB
be a chord of the larger circle touching the smaller circle at P.

5 cm
3 cm

A P B

Then AP = PB and OP⊥AB ½


Applying Pythagoras theorem in ∆OPA, we have
OA2 = OP2 + AP2 ⇒ 25 = 9 + AP2 ½
⇒ AP2 = 16 ⇒ AP = 4 cm ½
∴ AB = 2AP = 8 cm ½

Sample Papers 223


]1 + sin ig]1 – sin ig ^1 – sin2 i h
24. Now, = ½
]1 + cos ig]1 – cos ig ^1 – cos2 i h

2
cos2 i cos i
= = d n ½
sin2 i sin i

= cot2
θ ½
7 2 49
=c m = ½
8 64

1
25. Perimeter of quadrant = 2r + × 2r r 1
4

1 22
⇒ Perimeter = 2 ×14 + × ×14 ½
2 7

⇒ Perimeter = 28 + 22 = 50 cm ½
OR
Area of the circle = Area of first circle + Area of second circle
⇒ π R2 = π (r1)2 + π (r1)2

⇒ π R2 = π (24)2 + π (7)2 ⇒ πR2 = 576π +49π ½


2 2
⇒ π R = 625 π ⇒ R = 625 ⇒ R = 25 ½
Thus, diameter of the circle = 2R = 50 cm. 1

SECTION-C
26. Let us assume to the contrary, that 5 is rational. Then we can find a and b ( ≠ 0) such

a
that 5 = (assuming that a and b are co-primes).
b
So, a = 5 b ⇒ a2 = 5b2

1
2
Here 5 is a prime number that divides a then 5 divides a also (Using the theorem, if a
is a prime number and if a divides p2, then a divides p, where a is a positive integer) ½

Thus 5 is a factor of a
Since 5 is a factor of a, we can write a = 5c (where c is a constant). Substituting a = 5c
We get (5c)2 = 5b2 ⇒ 5c2 = b2

½
2
This means 5 divides b so 5 divides b also (Using the theorem, if a is a prime number
and if a divides p2, then a divides p, where a is a positive integer). ½

Hence a and b have at least 5 as a common factor.


But this contradicts the fact that a and b are coprime. This is the contradiction to our
assumption that p and q are co-primes.
So, 5 is not a rational number. Therefore, the 5 is irrational. ½

224 Mathematics–X
27. 6x2 – 7x – 3 = 0

⇒ 6x2 – 9x + 2x – 3 = 0
⇒ 3x(2x – 3) + 1(2x – 3) = 0 ⇒ (2x – 3)(3x + 1) = 0 ½

⇒ 2x – 3 = 0 & 3x + 1 = 0
3 –1
x= & x= ½
2 3

3 –1
Hence, the zeros of the quadratic polynomials are and .
2 3

For verification
– ] –7g

+c m=
– coefficient of x 3 –1 7 7
Sum of zeros = ⇒ ⇒ = 1
2 2 3 6 6 6

coefficient of x
] –3 g
×c m=
constant 3 –1 –1 –1
Product of roots = ⇒ ⇒ = 1
coefficient of x2 2 3 6 2 2

Therefore, the relationship between zeros and their coefficients is verified.


28. Let the fixed charge be ™ x and additional charge be ™ y per day.

Number of days for Latika = 6 = 2 + 4


Hence, Charge x + 4y = 22
x = 22 – 4y …(i) ½
Number of days for Anand = 4 = 2 + 2
Hence, Charge x + 2y = 16
x = 16 – 2y …. (ii) ½
On comparing equation (i) and (ii), we get,
22 – 4y = 16 – 2y ⇒ 2y = 6 ⇒ y=3 1
Substituting y = 3 in equation (1), we get,
x = 22 – 4 (3) ⇒ x = 22 – 12 ⇒ x = 10
Therefore, fixed charge = ™ 10 and additional charge = ` 3 per day 1
OR

A P

Q B
100 km

Let speed of first car be x km/h and second car be y km/h.


AB = 100 km. We know that, Distance = Speed × Time.

AP – BP = 100 ⇒ 5x − 5y = 100 ⇒ x − y = 20 ...(i) ½


AQ + BQ = 100 ⇒ x + y = 100 ...(ii) ½
Adding equations (i) and (ii), we get,
x − y + x + y = 20 +100 ⇒ 2x = 120 ⇒ x = 60 1
Substituting x = 60 in equation (ii), we get, 60 + y = 100 ⇒ y = 40 1
Therefore, the speed of the first car is 60 km/h and the speed of the second car is
40 km/h.

Sample Papers 225


29. Since OT is perpendicular bisector of PQ.

5 cm
8 cm
T O
R

Therefore, PR=RQ=4 cm ½

Now, OR = OP2 – PR2 = 52 – 42 = 3 cm =3cm ½


Now, ∠TPR + ∠RPO = 90° ( a ∠TPO = 90°)
& ∠TPR + ∠PTR = 90° ( a ∠TRP = 90°)
So, ∠RPO = ∠PTR ½
So, TRP ~ PRO
D [By AA Rule of similar triangles]
D ½
TP RP
So, = ½
PO RO

TP 4 20
⇒ = ⇒ TP = cm ½
5 3 3

1
tan i cot i tan i tan i
30. LHS = + = +
1 – cot i 1 – tan i 1 1 – tan i

1–
tan i
tan2 i 1
= + ½
tan i – 1 tan i ]1 – tan ig

tan3 i – 1
= ½
tan i ]tan i – 1g

]tan i – 1g^tan2 i + tan i + 1 h


= ½
tan i ]tan i – 1g

^tan2 i + tan i + 1 h
=
tan i

= tan + 1 + cot q = 1 + tan + cot


θ θ θ ½
sin i cos i
= 1+ +
cos i sin i

sin2 i + cos2 i
= 1+ ½
sin i cos i

1
= 1+ = 1 + sec i cosec i ½
sin i cos i

226 Mathematics–X
OR

sin θ + cos θ = 3 ⇒ (sin θ + cos )2 = 3


θ ½
2 2
⇒ sin θ + cos θ + 2sin θ cos θ =3
⇒ 1 + 2sin
=3 ⇒
θ cos θ sin θ cos θ =1 ½
sin i cos i
Now tan + cot = + 1
cos i sin i
θ θ

sin2 i + cos2 i
= ½
sin i cos i

1 1
= = =1 ½
sin i cos i 1

5
31. (i) P(8) = 1
36

0
(ii) P(13) = =0 1
36

(iii) P(less than or equal to 12) = 1


1

SECTION-D
32. Let the average speed of passenger train = x km/h

and the average speed of express train = (x + 11) km/h. ½


As per given data, time taken by the express train to cover 132 km is 1 hour less than the
passenger train to cover the same distance. Therefore,
132 132

x – x + 11 = 1 1

132 ] x + 11 – xg 132 ×11


x ] x + 11g x ] x + 11g
⇒ =1 ⇒ =1 ½

⇒ 132 × 11 = x(x + 11) ⇒ x2 + 11x – 1452 = 0


⇒ x2 + 44x – 33x – 1452 = 0 1
⇒ x (x + 44) – 33(x + 44) = 0 ⇒ (x + 44)(x – 33) = 0 1
⇒ x = – 44, 33 ½
As the speed cannot be negative, the speed of the passenger train will be 33 km/h and
the speed of the express train will be 33 + 11 = 44 km/h. ½

OR
Let the speed of the stream be x km/h. ½
So, the speed of the boat in upstream = (18 – x) km/h
& the speed of the boat in downstream = (18 + x) km/h ½
distance distance
ATQ, – =1 1
upstream speed downstream speed

24 24
⇒ – =1
18 – x 18 + x

Sample Papers 227


18 + x – ]18 – xg
24 ; E =1 24 = G =1
1 1
]18 – xg . ]18 + xg
⇒ – ⇒ 1
18 – x 18 + x

2x
24 = G =1
]18 – xg . ]18 + xg

⇒ 48x = 324 – x2 ⇒ x2 + 48x – 324 = 0 1


⇒ (x + 54)(x – 6) = 0 ⇒ x = –54 or 6 ½
As speed to stream can never be negative, the speed of the stream is 6 km/h. ½

33. Given: A triangle ABC in which a line parallel to side BC intersects other two sides AB

and AC at D and E respectively.


AD AE
To Prove: = 1
DB EC


Construction: Join BE and CD and then draw DM ⊥ AC and EN ⊥ AB.


1 
Proof: Area of ∆ADE =  base × height  ½
2 


1
So, ar(∆ADE) = (AD × EN)
2

1
and ar(∆BDE) = (DB × EN)
2

1
Similarly, ar(∆ADE) = (AE × DM)
2

1
and ar(∆DEC) = (EC × DM) 1
2


1
AD × EN
ar ^DADEh 2 AD
Therefore, = = …(i)
ar ^DBDEh 1
DB × EN DB
  

2
1
ar ^DADEh 2
AE × DM
AE
and = = …(ii) 1
ar ^DDECh 1 EC


EC × DM
2
Now, ∆BDE and ∆DEC are on the same base DE and between the same parallel lines BC
and DE.
So,    ar(∆BDE) = ar(∆DEC) ...(iii) ½ 

AD AE
Therefore, from (i), (ii) and (iii) we have, = .
DB EC
Second part: We have from given figure,
AD AE 2 3 1 3
= & = & = & EC = 9 cm 1
DB EC 6 EC 3 EC


228 Mathematics–X
1
34. Volume of one conical depression = × r r2 h ½
3

1 22
= × × (0.5)2 × 1.4 cm3
3 7

= 0.366 cm3


3
Volume of 4 conical depressions = 4 × 0.366 cm
= 1.464 cm3
Volume of cuboidal box = L × B × H ½
3
= 15 × 10 × 3.5 cm
= 525 cm3


Remaining volume of box = Volume of cuboidal box – Volume of 4 conical depressions
= 525 cm3 − 1.464 cm3
= 523.5 cm3

1
OR
Let h be height of the cylinder, and r the common
radius of the cylinder and hemisphere.
Then, the total surface area 30 cm
= CSA of cylinder + CSA of hemisphere ½
2
= 2πrh + 2πr = 2πr(h + r) 2
22
= 2× × 30(145 + 30) cm2

1.45 m 1
7

22
= 2× × 30 × 175 cm2

½
7

= 33000 cm2 = 3.3 m2 1


35.

Class interval Number of policy holders (f) Cumulative Frequency (cf)


Below 20 2 2
20–25 4 6
25–30 18 24
30–35 21 45
35–40 33 78
40–45 11 89
45–50 3 92
50–55 6 98
55–60 2 100
1
n
n = 100 ⇒ = 50, Therefore, median class = 35 – 40,
2

Class size, h = 5, Lower limit of median class, l = 35,


frequency f = 33, cumulative frequency cf = 45 1

Sample Papers 229


RS V
SS n – cf WWW
⇒ Median = l + SS 2 WW × h ½
SS W

f W
T X
; E×5
50 – 45
⇒ Median = 35 +
33

25
= 35 + = 35 + 0.76 1½
33

= 35.76

Therefore, median age is 35.76 years. 1

SECTION-E
36.
(i) Since the production increases uniformly by a fixed number every year, the number

of Cars manufactured in 1st, 2nd, 3rd, . . .,years will form an AP.


So, a + 3d = 1800 & a + 7d = 2600 ½
So d = 200 & a = 1200 ½
(ii) t12 = a + 11d ⇒ t30 = 1200 + 11 × 200
½
⇒ t12 = 3400
½

Sn = 62a + ] n – 1g d@ ⇒ Sn =
n 10
(iii) [2 × 1200 + (10 – 1)200] ½
2 2

13
⇒ S10 = 62 ×1200 + 9 × 200@ ½
2

⇒ S10 = 5 × [2400 + 1800]


½
⇒ S10 = 5 × 4200 = 21000 ½
OR
Let in n years the production will reach to 31200.

Sn = 62a + ] n – 1g d@ = 31200 ⇒ 62 ×1200 + ] n – 1g 200@ = 31200


n n
½
2 2

n
⇒ [2 × 1200 + (n − 1)200] = 31200 ⇒ n[12 + (n − 1) ] = 312 ½
2

2
⇒ n + 11n – 312 = 0
⇒ n2 + 24n – 13n – 312 = 0
½
⇒ (n +24)(n – 13) = 0
⇒ n = 13 or – 24. ½
As n can’t be negative. So n = 13

230 Mathematics–X
37.

(i) LB = _ x2 – x1 i + _ y2 – y1 i LB = ]0 – 5g + ]7 – 10g
2 2 2 2
⇒ ½

LB = ]5g + ]3g
2 2
⇒ LB = 25 + 9 & LB = 34

Hence the distance is 34 km. ½

3 × 0 + 2 × 5 3 × 7 + 2 ×10
(ii) Coordinate of Kota (K) is d , n ½
3+2 3+2

0 + 10 21 + 20
n = c 2, m
41
=d , ½
5 5 5

(iii) L(5, 10), N(2,6), P(8,6)


LN = ]2 – 5g2 + ]6 – 10g2 = ] – 3g2 + ] – 4g2 = 9 + 16 = 25 = 5 ½

NP = ]8 – 2g + ]6 – 6g = ]6g + ]0g = 6
2 2 2 2
½

PL = ]8 – 5g2 + ]6 – 10g2 = ]3g2 + ]– 4g2 ⇒ LB = 9 + 16 = 25 = 5 ½


as LN = PL ≠ NP, so Δ LNP is an isosceles triangle. ½
OR

Let A (0, b) be a point on the y – axis then AL = AP.

⇒ ]5 – 0g2 + ]10 – bg2 = ]8 – 0g2 + ]6 – bg2 ½

⇒ (5)2 + (10 − b)2 = (8)2 + (6 − b)2



½
25
⇒ 25 + 100 − 20b + b2 = 64 + 36 − 12b + b2 ⇒ 8b = 25 ⇒ b = ½
8

So, the coordinate on y axis is c 0, m.


25
½
8

Sample Papers 231


38.

P 30°
60° 30° R

A C
B

PC
(i) sin 60° = ½
PA

3 18
⇒ = ⇒ PA = 12 3 m ½
2 PA

PC
(ii) sin 30° = ½
PB

1 18
⇒ = ⇒ PB = 36 m ½
2 PB

PC 18
(iii) tan 60° = ⇒ 3= ⇒ AC = 6 3 m 1
AC AC

PC 1 18
tan 30° = = ⇒ ⇒ CB = 18 3 m ½
CB 3 CB

Width AB = AC + CB = 6 3 + 18 3 = 24 3 m ½
OR
RB = PC =18 m & PR = CB = 18 3 m ½
QR 1 QR
tan 30° = ⇒ = ⇒ QR = 18 m 1
PR

3 18 3
QB = QR + RB = 18 + 18 = 36 m. ½
Hence height BQ is 36m.
zzz

232 Mathematics–X
EXAM BITES

This Pdf Is
Downloaded From
www.exambites.in

Visit www.exambites.in for


More Premium Stuffs,Latest
Books,Test Papers,Lectures etc.
jeeneetadda
jna_official
jeeneetadda

VISIT NOW !!

You might also like